Você está na página 1de 181

2012

MATEMTICA
EXTENSIVO

Professor
Roberto Bayestorff
atendimento.ecp@gmail.com

____________

_____________________________ MATEMTICA _________________________________ Prof. Roberto

MATEMTICA
Prof. Roberto Bayestorff
1

Conjunto

Trata-se de uma coletnea de qualquer coisa. um conceito primitivo; no necessita, portanto, de definio.
Exemplo: conjunto dos nmeros pares positivos: P = {2,4,6,8,10,12, ... }.
Esta forma de representar um conjunto, pela enumerao dos seus elementos, chama-se forma de listagem. O
mesmo conjunto tambm poderia ser representado por uma propriedade dos seus elementos, ou seja, sendo x um
elemento qualquer do conjunto P acima, poderamos escrever:
P = { x | x par e positivo } = { 2,4,6, ... }.
1.1 Relao de pertinncia
Sendo x um elemento do conjunto A , escrevemos x A , onde o smbolo significa "pertence a".
Sendo y um elemento que no pertence ao conjunto A , indicamos esse fato com a notao y A.
O conjunto que no possui elementos , denominado conjunto vazio e representado por ou por { }..
Com o mesmo raciocnio, e, opostamente, ao conjunto vazio, define-se o conjunto ao qual pertencem todos os
elementos, denominado conjunto universo, representado pelo smbolo U.
Assim que, pode-se escrever como exemplos:
= { x; x x} e U = {x; x = x}.
1.2

Subconjunto

Se todo elemento de um conjunto A tambm pertence a um conjunto B, ento dizemos que


A subconjunto de B e indicamos isto por A B.
Notas:
a) todo conjunto subconjunto de si prprio. ( A A )
b) o conjunto vazio subconjunto de qualquer conjunto. (
A)
m
c) se um conjunto A possui m elementos ento ele possui 2 subconjuntos.
d) o conjunto formado por todos os subconjuntos de um conjunto A denominado conjunto das
partes de A e indicado por P(A).
Assim, se A = {c, d} , o conjunto das partes de A dado por P(A) = {
, {c}, {d}, {c,d}}
e) um subconjunto de A tambm denominado parte de A.
1.3 Conjuntos numricos fundamentais
Entendemos por conjunto numrico, qualquer conjunto cujos elementos so nmeros. Existem infinitos conjuntos
numricos, entre os quais, os chamados conjuntos numricos fundamentais, a saber:
Conjunto dos nmeros naturais
N = {0,1,2,3,4,5,6,... }
Conjunto dos nmeros inteiros
Z = {..., -4,-3,-2,-1,0,1,2,3,... }
Obs: evidente que N Z.
Conjunto dos nmeros racionais
Q = {x; x = p/q com p Z , q Z e q 0 }.
Temos ento que nmero racional aquele que pode ser escrito na forma de uma frao p/q onde p e q so nmeros
inteiros, com o denominador diferente de zero.
Lembre-se que no existe diviso por zero!.
So exemplos de nmeros racionais:
__________________________________________________________________________________________________________________________

ATUALIZADO AT MAIO/2012

www.CARREIRAPUBLICA.com.br

(48) 4141-3220

4141-3222

____________

_____________________________ MATEMTICA _________________________________ Prof. Roberto

2/3, -3/7, 0,001=1/1000, 0,75=3/4,

0,333... = 1/3, 7 = 7/1, etc.

Notas:
a) evidente que N Z Q.
b) toda dzima peridica um nmero racional, pois sempre possvel escrever uma dzima peridica na forma de
uma frao.
Exemplo: 0,4444... = 4/9
Conjunto dos nmeros irracionais
I = {x; x uma dzima no peridica}.
Exemplos de nmeros irracionais:
= 3,1415926... (nmero pi = razo entre o comprimento de qualquer circunferncia e o seu dimetro)
2,01001000100001... (dzima no peridica)
3 = 1,732050807... (raiz no exata).
Conjunto dos nmeros reais
R = { x; x racional ou x irracional}.
Notas:
a) bvio que N Z Q R
b) I R
c) I Q = R
d) um nmero real racional ou irracional, no existe outra hiptese!
1.4

Intervalos numricos

Dados dois nmeros reais p e q, chama-se intervalo a todo conjunto de todos nmeros reais compreendidos entre p
e q , podendo inclusive incluir p e q. Os nmeros p e q so os limites do
intervalo, sendo a diferena p - q , chamada amplitude do intervalo.
Se o intervalo incluir p e q , o intervalo fechado e caso contrrio, o intervalo dito aberto.
A tabela abaixo, define os diversos tipos de intervalos.

TIPOS
INTERVALO FECHADO
INTERVALO ABERTO
INTERVALO FECHADO A ESQUERDA
INTERVALO FECHADO DIREITA
INTERVALO SEMI-FECHADO
INTERVALO SEMI-FECHADO
INTERVALO SEMI-ABERTO
INTERVALO SEMI-ABERTO

REPRESENTAO
[p;q] = {x R; p x q}
(p;q) = { x R; p < x < q}
[p;q) = { x R; p x < q}
(p;q] = {x R; p < x q}
[p;
) = {x R; x p}
(- ; q] = { x R; x q}
(-
; q) = { x R; x < q}
(p; ) = { x > p }

OBSERVAO
inclui os limites p e q
exclui os limites p e q
inclui p e exclui q
exclui p e inclui q
valores maiores ou iguais a p.
valores menores ou iguais a q.
valores menores do que q.
valores maiores do que p.

Nota: fcil observar que o conjunto dos nmeros reais, (o conjunto R) pode ser representado na forma de intervalo
como R = ( -
; + ).
1.5

Operaes com conjuntos

1.5.1 Unio ( )
Dados os conjuntos A e B , define-se o conjunto unio A B = { x; x A ou x B}.
Exemplo: {0,1,3} { 3,4,5 } = { 0,1,3,4,5}.
Percebe-se facilmente que o conjunto unio contempla todos os elementos do conjunto A ou do conjunto B.
Propriedades imediatas:
a) A A = A
b) A = A

__________________________________________________________________________________________________________________________

ATUALIZADO AT MAIO/2012

www.CARREIRAPUBLICA.com.br

(48) 4141-3220

4141-3222

____________

_____________________________ MATEMTICA _________________________________ Prof. Roberto

c) A B = B A (a unio de conjuntos uma operao comutativa)


d) A U = U , onde U o conjunto universo.

1.5.2

Interseo ( )

Dados os conjuntos A e B , define-se o conjunto interseo A B = {x; x A e x B}.


Exemplo: {0,2,4,5} { 4,6,7} = {4}.
Percebe-se facilmente que o conjunto interseo contempla os elementos que so comuns aos conjuntos A e B.
Propriedades imediatas:
a) A A = A
b) A =
c) A B = B A ( a interseo uma operao comutativa)
d) A U = A onde U o conjunto universo.
So importantes tambm as seguintes propriedades :
P1. A ( B C ) = (A B) ( A C) (propriedade distributiva)
P2. A ( B C ) = (A B ) ( A C) (propriedade distributiva)
P3. A (A B) = A (lei da absoro)
P4. A (A B) = A (lei da absoro)
Obs: Se A B = , ento dizemos que os conjuntos A e B so disjuntos.
1.5.3 Diferena
A - B = {x ; x A e x B}
Observe que os elementos da diferena so aqueles que pertencem ao primeiro conjunto, mas no pertencem ao
segundo.
Exemplos:
{ 0,5,7} - {0,7,3} = {5}.
{1,2,3,4,5} - {1,2,3} = {4,5}.
Propriedades imediatas:
a) A - = A
b) - A =
c) A - A =
d) A - B B - A ( a diferena de conjuntos no uma operao comutativa).
1.5.3.1 Complementar de um conjunto
Trata-se de um caso particular da diferena entre dois conjuntos. Assim , que dados dois conjuntos A e B, com a
condio de que B A , a diferena A - B chama-se, neste caso, complementar de B em relao a A .
Simbologia: CAB = A - B.
Caso particular: O complementar de B em relao ao conjunto universo U, ou seja , U - B , indicado pelo smbolo B'
.Observe que o conjunto B' formado por todos os elementos que no pertencem ao conjunto B, ou seja:
B' = {x; x B}. bvio, ento, que:
a) B B' =
b) B B' = U
c) ' = U
d) U' =
1.6 Partio de um conjunto
Seja A um conjunto no vazio. Define-se como partio de A, e representa-se por part(A), qualquer subconjunto do
conjunto das partes de A (representado simbolicamente por P(A), que satisfaz simultaneamente, s seguintes
condies:
1 - nenhuma dos elementos de part(A) o conjunto vazio.
__________________________________________________________________________________________________________________________

ATUALIZADO AT MAIO/2012

www.CARREIRAPUBLICA.com.br

(48) 4141-3220

4141-3222

____________

_____________________________ MATEMTICA _________________________________ Prof. Roberto

2 - a interseo de quaisquer dois elementos de part(A) o conjunto vazio.


3 - a unio de todos os elementos de part(A) igual ao conjunto A.
Exemplo: Seja A = {2, 3, 5}
Os subconjuntos de A sero: {2}, {3}, {5}, {2,3}, {2,5}, {3,5}, {2,3,5}, e o conjunto vazio - .
Assim, o conjunto das partes de A ser:
P(A) = { {2}, {3}, {5}, {2,3}, {2,5}, {3,5}, {2,3,5}, }
Vamos tomar, por exemplo, o seguinte subconjunto de P(A):
X = { {2}, {3,5} }
Observe que X uma partio de A - cuja simbologia part(A) - pois:
a) nenhum dos elementos de X .
b) {2} {3, 5} =
c) {2} U {3, 5} = {2, 3, 5} = A
Sendo observadas as condies 1, 2 e 3 acima, o conjunto X uma partio do conjunto A.
Observe que Y = { {2,5}, {3} } ; W = { {5}, {2}, {3} }; S = { {3,2}, {5} } so outros exemplos de parties do conjunto A.
Outro exemplo: o conjunto Y = { {0, 2, 4, 6, 8, ...}, {1, 3, 5, 7, ...} } uma partio do conjunto N dos nmeros
naturais, pois {0, 2, 4, 6, 8, ...} {1, 3, 5, 7, ...} = e {0, 2, 4, 6, 8, ...} U {1, 3, 5, 7, ...} = N .

1.7 Nmero de elementos da unio de dois conjuntos


Sejam A e B dois conjuntos, tais que o nmero de elementos de A seja n(A) e o nmero de elementos de B seja
n(B).
Nota: o nmero de elementos de um conjunto, tambm conhecido com cardinal do conjunto.
Representando o nmero de elementos da interseo A B por n(A B) e o nmero de elementos da unio A B
por n(A B) , podemos escrever a seguinte frmula:
n(A B) = n(A) + n(B) - n(A B)
2 Fraes

O smbolo

significa a:b, sendo a e b nmeros naturais e b diferente de zero.

Chamamos:

de frao;

a de numerador

Se a mltiplo de b, ento

b de denominador.

um nmero natural.

Veja um exemplo:

A frao

igual a 8:2. Neste caso, 8 o numerador e 2 o denominador. Efetuando a diviso de 8 por 2,

obtemos o quociente 4. Assim,

um nmero natural e 8 mltiplo de 2.

Durante muito tempo, os nmeros naturais foram os nicos conhecidos e usados pelos homens. Depois
comearam a surgir questes que no poderiam ser resolvidas com nmeros naturais. Ento surgiu o conceito de
nmero fracionrio.
2.1 O significado de uma frao

__________________________________________________________________________________________________________________________

ATUALIZADO AT MAIO/2012

www.CARREIRAPUBLICA.com.br

(48) 4141-3220

4141-3222

____________

_____________________________ MATEMTICA _________________________________ Prof. Roberto

Algumas vezes,

um nmero natural. Outras vezes, isso no acontece. Neste caso, qual o significado de

?
Uma frao envolve a seguinte idia: dividir algo em partes iguais. Dentre essas partes, consideramos uma ou
algumas, conforme nosso interesse.

Exemplo: Monique comeu de um chocolate. Isso significa que, se dividssemos o chocolate em 4 partes iguais,
Monique teria comido 3 partes:

Na figura acima, as partes pintadas seriam as partes comidas por Monique, e a parte branca a parte que sobrou
do chocolate.

2.2 Classificao das fraes

Frao prpria: o numerador menor que o denominador:

Frao imprpria: o numerador maior ou igual ao denominador.

Frao aparente: o numerador mltiplo do denominador.

2.3 Fraes equivalentes


Fraes equivalentes so fraes que representam a mesma parte do todo.

Exemplo:

so equivalentes

Para encontrar fraes equivalentes devemos multiplicar o numerador e o denominador por um mesmo nmero
natural, diferente de zero.

Exemplo: obter fraes equivalentes frao

__________________________________________________________________________________________________________________________

ATUALIZADO AT MAIO/2012

www.CARREIRAPUBLICA.com.br

(48) 4141-3220

4141-3222

____________

_____________________________ MATEMTICA _________________________________ Prof. Roberto

Portanto as fraes

so algumas das fraes equivalentes a

2.4 Simplificao de fraes

Uma frao equivalente a


frao

, com termos menores,

pelo fator comum 3. Dizemos que a frao

. A frao

foi obtida dividindo-se ambos os termos da

uma frao simplificada de

A frao no pode ser simplificada, por isso chamada de frao irredutvel. A frao
simplificada porque 3 e 4 no possuem nenhum fator comum

no pode ser

2.5 Nmeros fracionrios


Seria possvel substituir a letra X por um nmero natural que torne a sentena abaixo verdadeira?
5.X=1
Substituindo X, temos:
X por 0 temos: 5.0 = 0
X por 1 temos: 5.1 = 5.
Portanto, substituindo X por qualquer nmero natural jamais encontraremos o produto 1. Para resolver esse
problema temos que criar novos nmeros. Assim, surgem os nmeros fracionrios.
Toda frao equivalente representa o mesmo nmero fracionrio.

Portanto, uma frao


fracionrio

(n diferente de zero) e todas fraes equivalentes a ela representam o mesmo nmero

Resolvendo agora o problema inicial, conclumos que X =

, pois

2.6 Adio e subtrao de nmeros fracionrios


Temos que analisar dois casos:
1) denominadores iguais
Para somar fraes com denominadores iguais, basta somar os numeradores e conservar o denominador.
Para subtrair fraes com denominadores iguais, basta subtrair os numeradores e conservar o denominador.
Observe os exemplos:

__________________________________________________________________________________________________________________________

ATUALIZADO AT MAIO/2012

www.CARREIRAPUBLICA.com.br

(48) 4141-3220

4141-3222

____________

_____________________________ MATEMTICA _________________________________ Prof. Roberto

2) denominadores diferentes
Para somar fraes com denominadores diferentes, uma soluo obter fraes equivalentes, de
denominadores iguais ao m.m.c dos denominadores das fraes. Exemplo: somar as fraes

Obtendo o m.m.c dos denominadores temos m.m.c(5,2) = 10.

(10:5).4 = 8
(10:2).5 = 25

Resumindo: utilizamos o m.m.c para obter as fraes equivalentes e depois somamos normalmente as fraes,
que j tero o mesmo denominador, ou seja, utilizamos o caso 1.

2.7 Multiplicao e diviso de nmeros fracionrios


Na multiplicao de nmeros fracionrios, devemos multiplicar numerador por numerador, e denominador por
denominador, assim como mostrado nos exemplos abaixo:

Na diviso de nmeros fracionrios, devemos multiplicar a primeira frao pelo inverso da segunda, como
mostrado no exemplo abaixo:

2.8 Potenciao e radiciao de nmeros fracionrios


Na potenciao, quando elevamos um nmero fracionrio a um determinado expoente, estamos elevando o
numerador e o denominador a esse expoente, conforme os exemplos abaixo:

__________________________________________________________________________________________________________________________

ATUALIZADO AT MAIO/2012

www.CARREIRAPUBLICA.com.br

(48) 4141-3220

4141-3222

____________

_____________________________ MATEMTICA _________________________________ Prof. Roberto

Na radiciao, quando aplicamos a raiz quadrada a um nmero fracionrio, estamos aplicando essa raiz ao
numerador e ao denominador, conforme o exemplo abaixo:

2.9 Dzimas peridicas

H fraes que no possuem representaes decimal exata. Por exemplo:


1/3 = 0,333333.....

19/90 = 0,211111...

Aos numerais decimais em que h repetio peridica e infinita de um ou mais algarismos, d-se o nome de
numerais decimais peridicos ou dzimas peridicas.
Numa dzima peridica, o algarismo ou algarismos que se repetem infinitamente, constituem o perodo dessa
dzima.
As dzimas classificam-se em dzimas peridicas simples e dzimas peridicas compostas. Exemplos:

(perodo: 5)

(perodo: 3)

(perodo: 12)

So dzimas peridicas simples, uma vez que o perodo apresenta-se logo aps a vrgula.
So dzimas peridicas compostas, uma vez que entre o perodo e a vrgula existe uma parte no peridica.
Observaes:
Consideramos parte no peridica de uma dzima o termo situado entre a vrgula e o perodo. Exclumos portanto da
parte no peridica o inteiro.
Podemos representar uma dzima peridica das seguintes maneiras:

2.10 Geratriz de uma dzima peridica


possvel determinar a frao (nmero racional) que deu origem a uma dzima peridica. Denominamos esta
frao de geratriz da dzima peridica.
Procedimentos para determinao da geratriz de uma dzima:
2.10.1 Dzima simples
A geratriz de uma dzima simples uma frao que tem para numerador o perodo e para denominador tantos
noves quantos forem os algarismos do perodo.

__________________________________________________________________________________________________________________________

ATUALIZADO AT MAIO/2012

www.CARREIRAPUBLICA.com.br

(48) 4141-3220

4141-3222

____________

_____________________________ MATEMTICA _________________________________ Prof. Roberto

Exemplos:

2.10.2 Dzima Composta


A geratriz de uma dzima composta uma frao da forma

, onde

n a parte no peridica seguida do perodo, menos a parte no peridica.


d tantos noves quantos forem os algarismos do perodo seguidos de tantos zeros quantos forem os algarismos da
parte no peridica.
Exemplos:

2.11 Mltiplos e divisores naturais


2.11.1 Mltiplo e divisor de um nmero natural

Dizemos que um nmero natural n divide um nmero natural m, quando m : n no deixa resto, ou seja, a diviso
exata. Representamos simbolicamente: n|m. Nestas condies, n um divisor de m e m um mltiplo de n.
Exemplos:
2 divide 16 ou seja, 2|16 porque 16:2 = 8 e resto = zero. Portanto, 2 divisor de 16 e 16 mltiplo de 2.
5 divide 35 ou seja, 5|35 porque 35:5 = 7 e resto = zero. Portanto, 5 divisor de 35 e 35 mltiplo de 5.
7 divide 105 ou seja, 7|105 porque 105:7 = 15 e resto = zero. Portanto, 7 divisor de 105 e 105 mltiplo de 7.
Observaes:
a) O conjunto dos divisores naturais de n ser representado por D(n).
Exemplos:
D(3) = {1,2,3}
D(20) = {1,2,4,5,10,20}
D(6) = {1,2,3,6}
b) O conjunto dos mltiplos naturais de n ser representado por M(n).
Exemplos:
M(2) = {0,2, 4, 6, 8, ...}
M(5) = {0,5,10,15, ...}
c) Os mltiplos de 2 so denominados nmeros pares. Os demais nmeros naturais so denominados nmeros
mpares. Assim, denotando por P o conjunto dos nmeros pares e por I o conjunto dos nmeros mpares, poderemos
escrever:
P = {0, 2, 4, 6, 8, 10, 12, ... }
__________________________________________________________________________________________________________________________

ATUALIZADO AT MAIO/2012

www.CARREIRAPUBLICA.com.br

(48) 4141-3220

4141-3222

____________

_____________________________ MATEMTICA _________________________________ Prof. Roberto

I = {1, 3, 5, 7, 9, 11, 13, ... }


Observa-se que ambos os conjuntos so infinitos.

2.11.2 Propriedades imediatas

P1) A unidade (ou seja, o nmero 1) divide qualquer nmero natural ou seja, n/1, para todo n natural.
P2) Zero no divide nenhum nmero natural, ou seja, no existe diviso por zero. Imagine se voc tivesse que dividir
dez objetos por zero pessoas. Claro que isto no seria possvel. Grave bem isto: a diviso por zero no existe.
P3) Todo nmero natural diferente de zero, divide o nmero zero, ou seja, para
n 0, 0/n, para todo n no nulo.
P4) Todo nmero natural diferente de zero, divide a si prprio, ou seja, para n 0, n / n para todo n no nulo. Esta
propriedade conhecida como propriedade reflexiva.
P5) Sendo m, n e p trs nmeros naturais, se m/ p e p / n ento m/ n. Esta propriedade conhecida com
propriedade transitiva.
Exemplo:
2 divide 6 pois 6 : 2 = 3 (diviso exata).
6 divide 42 pois 42 : 6 = 7 (diviso exata). Logo,
2 divide 42. Realmente, 42 :2 = 21 (diviso exata).
P6) Todo nmero natural no nulo, mltiplo de si mesmo. Isto decorre da propriedade P4.
P7) Zero mltiplo de todo nmero natural no nulo. Isto decorre da propriedade P3.
( p 1) primo quando ele s possui dois divisores: ele prprio e a unidade. Caso contrrio, o nmero
composto.
Assim, se o conjunto dos divisores naturais de p, representado por D(p), for igual a
D(p) = {1, p}, p um nmero primo.
Ora, os divisores de 2, so apenas a unidade (1) e ele mesmo (2). Logo, 2 um nmero primo. Portanto, 2 o nico
nmero natural primo que par.
Sendo o conjunto dos nmeros primos, poderemos escrever:
= {2, 3, 5, 7, 11, 13, 17, 19, 23, 29, 31, 37, 41, 47, 53, 57, 59, 61, ..., 359, ... , }
O conjunto dos nmeros primos infinito.
Todo nmero composto pode ser escrito como um produto de nmeros primos. Isto conhecido como Teorema
Fundamental da Aritmtica TFA.
Exemplos:
12 = 3.2.2
15 = 3.5
49 = 7.7
105 = 7.5.3
4
240 = 2.120 = 2.5.2.2.2.3 = 5.2 .3
Na prtica, podemos usar o seguinte esquema:
Seja o caso de 240 acima. Teremos:
240 |2
120 |2
60 |2
30 |2
15 |3
5|5
1|

10

__________________________________________________________________________________________________________________________

ATUALIZADO AT MAIO/2012

www.CARREIRAPUBLICA.com.br

(48) 4141-3220

4141-3222

____________

_____________________________ MATEMTICA _________________________________ Prof. Roberto

Ento: 240 = 2.2.2.2.3.5 = 24.3.5


A decomposio de um nmero em fatores primos, conhecida tambm como fatorao , j que o nmero decomposto em
fatores de uma multiplicao.

3 m.d.c. Mximo divisor comum

Dois nmeros naturais sempre tm divisores comuns. Por exemplo: os divisores comuns de 12 e 18 so 1,2,3 e 6.
Dentre eles, 6 o maior. Ento chamamos o 6 de mximo divisor comum de 12 e 18 e indicamos m.d.c.(12,18) =
6.
O maior divisor comum de dois ou mais nmeros chamado de mximo divisor comum
desses nmeros. Usamos a abreviao m.d.c.
Alguns exemplos:
mdc (6,12) = 6
mdc (12,20) = 4
mdc (20,24) = 4

3.1 Clculo do m.d.c.


Um modo de calcular o m.d.c. de dois ou mais nmeros utilizar a decomposio desses nmeros em fatores
primos.
1) decompomos os nmeros em fatores primos;
2) o m.d.c. o produto dos fatores primos comuns.
Acompanhe o clculo do m.d.c. entre 36 e 90:
36 = 2 x 2 x 3 x 3
90 =
2x3x3x5
O m.d.c. o produto dos fatores primos comuns => m.d.c.(36,90) = 2 x 3 x 3
Portanto m.d.c.(36,90) = 18.
Escrevendo a fatorao do nmero na forma de potncia temos:
2
2
36 = 2 x 3
2
90 = 2 x 3 x5
2
Portanto m.d.c.(36,90) = 2 x 3 = 18.
O m.d.c. de dois ou mais nmeros, quando fatorados, o produto dos fatores comuns a eles,
cada um elevado ao menor expoente.

3.1.1 Clculo do m.d.c. pelo processo das divises sucessivas


Nesse processo efetuamos vrias divises at chegar a uma diviso exata. O divisor desta diviso o m.d.c.
Acompanhe o clculo do m.d.c.(48,30).
Regra prtica:
1) dividimos o nmero maior pelo nmero menor;
48 / 30 = 1 (com resto 18)

__________________________________________________________________________________________________________________________

ATUALIZADO AT MAIO/2012

www.CARREIRAPUBLICA.com.br

(48) 4141-3220

4141-3222

11

____________

_____________________________ MATEMTICA _________________________________ Prof. Roberto

2) dividimos o divisor 30, que divisor da diviso anterior, por 18, que o resto da diviso anterior, e assim
sucessivamente;
30 / 18 = 1 (com resto 12)
18 / 12 = 1 (com resto 6)
12 / 6 = 2 (com resto zero - diviso exata)
3) O divisor da diviso exata 6. Ento m.d.c.(48,30) = 6.
3.1.2 Nmeros primos entre si
Dois ou mais nmeros so primos entre si quando o mximo divisor comum
desses nmeros 1.
Exemplos:
Os nmeros 35 e 24 so nmeros primos entre si, pois mdc (35,24) = 1.
Os nmeros 35 e 21 no so nmeros primos entre si, pois mdc (35,21) = 7.

3.2 Propriedade do m.d.c.

Dentre os nmeros 6, 18 e 30, o nmero 6 divisor dos outros dois. Neste caso, 6 o m.d.c.(6,18,30). Observe:
6=2x3
2
18 = 2 x 3
30 = 2 x 3 x 5
Portanto m.d.c.(6,18,30) = 6
Dados dois ou mais nmeros, se um deles divisor de todos os outros, ento ele o
m.d.c. dos nmeros dados.
Dados dois nmeros naturais a e b no nulos, define-se o mximo divisor comum m.d.c. como sendo o maior
natural que divide simultaneamente a e b.
O m.d.c. de dois nmeros ser indicado por (a, b).
bvio que se tivermos o m.d.c. de n nmeros naturais a1, a2, a3, ... , an , indicaremos por
(a1, a2, a3, ... , an)
Exemplos:
Determine o m.d.c. dos naturais 10 e 14, ou seja, determine (10, 14).
Os divisores positivos de 10 so: 1, 2, 5, 10.
Os divisores positivos de 14 so: 1, 2, 7, 14.
Os divisores comuns, so, portanto: 1 e 2.
Portanto, o mximo divisor comum igual a 2 e, indicamos: (10, 14) = 2.
Pode-se indicar tambm como: m.d.c.(10,14) = 2. Preferimos a primeira forma, por ser mais sinttica.
Determine (4, 10, 14, 60), ou seja, o m.d.c. dos nmeros naturais 4,10,14 e 60.
Os divisores positivos de 4 so: 1, 2, 4
Os divisores positivos de 10 so: 1, 2, 5, 10
Os divisores positivos de 14 so: 1, 2, 7, 14
Os divisores positivos de 60 so: 1, 2, 3, 4, 5, 6, 10, 12, 15, 60

12

__________________________________________________________________________________________________________________________

ATUALIZADO AT MAIO/2012

www.CARREIRAPUBLICA.com.br

(48) 4141-3220

4141-3222

____________

_____________________________ MATEMTICA _________________________________ Prof. Roberto

Os divisores comuns so, portanto: 1 e 2.


Portanto o m.d.c igual a 2, ou seja: (4, 10, 14, 60) = 2
O mtodo de decomposio de um nmero num produto de fatores primos, sugere uma nova forma para o clculo do
m.d.c. de dois nmeros naturais no nulos, a e b, ou seja, para o clculo de (a,b).
Assim, seja calcular o m.d.c. de 408 e 240.
Como j vimos acima, temos:
3

408 = 2.2.2.3.17 = 2 .3.17


4
240 = 2.2.2.2.3.5 = 2 .3.5
Tomando os fatores comuns elevados aos menores expoentes, teremos:
3

(408, 240) = 2 .3 = 8.3 = 24 , que o m.d.c. procurado.


Portanto, (408, 240) = 24.
O m.d.c. do exemplo anterior, poderia ser tambm determinado pelo mtodo das divises sucessivas, cujo dispositivo
prtico mostrado a seguir:

408
168

1
240
72

1
168
24

2
72
0

3
24

Para entender o dispositivo prtico, basta observar que:


408:240 = 1
240:168 = 1
168:72 = 2
72:24 = 3

com resto 168


com resto 72
com resto 24
com resto zero.

Portanto o m.d.c. procurado igual a 24, conforme j tnhamos visto antes.


Nota: Se o m.d.c. de dois nmeros naturais a e b for igual unidade, ou seja,
(a,b) = 1, dizemos que a e b so primos entre si, ou que a e b so co-primos.
Ou seja:
(a, b) = 1 a e b so primos entre si (co-primos).
Exemplo: (7, 5) = 1 5 e 7 so primos entre si.

4 m.m.c. Mnimo mltiplo comum

4.1 Mltiplo de um nmero natural


Como 24 divisvel por 3 dizemos que 24 mltiplo de 3.
24 tambm mltiplo de 1, 2, 3, 4, 6, 8, 12 e 24.
Se um nmero divisvel por outro, diferente de zero, ento dizemos que ele
mltiplo desse outro.
__________________________________________________________________________________________________________________________

ATUALIZADO AT MAIO/2012

www.CARREIRAPUBLICA.com.br

(48) 4141-3220

4141-3222

13

____________

_____________________________ MATEMTICA _________________________________ Prof. Roberto

Os mltiplos de um nmero so calculados multiplicando-se esse nmero pelos nmeros naturais.


Exemplo: os mltiplos de 7 so:
7x0 , 7x1, 7x2 , 7x3 , 7x4 , ... = 0 , 7 , 14 , 21 , 28 , ...
Observaes importantes:
1) Um nmero tem infinitos mltiplos
2) Zero mltiplo de qualquer nmero natural
4.2 Mnimo mltiplo comum (m.m.c.)
Dois ou mais nmeros sempre tm mltiplos comuns a eles.
Vamos achar os mltiplos comuns de 4 e 6:
Mltiplos de 6: 0, 6, 12, 18, 24, 30,...
Mltiplos de 4: 0, 4, 8, 12, 16, 20, 24,...
Mltiplos comuns de 4 e 6: 0, 12, 24,...
Dentre estes mltiplos, diferentes de zero, 12 o menor deles. Chamamos o 12 de mnimo mltiplo
comum de 4 e 6.
O menor mltiplo comum de dois ou mais nmeros, diferente de zero, chamado de mnimo mltiplo
comum desses nmeros. Usamos a abreviao m.m.c.
Clculo do m.m.c.
Podemos calcular o m.m.c. de dois ou mais nmeros utilizando a fatorao. Acompanhe o clculo do m.m.c.
de 12 e 30:
1) decompomos os nmeros em fatores primos
2) o m.m.c. o produto dos fatores primos comuns e no-comuns:
12 = 2 x 2 x 3
30 = 2 x 3 x 5
m.m.c (12,30) = 2 x 2 x 3 x 5
Escrevendo a fatorao dos nmeros na forma de potncia, temos:
2
12 = 2 x 3
30 = 2 x 3 x 5
2
m.m.c (12,30) = 2 x 3 x 5
O m.m.c. de dois ou mais nmeros, quando fatorados, o produto dos fatores comuns e
no-comuns a eles, cada um elevado ao maior expoente.
Processo da decomposio simultnea
Neste processo decompomos todos os nmeros ao mesmo tempo, num dispositivo como
mostra a figura ao lado. O produto dos fatores primos que obtemos nessa decomposio o
m.m.c. desses nmeros. Ao lado vemos o clculo do m.m.c.(15,24,60)
Portanto, m.m.c.(15,24,60) = 2 x 2 x 2 x 3 x 5 = 120

4.2.1 Propriedade do m.m.c.


Entre os nmeros 3, 6 e 30, o nmero 30 mltiplo dos outros dois. Neste caso, 30 o m.m.c.(3,6,30).
Observe:

14

__________________________________________________________________________________________________________________________

ATUALIZADO AT MAIO/2012

www.CARREIRAPUBLICA.com.br

(48) 4141-3220

4141-3222

____________

_____________________________ MATEMTICA _________________________________ Prof. Roberto

m.m.c.(3,6,30) = 2 x 3 x 5 = 30
Dados dois ou mais nmeros, se um deles mltiplo de todos os outros, ento ele o
m.m.c. dos nmeros dados.
Considerando os nmeros 4 e 15, que so primos entre si. O m.m.c.(4,15) igual a 60, que o produto de 4 por
15. Observe:

m.m.c.(4,15) = 2 x 2 x 3 x 5 = 60
Dados dois nmeros primos entre si, o m.m.c. deles o produto desses nmeros.

5 Potncia de expoente natural

Sendo a um nmero real e n um nmero natural maior ou igual a 2, definimos a n-sima (ensima) potncia de a
como sendo:

an = a.a.a.a.a. ^ .a
onde o fator a repetido n vezes, ou seja, o produto possui n fatores.
n

Denominamos o fator a de base e n de expoente; a a n-sima potncia de a.


Portanto, potncia um produto de n fatores iguais.
A operao atravs da qual se obtm uma potncia, denominada potenciao.
Exemplos:
2

7 = 7.7 = 49
5
2 = 2.2.2.2.2 = 32
3
6 = 6.6.6 = 216
7
10 = 10.10.10.10.10.10.10 = 10000000 (dez milhes)
6
10 = 10.10.10.10.10.10 = 1000000 (um milho)
Nota:
n
Observe que a potncia 10 igual a 1 seguido de n zeros.
10
Assim, por exemplo, 10 = 10000000000 (dez bilhes).

5.1 Convenes
0

a) potncia de expoente zero: a = 1


0
0
0
Exemplos: 4567 = 1; 243 = 1; (- 2001) = 1

__________________________________________________________________________________________________________________________

ATUALIZADO AT MAIO/2012

www.CARREIRAPUBLICA.com.br

(48) 4141-3220

4141-3222

15

____________

_____________________________ MATEMTICA _________________________________ Prof. Roberto


1

b) potncia de expoente unitrio : a


1
1
Exemplos: 23 = 23; 2001 = 2001.

=a

As potncias de expoente 2 e 3 recebem nomes especiais, a saber:


2
a = a.a, lido como a ao quadrado.
3
a = a.a.a, lido como a ao cubo.

5.2

Propriedades das potncias

So vlidas as seguintes propriedades das potncias de expoentes naturais, facilmente demonstrveis:


m

m+n

P1) a . a = a
5 3
5+3
8
Exemplo: 2 .2 = 2 = 2 = 2.2.2.2.2.2.2.2 = 256
m

m-n

P2) a : a = a
7 4
7-4
3
Exemplo: 5 :5 = 5 = 5 = 5.5.5 = 125
m n

m.n

P3) (a ) = a
2 3
2.3
6
Exemplo: (4 ) = 4 = 4 = 4.4.4.4.4.4 = 4096
m

P4) a .b = (a.b)
3 3
3
3
Exemplo: 2 .4 = (2.4) = 8 = 8.8.8 = 512
m

P5) a :b = (a:b)
4 4
4
4
Exemplo: 12 :3 = (12:3) = 4 = 4.4.4.4 = 256
-n

P6) a = 1/a
-2
2
Exemplo: 5 = 1/5 = 1/5.5 = 1/25
-n

0-n

Esta propriedade decorre de P2, ou seja: a = a /a = a

-n

=a .

Nota: estas propriedades tambm so vlidas para expoentes reais.


Exerccio:
Calcule o valor da expresso a seguir:
3

5 -2

A = {[(2 .2 : 4 )] }
Desenvolvimento:
7

2 3 5 -2

6 5 -2

1 5 -2

-10

A = {[(2 : (2 ) )] } = {[2 : 2 ] } = {[2 ] } = 2

= 1/2

10

= 1/1024

6 Radicais

A forma mais genrica de um radical :


onde c = coeficiente, n = ndice e A = radicando.
O radical acima lido como: c raiz n-sima (ensima) de A.
Se n = 2, costuma-se no representar o nmero 2 e l-se como c raiz quadrada de A.

16

__________________________________________________________________________________________________________________________

ATUALIZADO AT MAIO/2012

www.CARREIRAPUBLICA.com.br

(48) 4141-3220

4141-3222

____________

_____________________________ MATEMTICA _________________________________ Prof. Roberto

Se n = 3, l-se o radical como c raiz cbica de A.


Exemplos:
que lido com 5 raiz cbica de 25, onde 5 o coeficiente, 3 o ndice e 25, o radicando.
3
10 que lido como 3 raiz quadrada de 10, onde 3 o coeficiente, 2 (no indicado, por conveno) o ndice e 10,
o radicando.

6.1 Potncia de expoente fracionrio

Exemplo:
A propriedade acima decorre de:
m/n
n
m/n n
n
m
Seja x = a . Podemos escrever: x = (a ) e, da, x = a de onde vem, extraindo-se a raiz n-sima de ambos os
membros:

6.2 Introduzindo o coeficiente num radical


Uma importante propriedade dos radicais a seguinte:

Exemplo:

Portanto, para introduzir um coeficiente num radical, basta elevar este coeficiente a um expoente igual ao seu
ndice.
Esta propriedade bastante til tambm, para a simplificao de radicais, pois s vezes, a depender do tipo de
problema que est sendo abordado, pode tornar-se necessrio percorrer o caminho inverso. Assim, por exemplo,

6.3 Raiz de raiz


Outra propriedade muito importante dos radicais a que segue:

Exemplo:

__________________________________________________________________________________________________________________________

ATUALIZADO AT MAIO/2012

www.CARREIRAPUBLICA.com.br

(48) 4141-3220

4141-3222

17

____________

_____________________________ MATEMTICA _________________________________ Prof. Roberto

A operao com radicais denominada radiciao e, esta operao a inversa da potenciao.


Isto decorre de:
Exemplos:
Como 2 elevado a 4 igual a 16, dizemos que 2 uma raiz quarta de 16.
Como 3 elevado a 2 igual a 9, dizemos que 3 uma raiz quadrada de 9.
Como 5 elevado a 3 igual a 125, dizemos que 5 uma raiz cbica de 125, etc

6.4 Radiciao
6.4.1 Potenciao de Radicais
Observando as potencias, temos que:

De modo geral, para se elevar um radical a um dado expoente, basta elevar o radicando quele expoente.
Exemplos:

6.4.2 Diviso de Radicais


Segundo as propriedades dos radicais, temos que:

De um modo geral, na diviso de radicais de mesmo ndice, mantemos o ndice e dividimos os radicais: Exemplos:

Se os radicais forem diferentes, devemos reduzi-los ao mesmo ndice e depois efetue a operao. Exemplos:

6.4.3 Racionalizao de denominadores


Considere a frao:

18

que seu denominador um nmero irracional.

__________________________________________________________________________________________________________________________

ATUALIZADO AT MAIO/2012

www.CARREIRAPUBLICA.com.br

(48) 4141-3220

4141-3222

____________

_____________________________ MATEMTICA _________________________________ Prof. Roberto

Vamos agora multiplicar o numerador e o denominador desta frao por

Observe que a frao equivalente

, obtendo uma frao equivalente:

possui um denominador racional.

A essa transformao, damos o nome de racionalizao de denominadores.


A racionalizao de denominadores consiste, portanto, na obteno de um frao com denominador racional,
equivalente a uma anterior, que possua um ou mais radicais em seu denominador.
Para racionalizar o denominador de uma frao devemos multiplicar os termos desta frao por uma expresso com
radical, denominado fator racionalizante, de modo a obter uma nova frao equivalente com denominador sem
radical.

6.4.4 Principais casos de racionalizao


1 Caso: O denominador um radical de ndice 2: Exemplos:

o fator racionalizante de

, pois

=a

2 Caso: O denominador um radical de ndice diferente de 2. Exemplos:

o fator racionalizante de
o fator racionalizante de
o fator racionalizante de
o fator racionalizante de

6.5 Exerccios
01.

Determine o m.m.c. entre 24 e 40

02. Determine o m.d.c. entre 24 e 40


__________________________________________________________________________________________________________________________

ATUALIZADO AT MAIO/2012

www.CARREIRAPUBLICA.com.br

(48) 4141-3220

4141-3222

19

____________

_____________________________ MATEMTICA _________________________________ Prof. Roberto

03. (CORREIOS 2006) O mnimo mltiplo comum entre os nmeros 6 e 8, elevado ao quadrado :
a) 1.024
b) 576 c) 324
d) 256
e) 144.
04. ( B.B 007) A proposio funcional existem nmeros que so divisvel por 2 e por 3 verdadeira para todos os
elementos do conjunto { 2 , 3 , 9 , 10 , 15 , 16 }

05. Quanto aos nmeros pares 0, 2, 4 e 8, CORRETO afirmar que:


a) esto em progresso aritmtica de razo 2.
b) esto em progresso geomtrica de razo de 2.
c) so potncias consecutivas da base 2.
d) so mltiplos consecutivos de 2.
e) tm mximo divisor comum igual a 2.
06. ( CORREIOS 2006 ) O mximo divisor comum de 6 e 9, elevado quarta potncia .
a) 81 b) 71

c) 61

d) 51

e) 27

07.

O nmero 18900 apresenta n divisores naturais, onde n igual a:


a) 12 b) 36
c) 72
d) 18
e) 24

08.

O nmero 2 . 3 . 5 tem 120 divisores. Qual o valor de a?

09.
(UFSC) Um pas lanou em 02/05/2000 os satlites artificiais A, B e C com as tarefas de fiscalizar o
desmatamento em reas de preservao, as nascentes dos rios e a pesca predatria no Oceano Atlntico. No dia
03/05/2000 podia-se observ-los alinhados, cada um em uma rbita circular diferente, tendo a Terra como centro. Se
os satlites A, B e C levam, respectivamente, 6, 10 e 9 dias para darem uma volta completa em torno da Terra, ento
o nmero de dias para o prximo alinhamento :
10.

Trs atletas correm numa pista circular e gastam, respectivamente, 2,4min, 2,0min e 1,6min para completar
uma volta na pista. Eles partem do mesmo local e no mesmo instante. Aps algum tempo, os trs atletas se
encontram, pela primeira vez, no local da largada. Nesse momento, o atleta MAIS VELOZ estar completando:
a) 12 voltas.
b) 15 voltas. c) 18 voltas. d) 10 voltas.

11.
Em 1982 ocorreu uma conjuno entre os planetas Jpiter e Saturno, o que significa que podiam ser vistos
bem prximos um do outro quando avistados da Terra. Se Jpiter e Saturno do uma volta completa ao redor do Sol
aproximadamente a cada 12 e 30 anos, respectivamente, em qual dos anos seguintes ambos estiveram em
conjuno no cu da Terra?
a) 1840 b) 1852 c) 1864 d) 1922 e) 960
12.

Trs viajantes partem num mesmo dia de uma cidade A. Cada um desses trs viajantes retorna cidade A
exatamente a cada 30, 48 e 72 dias, respectivamente. O nmero mnimo de dias transcorridos para que os trs
viajantes estejam juntos novamente na cidade A :
a) 144. b) 240. c) 360. d) 480. e) 720.

13.
Considere dois rolos de barbante, um com 96 m e outro com 150 m de comprimento. Pretende-se cortar todo
o barbante dos dois rolos em pedaos de mesmo comprimento. O menor nmero de pedaos que poder ser obtido
a) 38 b) 41 c) 43 d) 52
e) 55
14. Entre algumas famlias de um bairro, foi distribudo um total de 144 cadernos, 192 lpis e 216 borrachas. Essa
distribuio foi feita de modo que o maior nmero possvel de famlias fosse contemplado e todas recebessem o
mesmo nmero de cadernos, o mesmo nmero de lpis e o mesmo nmero de borrachas, sem haver sobra de
qualquer material. Nesse caso, o nmero de CADERNOS que cada famlia ganhou foi:
a) 4
b) 6
c) 8
d) 9

15. Para levar os alunos de certa escola a um museu, pretende-se formar grupos que tenham iguais quantidades de
alunos e de modo que em cada grupo todos sejam do mesmo sexo. Se nessa escola estudam 1.350 rapazes e 1.224
garotas e cada grupo dever ser acompanhado de um nico professor, o nmero mnimo de professores necessrios
para acompanhar todos os grupos nessa visita :
a) 18 b) 68
c) 75
d) 126 e) 143
20

__________________________________________________________________________________________________________________________

ATUALIZADO AT MAIO/2012

www.CARREIRAPUBLICA.com.br

(48) 4141-3220

4141-3222

____________

_____________________________ MATEMTICA _________________________________ Prof. Roberto

16. nibus da linha 572 passam pelo Largo do Machado de 7 em 7 minutos. Se um nibus passou s 15h 42min,
quem chegar ao Largo do Machado s 18h 3min esperar quantos minutos pelo prximo nibus?
a) 1
b) 2
c) 4
d) 5 e) 6
17. ( CORREIOS 2006 ) Um carpinteiro adquiriu seis dzias de pregos e quatro dezenas de tachas. A quantidade
total de peas adquiridas igual a
a) 110. b) 100. c) 112. d) 122. e) 132.
18. Um trem faz o percurso da estao A at a estao B em 2 horas, 22 minutos e 35 segundos. Se o trem chegou
na estao B exatamente s 10 horas, o seu horrio de partida da estao A foi:
a) 6 horas, 38 minutos e 35 segundos
b) 6 horas, 37 minutos e 25 segundos
c) 7 horas, 37 minutos e 25 segundos
d) 7 horas, 38 minutos e 35 segundos
e) 7 horas, 22 minutos e 25 segundos

19.

Uma pessoa tem 36 moedas. Um quarto dessas moedas de 25 centavos, um tero de 5 centavos, e as
restantes so de 10 centavos. Essas moedas totalizam a quantia de:
a) 8,75 b) 7,35 c) 5,45 d) 4,35

20.
( FEPESE-2006) Assinale a alternativa correta. As cidades A e B so ligadas por uma nica rodovia. Entre
as cidades A e B tem-se o acesso para a cidade C. De A at B so 131 quilmetros; de A at o acesso para C so 25
quilmetros. Quantos so os quilmetros de B at o acesso para C, usando-se a mesma rodovia?
a) 10 km b) 25 km c) 106km d)131km e) 156km
21.
( CORREIOS2006 ) Calcule o valor da expresso matemtica ( 0,48 2 ) 10 + ( 3,6 4 ) e assinale a
alternativa que contm o resultado.
a) 4,6. b) 2,6. c) 3.
d) 3,3.
e) 3,6.
22.

Ao

analisar

as

notas

fiscais

de

uma

firma,

auditor

deparou-se

com

seguinte

situao:
No era possvel ver o nmero de metros vendidos, mas sabia-se que era um nmero inteiro. No valor total, s
apareciam os dois ltimos dos trs algarismos da parte inteira.
Com as informaes anteriores, o auditor concluiu que a quantidade de cetim, em metros, declarada nessa nota foi:
a) 16
b) 26 c) 36
d) 46

23. A escada representada na figura tem sete degraus e altura 1,54m. A altura de cada degrau, em cm, :

a) 18

b) 22

c) 25

d) 28

24. (ACAFE) Suponha que uma companhia de gua cobre o consumo residencial pela seguinte tabela:
__________________________________________________________________________________________________________________________

ATUALIZADO AT MAIO/2012

www.CARREIRAPUBLICA.com.br

(48) 4141-3220

4141-3222

21

____________

_____________________________ MATEMTICA _________________________________ Prof. Roberto

Faixa de consumo
3
por m
0 - 10
11 - 25
mais de 25

Valor em reais
3
por m
1,20
2,00
2,50
3

O proprietrio de uma residncia, que num determinado ms consumiu 27m de gua, pagar, em reais:
a) 55,00 b) 67,50 c) 54,00 d) 45,00 e) 47,00

25. (UFSC) Suponha que em uma determinada espcie de animais os indivduos tenham seus primeiros filhotes aos
8 meses, e que a partir de ento para cada adulto da populao nasam, em mdia, 3 filhotes a cada 3 meses. Se no
incio de janeiro nascerem os primeiros 12 filhotes de 4 indivduos com os quais se esteja iniciando uma criao, qual
ser o nmero provvel de indivduos que a populao atingir no incio de outubro, no havendo mortes?
26. Visando evitar o desperdcio de gua, uma Companhia de Saneamento estipulou vrias faixas de consumo para
cobrar do usurio. Vejamos:

O clculo do valor a ser pago efetuado distribuindo-se o volume de gua gasto por faixa de consumo. Os primeiros
3
3
10 m so calculados segundo a 1 faixa. O excedente, ou seja, os prximos 10 m so cobrados pela segunda faixa,
3
o excedente pela 3 faixa e assim sucessivamente. Se uma famlia consumir 30 m , vai pagar:
a) R$ 22,07
b) R$ 29,77
c) R$ 42,62
d) R$ 53,85
e) R$ 77,10

27. ( FEPESE 2006) Observe os dois anncios de duas lojas de eletrodomsticos:

Assinale a resposta correta:


a ) A loja B tem a melhor oferta pois ao comparar os dois valores a que apresenta o menor valor total.
b ) A loja A tem a melhor oferta pois ao comparar os dois valores a que apresenta o menor valor total.
c ) As duas lojas apresentam ofertas iguais.
e ) Para comprar dois celulares na loja A vai ser necessrio pagar o valor total de R$ 265,00
d ) O valor da mensalidade do celular da loja B de R$ 12,00.

28. Antnio possui um carro a lcool que consome 1 litro de combustvel a cada 8km percorridos, enquanto Jos
possui um carro a gasolina cujo consumo de 12km por litro. Sabendo-se que o litro de lcool custa R$ 1,14 e o litro
de gasolina R$ 1,60, e que Jos e Antnio dispem da mesma quantidade de dinheiro, quantos quilmetros ir
percorrer Jos, tendo em vista que Antnio percorreu 320km?
22

__________________________________________________________________________________________________________________________

ATUALIZADO AT MAIO/2012

www.CARREIRAPUBLICA.com.br

(48) 4141-3220

4141-3222

____________

_____________________________ MATEMTICA _________________________________ Prof. Roberto

29. (UFSC) No livro O Cdigo da Vinci, de Dan Brown, no local onde o corpo de Jacques Sauniere encontrado,
alguns nmeros esto escritos no cho. Estes nmeros fazem parte da Seqncia de Fibonacci, que uma
seqncia infinita de nmeros em que cada termo, a partir do terceiro, igual soma dos dois termos que
imediatamente o antecedem. Assim, o dcimo primeiro termo da Seqncia de Fibonacci 1, 1, 2, 3, 5, 8, 13, ... o
nmero:
30. Em certo pas, o limite legal para que uma pessoa, aps consumo de bebida alcolica, possa conduzir um carro
de 80 miligramas de lcool para cada 100 mililitros de sangue.
A tabela a seguir mostra a quantidade de lcool que ainda permanece no sangue de uma pessoa a cada hora aps o
consumo, em funo da quantidade retida inicialmente. Todos os valores so dados em mg de lcool/100mL de
sangue.

a) Sabe-se que o consumo de uma garrafa de cerveja provoca uma reteno inicial de 30mg de lcool/100 mL de
sangue. Suponha que Luiz beba trs garrafas de cerveja.
Determine a quantidade de lcool no sangue de Luiz trs horas aps o consumo.
b) Sabe-se que o consumo de uma dose de licor provoca uma reteno inicial de 25mg de lcool/100mL de sangue.
Suponha que Virgnia deseja dirigir seu carro, sem infringir a lei, duas horas aps consumir algumas doses de licor.
Determine o nmero mximo de doses de licor que Virgnia poder tomar.

31. ( ENEM) No quadro a seguir esto as contas de luz e gua de uma mesma residncia. Alm do valor a pagar,
3
cada conta mostra como calcul-lo, em funo do consumo de gua (em m ) e de eletricidade (em kWh). Observe
que, na conta de luz, o valor a pagar igual ao consumo multiplicado por um certo fator. J na conta de gua, existe
uma tarifa mnima e diferentes faixas de tarifao.

Suponha que, no prximo ms, dobre o consumo de energia eltrica dessa residncia. O novo valor da conta ser de:
a) R$ 55,23
b) R$ 106,46 c) R$ 802,00
d) R$ 100,00 e) R$ 22,90

32. O REAL ENFERRUJOU

"(...) as moedas 1 e 5 centavos oxidam antes do previsto (...) At agora, apenas 116 milhes entre os sete bilhes de
moedas em circulao tm nova
roupagem lanada pelo governo no dia 1 julho (...)"
(ISTO , 09/09/98)

__________________________________________________________________________________________________________________________

ATUALIZADO AT MAIO/2012

www.CARREIRAPUBLICA.com.br

(48) 4141-3220

4141-3222

23

____________

_____________________________ MATEMTICA _________________________________ Prof. Roberto

Desses 116 milhes de moedas, metade de R$0,50, a metade do nmero restante de R$0,10, a metade do que
sobrou de R$0,05 e as ltimas moedas so de R$0,01.
O total de moedas de R$0,01 corresponde, em reais, a:
a) 14.500
b) 29.000
c) 145.000
d) 290.000

33. Desejo enviar uma mercadoria para Buenos Aires e consultei uma transportadora sobre preos de
areo de cargas. Recebi como resposta o fax a seguir.

transporte

Destino: Buenos Aires/Argentina


Cia Area: VIASUL
Material: Bagagem desacompanhada
Frete areo:
at 45kg
R$ 2,60 por quilo
mais de 45kg, at 100kg R$ 2,30 por quilo
mais de 100kg R$ 2,10 por quilo
Despesas adicionais obrigatrias:
Agentes de Cargas: R$ 100,00
INFRAERO: R$ 10,00
Obs.: Os Agentes de Cargas so os encarregados do embarque e desembarque das mercadorias nos respectivos
aeroportos.
Se a mercadoria que desejo enviar tem 78,5kg, quanto deverei desembolsar?
a) R$ 310,10 b) R$ 290,55 c) R$ 264,65 d) R$ 201,10 e) R$ 180,55

34. (ENEM) No grfico esto representados os gols marcados e os gols sofridos por uma equipe de futebol nas dez
primeiras partidas de um determinado campeonato

Considerando que, neste campeonato, as equipes ganham 3 pontos para cada vitria, 1 ponto por
empate e 0 ponto em caso de derrota, a equipe em questo, ao final da dcima partida, ter acumulado um nmero
de pontos igual a
a) 15. b) 17. c) 18. d) 20.
e) 24.

35. Observe as figuras 1 e 2. Todos os robs so formados por estrelinhas iguais a da figura 1.
O rob 1 no tem pernas. Todos os outros tm pernas, alm de ps, tronco e cabea, como voc pode ver na figura
3.
Veja a tabela.
Apesar do rob 4 no estar desenhado, foi possvel completar a tabela com o nmero de estrelinhas necessrias.
Isso, porque existe uma regra para construo dos robs.
Descubra que regra essa e faa os itens a seguir.
a) Na tabela, complete quantas estrelinhas sero usadas para formar, no rob 5, cada p, cada perna, a altura do
tronco, a largura do tronco e a cabea.
b) Quantas estrelinhas sero necessrias para formar cada p do rob 21?
c) Quantas estrelinhas sero necessrias para formar cada perna do rob 10?
d) Quantas estrelinhas sero necessrias para formar a altura do tronco do rob 11?
e) Quantas estrelinhas sero necessrias para formar a largura do tronco do rob 11?
24

__________________________________________________________________________________________________________________________

ATUALIZADO AT MAIO/2012

www.CARREIRAPUBLICA.com.br

(48) 4141-3220

4141-3222

____________

_____________________________ MATEMTICA _________________________________ Prof. Roberto

36. Calcule:
2
a)
+5
3
37. Determine:
2 4
a) .
5 3

b)

3 1

2 4

c)

5 3
:
2 4

c) 5 +

d) 3

1 2 1
+ +
3
5 30 tem-se:
38. Efetuando-se
23
30
529
21
23
b)
c)
a)
900
23
30
39. Ache o valor numrico de

a) 0

b)

3 1

2 3

d) 1

2 1
:
3 5

e)

7
23

a 1
1
, a = e b = 5
3+b
2

1
1
c)
4
4

d)1

e) 2

40. O IDH procura refletir a qualidade de vida dos cidados. No entanto, atravs de sua anlise no possvel
averiguar algumas desigualdades como o caso, por exemplo, dos dados sobre trabalho feminino divulgados pela
OIT (Organizao Internacional do Trabalho). Segundo a organizao, na dcada de 90 do sculo XX, o trabalho
feminino correspondeu a 2/3 do total de horas trabalhadas no planeta enquanto o trabalho masculino apenas 1/3.
Com base nesses dados vlido afirmar que, em termos de horas trabalhadas, as mulheres trabalharam em relao
aos homens
a) a tera parte.
b) menos da metade.
c) a metade.
d) o dobro.
e) o triplo.
41. ( B.B 2007) A proposio funcional para qualquer x tem se x2 > x verdadeira para todos os valores de x
3
1
5
que esto no conjunto 5 , , 3 , , 2 ,
2
2
2
42. Em julho de 2007, sero realizados os XV Jogos Pan-Americanos na cidade do Rio de Janeiro. Durante o
perodo dos jogos, o Centro Aqutico Nacional, que est sendo construdo na Barra da Tijuca, com capacidade para
10 mil pessoas, ser utilizado do seguinte modo:
Em 1/6 do perodo ocorrero apenas competies de saltos ornamentais;
Em 2/9 do perodo ocorrero as competies de nado sincronizado;
Em 1/3 do perodo ocorrero as competies de natao;
Nos demais dias, no haver atividades no centro aqutico.
__________________________________________________________________________________________________________________________

ATUALIZADO AT MAIO/2012

www.CARREIRAPUBLICA.com.br

(48) 4141-3220

4141-3222

25

____________

_____________________________ MATEMTICA _________________________________ Prof. Roberto

Sendo assim, responda:


a) Que atividade esportiva ocupar por mais tempo o Centro Aqutico Nacional durante os jogos: nado sincronizado
ou natao? Por qu?
b) Em que frao do perodo dos jogos no haver atividades no centro aqutico?
c) A tabela abaixo apresenta os dias dos XV Jogos Pan-Americanos em que ocorrero as competies de dois dos
seguintes esportes: saltos ornamentais, nado sincronizado ou natao.
Descubra quais so esses dois esportes e use-os para completar, adequadamente, os espaos em branco na tabela.

43. Snia coleciona papis de carta. Sabendo que 2/7 das folhas ela ganhou de sua me, 3/5 ela ganhou de suas
avs e outras 4 folhas restantes ela ganhou de suas amigas, determine o nmero de folhas da coleo de Snia.
44. Claudete leu 3/5 de um livro e ainda faltam 48 pginas para ela terminar de ler o livro todo. Quantas pginas
desse livro ela j leu? Qual o total de folhas que tem esse livro?
45. ( CORREIOS 2006 ) Se dividirmos 0,144 por 1,2 obteremos o valor:
a) 1,21.
b) 0,2. c) 0,12. d) 12,1. e) 11.
46. ( FEPESE 2006) Assinale a operao que est com a resposta correta.
1 1
a) 0,5 + 0,34 = 0,74 b)
+ = 0,25
4 4
1 1 2
1
c) + =
d) 12 + = 12,5 e) 0,2 + 0,2 + 0,2 = 0,8
4 2 6
2
47. ( PRF-1998) A distncia entre duas cidades A e B de 265 metros e o nico posto de gasolina entre elas
encontra-se a 3/5 desta distncia, partindo de A. O total de quilmetros a serem percorridos da cidade B at este
posto de:
a)

57

b) 106 c) 110 d) 159

e) 212

48. (FGV) No oramento da Prefeitura de uma determinada cidade, a verba mensal total de R$ 24.000.000,00
destinada Educao. Sabe-se que 1/8 deste montante dirigido Educao Infantil e 3/8 ao Ensino Fundamental.
Sabe-se tambm que 1/3 dos recursos dirigidos Educao Infantil so destinados ao pagamento de salrios e o
restante para outras despesas. Sabe-se ainda que 2/5 dos recursos dirigidos ao Ensino Fundamental destinam-se ao
pagamento de salrios e o restante para outras despesas. Pede-se:
a) Quais so, em reais, os recursos destinados para a Educao Infantil e para o Ensino Fundamental?
b) Quais so as fraes da verba total correspondentes aos recursos para pagamento de salrios em cada um dos
dois nveis de Ensino?
c) Qual a frao da verba total correspondente a outras despesas para a Educao Infantil?
d) Mantidos os nmeros do enunciado, exceto a ltima frao (2/5) referente aos recursos dirigidos para o pagamento
de salrios do Ensino Fundamental, pergunta-se qual dever ser o novo valor desta ltima frao para que os
recursos para pagamento de salrios sejam iguais nos dois nveis de Ensino?

49. Em uma cidade, 5/8 da populao torce pelo time A e, entre esses torcedores, 2/5 so mulheres. Se o nmero
de torcedores do sexo masculino, do time A, igual a 120.000, a populao dessa cidade constituda por
a) 340.000 habitantes. b) 320.000 habitantes.
c) 300.000 habitantes. d) 280.000 habitantes.
e) 260.000 habitantes.

26

__________________________________________________________________________________________________________________________

ATUALIZADO AT MAIO/2012

www.CARREIRAPUBLICA.com.br

(48) 4141-3220

4141-3222

____________

_____________________________ MATEMTICA _________________________________ Prof. Roberto

50. (FEPESE 2006) Um operrio levou trs dias para fazer um servio de manuteno. No primeiro dia, concluiu
1
3
do trabalho total e no segundo dia, concluiu
do trabalho total. Assinale a alternativa que indica a frao do
4
5
trabalho total que foi realizado no terceiro dia.
a)

3
30

b)

17
20

c)

4
9

d)

8
20

e)

3
20

51. Numa escola foi feito um levantamento para saber quais os tipos de calados mais usados pelas crianas. Foi
obtido o seguinte resultado: um tero usa sandlias; um quarto usa tnis; um quinto usa sapatos, e os 52 restantes
usam outros tipos de calados.
Pode-se concluir que, pelos tipos de calados encontrados, h nessa escola um total de
a) 240 crianas. b) 250 crianas.
c) 260 crianas. d) 270 crianas.
e) 280 crianas.
52. ( FEPESE 2006) Um avio decola em Florianpolis, com destino a Salvador, s 8h, num vo que tem a
3
do tempo total de durao do vo,
durao total de 4 horas. Supondo que o avio precise reabastecer aps
5
assinale a alternativa que indica o horrio em que o avio dever pousar para reabastecimento.
a) 10 h 40 min. b) 11 h.

c) 10 h 24 mim. d) 10 h 44 min. e) 09 h 44 min.

53. (FEPESE 2006) Um grupo de pessoas saiu para caminhar em um parque, aproveitando o sol
do outono. Utilizando uma equao, pode-se dizer que o nmero de pessoas que iniciou a caminhada igual ao
quadrado da metade do nmero de pessoas que iniciou a caminhada. Aps uma parada,

9
do nmero de pessoas
12

que iniciou a caminhada continuou at o final da aventura. Assinale a alternativa que indique a razo entre o nmero
de pessoas que iniciou a caminhada e o nmero
de pessoas que chegou ao final da aventura.

3
3
4
b) 3
c)
d)
e) 4
12
4
3
54. (FEPESE 2006) Uma construtora est executando uma obra e prev a sua realizao em quatro etapas. A
a)

tabela abaixo relaciona a frao do servio total que foi executado, aps a concluso de cada uma das trs primeiras
etapas:
ETAPAS
Frao do servio total executado
Etapa 1

Etapa 2
Etapa 3

2
5
1
3
1
5

Assinale a alternativa que indica a frao do servio total de execuo da obra que deve ser realizada na etapa 4
para que a obra seja concluda.
a)

14
4
9
1
b)
c)
d)
15
13
13
15

e)

2
75

55. Uma pessoa tem 36 moedas. Um quarto dessas moedas de 25 centavos, um tero de 5 centavos, e as
restantes so de 10 centavos. Essas moedas totalizam a quantia de:
a) 8,75
b) 7,35
c) 5,45 d) 4,35
56. Um pai tem o triplo da idade de seu filho que est com 10 anos. A soma das idades dos dois, em anos, quando o
filho tiver a idade atual do pai ser
a) 70
b) 80 c) 90
d) 100
57. ( CORREIOS 2006 ) Assinale a alternativa correta para o resultado do nmero 12 elevado ao expoente zero:
a) -12

b) 12

c) 0

d) -1

e) 1

__________________________________________________________________________________________________________________________

ATUALIZADO AT MAIO/2012

www.CARREIRAPUBLICA.com.br

(48) 4141-3220

4141-3222

27

____________

_____________________________ MATEMTICA _________________________________ Prof. Roberto

58. ( CORREIOS 2006 ) O valor da expresso [(-2)-(-3)]+[(-2)4+(-2)] igual a


a) -8.

b) 8.

59. Calcule: a) 5

c) 16.

b) ( -5 )

d) -27. e) 27.

c) ( -2 ) d) 3

60. O valor da expresso 66 + 66 + 66 + 66 + 66 + 66 :


6

a) 6

b) 6

c) 7

61. Qual a metade de 222 ?


11

22

a) 2

b) 1

11

c) 1

21

d) 2

e) 243

36

d) 6

f) 1 g)

2

3

1
h) (0,5)

i)

(2 )

2 3

e) 36

e) n.d.a

2
3

62. Calcule 8 + 9 0 ,5
63. Uma calculadora apresentava, em sua tela, o resultado da soma dos gastos do ms realizados por um pai
"coruja" que permitiu a seu filho apertar algumas teclas, alterando esse resultado. O pai observou que o menino
havia apertado as teclas, uma nica vez, na ordem mostrada na figura 1.

Para recuperar o resultado que estava na tela, o pai dever apertar as teclas.

64. Calcule as seguintes razes:


a) 81
b) 400
c)
4
d) 256

216

65. Simplifique as seguintes razes:


a) 12
b) 192
c)

d)

108

48

66. A expresso 3 75 2 48 27 + 3 12 vale:


a) 10 3
b) 12 3
c) 6 3
d) 4 3

e)

15

67. Simplifique a expresso:


68. Efetue: 4 5

54 150 + 2 24 + 6

1
20 + 125 1445
2

69. Racionalize os denominadores.


3
8
5
a)

b)

28

c)

2 2

2 5

__________________________________________________________________________________________________________________________

ATUALIZADO AT MAIO/2012

www.CARREIRAPUBLICA.com.br

(48) 4141-3220

4141-3222

____________

d)

_____________________________ MATEMTICA _________________________________ Prof. Roberto

2
5 2

70. Uma indstria consegue modelar a funo do nmero de produtos montados em um dia, por um grupo de
3
4

funcionrios, da seguinte forma: P(t) = 1000 .t


sendo P o nmero de produtos montados e t o nmero de horas
trabalhadas. Determine a alternativa que representa o nmero de produtos montados em 90 minutos.
a)

1000 x 4 1,5

c) 1000 x 3

b)

1000 x 4 2,2,5

3,375 d) 1000 x 4 3,375


3

e) 1000 x 90 4

71. Calculando o valor da expresso:


13
13
b)
12
100
4
e)
3
a)

c)

10
3

1 1
2

(1 0,7 ) + 0,75 obtemos:


5
2 4

19
d)
12

5 1 2 1 9 1
1
72. Resolvendo a expresso 2 obtemos:
3
6 3 5 4 2
a)

17
5
3
12
b)
c)
d)
12
17
85
17

e)

3
8
2

1 12
1 3 2

73. O valor da expresso 7 + 30 + + 7 0 :


5 35
2 4 3

17
27
34
23
43
a)
b)
c)
d)
e)
6
6
6
6
6
2

2
1
1
+
3
2
4
74. Calcule: 1

1
1
4 2 2
3
+
9
5
2
140
243
1
a)
b)
c)
3
56
35
81
d)
e) 1
4

75. Escreva a frao decimal equivalente a cada numeral decimal a seguir:


a) 12,4 b) 7,52 c) 0,003

d) 10,8 e) 1,887

76. Quais dos seguintes nmeros so racionais e quais so irracionais:


a)

0,2222...

b)

7,1317...

d)7,212121...
e)

g) 3 + 16
h) 1 +

__________________________________________________________________________________________________________________________

ATUALIZADO AT MAIO/2012

www.CARREIRAPUBLICA.com.br

(48) 4141-3220

4141-3222

29

____________

c)

_____________________________ MATEMTICA _________________________________ Prof. Roberto

0,123456...

f)

64

i) 3 27

77. Escreva sob a forma de frao:


a)
b)
c)

0,555...
0,373737...
1,3333...

78. Resolva as expresses:


a) 5 0,444...
b) 0,3 . 0,333...

1
3 + 0,666...
0,60606...
d)
0,121212...
c) 2 +

79. Se a = 0,444... e b = 0,333... , ento b. a igual a:


1
2
2
7
5
a)
b)
c)
d)
e)
9
9
3
9
3
80. Sendo : A = { 1 , 2 , 5 , 6 } e B = { 1 , 4 , 5 , 6, 7 } , determine:
AB
AB
AB
(AB)B
(A B ) (A B )

a)
b)
c)
d)
e)

81. Coloque V ou F , considerando-se os conjuntos


A = { x IN, x < 4 },
B = { x Z, 2x + 3 = 7 },
2
C = { x IR, x + 5x + 6 = 0 },
( )AB=A
( )AB={2,3}
( )A B={0,1,3}
( ) A C = IR
82. Uma prova com duas questes foi aplicada em uma classe de 40 alunos. Aps a correo, constatou-se que 10
alunos acertaram as duas questes, 25 alunos acertaram a primeira questo e 20 alunos acertaram a segunda
questo.
O nmero de alunos que erraram as duas questes :
a) 5
b) 15 c) 10 d) 20 e) 30
83. Sejam A e B dois conjuntos, onde (A B) possui 134 elementos e (A B) possui 49 elementos. Se A possui
15 elementos a mais do que B, ento o nmero de elementos de A :

84. Numa pesquisa de preferncia pelas disciplinas de Matemtica (M), Fsica (F), e Portugus (P), feitas aos alunos
de um colgio, foram colhidos os seguintes resultados:
Disciplina
M
F
P
MeF
MeP
FeP
M, F e P
Alunos

400

300

200

150

50

30

20

O nmero total de alunos entrevistados foi de:


a) 900 b) 690 c) 650 d) 500 e) 140

85. Dos 540 alunos inscritos em uma academia, 200 fazem musculao, 250 natao e o restante, de 240, fazem
outras modalidades de esportes.
Assinale a alternativa correta.
30

__________________________________________________________________________________________________________________________

ATUALIZADO AT MAIO/2012

www.CARREIRAPUBLICA.com.br

(48) 4141-3220

4141-3222

____________

_____________________________ MATEMTICA _________________________________ Prof. Roberto

a) O nmero de alunos que fazem apenas musculao 100.


b ) O nmero de alunos que fazem apenas natao 50.
c ) 450 alunos fazem natao ou musculao.
d ) 150 alunos fazem natao e musculao.
e ) 300 fazem apenas uma modalidade de esporte.

GABARITO
01.
120
02.
8
03.
b
04.
Falso
05.
e
06.
a
07.
c
08.
5
09.
90
10.
b
11.
d
12.
e
13.
b
14.
b
15.
e
16.
e
17.
c
18.
c
19.
d
20.
c
21.
d
22.
c
23.
b
24.
e
25.
88
26.
c
27.
a
28.
342 km
29.
89
30.
a) 75 mg/mL
31.
b
32. c
33. b
34. c

b) 3 doses

35.

Observe a tabela a seguir:

__________________________________________________________________________________________________________________________

ATUALIZADO AT MAIO/2012

www.CARREIRAPUBLICA.com.br

(48) 4141-3220

4141-3222

31

____________

b) 2 estrelinhas.
d) 11 estrelinhas.

_____________________________ MATEMTICA _________________________________ Prof. Roberto

c) 9 estrelinhas.
e) 13 estrelinhas.

a) 17/3 b) 5/4 c) 37/6


a) 8/15 b) 10/3 c) -1/3
d
c
d
Falso
a) Natao: 1/3 = 3/9
Nado Sincronizado: 2/9 Natao, pois 3/9 > 2/9. b) 5/18
c) Natao; Nado
Sincronizado
43. 35
44. 72 e 120
45. c
46. d
47. b
48. a) 3 milhes de reais para a Educao Infantil e 9 milhes de reais para o Ensino Fundamental. b) 1/24 e 3/20
da verba total, respectivamente, para a Educao Infantil e para o Ensino Fundamental. c) 1/12 da verba total. d) 1/9
dos recursos dirigidos ao ensino fundamental.
49. b
50. e
51. a
52. c
53. d
54. d
55. d
56. b
57. e
58. e
59. a) 25 b) 25 c) -8 d) 1 e) 243 f) 1 g) 9/4 h) 2 i) 64
60. b
61. d
62. 7
63. b
64. a) 9 b) 20 c) 6 d) 4

36.
37.
38.
39.
40.
41.
42.

4
3
65. a) 2 3 b) 8 3 c) 3 4 d) 2 3
66. a
67. 3 6

68. 9 5
3 2
5
2
69.
b) 2 2 c) 2
d) 2 5 + 4
70.
d
71.
a
72.
c
73.
e
74.
b
75.
a) 124/100
b) 752/100
c) 3/1000
d) 108/10
e) 1887/1000
76.
a) Q b) I c) I d) Q e) I f) Q g) I h) Q i) Q
77.
a) 5/9 b) 37/99 c) 4/3
78.
a) 13/3 b) 1/10 c) 25/11 d) 5
79.
b
80.
a) { 1, 2, 4, 5, 6, 7 } b) { 1, 5, 6 } c) { 2 } d) { } ou e) { 1, 5, 6 }
81.
V,F,V,F
82.
a

32

__________________________________________________________________________________________________________________________

ATUALIZADO AT MAIO/2012

www.CARREIRAPUBLICA.com.br

(48) 4141-3220

4141-3222

____________

_____________________________ MATEMTICA _________________________________ Prof. Roberto

Desafio
Desafio
Desafio

83.
84.
85.

7 Produtos Notveis
Vamos relembrar aqui, identidades especiais, conhecidas particularmente como Produtos Notveis.
7.1 Quadrado da soma e da diferena
2

(a + b) = a + 2ab + b
2
2
2
(a b) = a 2ab + b
Das duas anteriores, poderemos concluir que tambm vlido que:
2
2
2
2
(a+b) + (a-b) = 2(a +b ) ou escrevendo de uma forma conveniente:

7.2 Diferena de quadrados


2

(a + b).(a b) = a b

7.3 Cubo de uma soma e de uma diferena


3

(a + b) = a + 3.a .b + 3.a.b + b
Para determinar o cubo da diferena, basta substituir na identidade acima, b por -b, obtendo:
3
3
2
2
3
(a b) = a 3.a .b + 3.a.b b
Uma forma mais conveniente de apresentar o cubo de soma, pode ser obtida fatorando-se a expresso como segue:
3
3
3
(a + b) = a + 3.a.b(a+b) + b
Ou:
3
3
3
(a + b) = a + b + 3ab(a + b)
Esta forma de apresentao, bastante til.
Exemplos:
1 A soma de dois nmeros igual a 10 e a soma dos seus cubos igual a 100. Qual o valor do produto desses
nmeros?
Soluo:
3
3
Temos: a + b = 10 e a + b = 100. Substituindo diretamente na frmula anterior, fica:
3
10 = 100 + 3ab(10) de onde tiramos 1000 = 100 + 30.ab
Da, vem: 900 = 30.ab, de onde conclumos finalmente que ab = 30, que a resposta solicitada.
Nota: os nmeros a e b que satisfazem condio do problema acima, no so nmeros reais e sim, nmeros
complexos. Voc pode verificar isto, resolvendo o sistema formado pelas igualdades a+b = 10 e ab = 30. Verifique
como exerccio!
Alerto para o fato de que muito trabalhoso. Mas, v l, faa! um bom treinamento sobre as operaes com
nmeros complexos. Pelo menos, fica caracterizada a importncia de saber a frmula acima. Sem ela, a soluo
DESTE PROBLEMA SIMPLES, seria bastante penosa!

__________________________________________________________________________________________________________________________

ATUALIZADO AT MAIO/2012

www.CARREIRAPUBLICA.com.br

(48) 4141-3220

4141-3222

33

____________

_____________________________ MATEMTICA _________________________________ Prof. Roberto

2 - Calcule o valor de F na expresso abaixo, para:


a = -700, b = - 33 , x = 23,48 e y = 9,14345.

Soluo: Com a substituio direta dos valores dados, os clculos seriam tantos que seria invivel! Vamos
desenvolver os produtos notveis indicados:

Se voc observar cuidadosamente a expresso acima, ver que o numerador e o denominador da frao so
IGUAIS, e, portanto, F = 1, independente dos valores de a, b, x e y.
Portanto, a resposta igual a 1, independente dos valores atribudos s variveis a, b, x e y.
Resp: 1
8 Decomposio em fatores primos
Todo nmero natural, maior que 1, pode ser decomposto num produto de dois ou mais fatores.
Decomposio do nmero 24 num produto:
24 = 4 x 6
24 = 2 x 2 x 6
3
24 = 2 x 2 x 2 x 3 = 2 x 3
No produto 2 x 2 x 2 x 3 todos os fatores so primos.
Chamamos de fatorao de 24 a decomposio de 24 num produto de fatores primos. Ento a fatorao de 24
3
2 x 3.

De um modo geral, chamamos de fatorao de um nmero natural, maior que 1, a sua


decomposio num produto de fatores primos.

8.1

Regra prtica para a fatorao

Existe um dispositivo prtico para fatorar um nmero. Acompanhe, no exemplo, os passos para montar esse
dispositivo:

1) Dividimos o nmero pelo seu menor divisor primo;


2) a seguir, dividimos o quociente obtido pelo menor
divisor primo desse quociente e assim sucessivamente
at obter o quociente 1.
A figura ao lado mostra a fatorao do nmero 630.

Ento 630 = 2 x 3 x 3 x 5 x 7.
2
630 = 2 x 3 x 5 x 7.

8.2 Nmeros Primos

34

__________________________________________________________________________________________________________________________

ATUALIZADO AT MAIO/2012

www.CARREIRAPUBLICA.com.br

(48) 4141-3220

4141-3222

____________

_____________________________ MATEMTICA _________________________________ Prof. Roberto

Nmeros primos so os nmeros naturais que tm apenas dois divisores diferentes: o 1 e ele mesmo.
Exemplos:
1) 2 tem apenas os divisores 1 e 2, portanto 2 um nmero primo.
2) 17 tem apenas os divisores 1 e 17, portanto 17 um nmero primo.
3) 10 tem os divisores 1, 2, 5 e 10, portanto 10 no um nmero primo.
Observaes:
=> 1 no um nmero primo, porque ele tem apenas um divisor que ele mesmo.
=> 2 o nico nmero primo que par.
Os nmeros que tm mais de dois divisores so chamados nmeros compostos.
Exemplo: 15 tem mais de dois divisores => 15 um nmero composto.
8.3 Reconhecimento de um nmero primo
Para saber se um nmero primo, dividimos esse nmero pelos nmeros primos 2, 3, 5, 7, 11 etc. at que
tenhamos:
=> ou uma diviso com resto zero e neste caso o nmero no primo,
=> ou uma diviso com quociente menor que o divisor e o resto diferente de zero. Neste caso o nmero
primo.
Exemplos:
1) O nmero 161:

no par, portanto no divisvel por 2;


1+6+1 = 8, portanto no divisvel por 3;
no termina em 0 nem em 5, portanto no divisvel por 5;
por 7: 161 / 7 = 23, com resto zero, logo 161 divisvel por 7, e portanto no um nmero primo.

2) O nmero 113:

no par, portanto no divisvel por 2;


1+1+3 = 5, portanto no divisvel por 3;
no termina em 0 nem em 5, portanto no divisvel por 5;
por 7: 113 / 7 = 16, com resto 1. O quociente (16) ainda maior que o divisor (7).
por 11: 113 / 11 = 10, com resto 3. O quociente (10) menor que o divisor (11), e alm disso o resto
diferente de zero (o resto vale 3), portanto 113 um nmero primo.

8.4 Determinao dos divisores de um nmero


Na prtica determinamos todos os divisores de um nmero utilizando os seus fatores primos.
Vamos determinar, por exemplo, os divisores de 90:

1) decompomos o nmero em fatores primos;


2) traamos uma linha e escrevemos o 1 no alto,
porque ele divisor de qualquer nmero;

__________________________________________________________________________________________________________________________

ATUALIZADO AT MAIO/2012

www.CARREIRAPUBLICA.com.br

(48) 4141-3220

4141-3222

35

____________

_____________________________ MATEMTICA _________________________________ Prof. Roberto

3) multiplicamos sucessivamente cada fator primo pelos


divisores j obtidos e escrevemos esses produtos ao lado de
cada fator primo;

4) os divisores j obtidos no precisam ser repetidos.

Portanto os divisores de 90 so 1, 2, 3, 5, 6, 9, 10, 15, 18, 30, 45, 90.

9 Sistemas de Medidas

9.1 Medidas de comprimento

9.1.1 Sistema Mtrico Decimal


Desde a Antiguidade os povos foram criando suas unidades de medida. Cada um deles possua suas prprias
unidades-padro. Com o desenvolvimento do comrcio ficavam cada vez mais difceis a troca de informaes e as
negociaes com tantas medidas diferentes. Era necessrio que se adotasse um padro de medida nico para
cada grandeza.
Foi assim que, em 1791, poca da Revoluo francesa, um grupo de representantes de vrios pases reuniu-se
para discutir a adoo de um sistema nico de medidas. Surgia o sistema mtrico decimal.
9.1.2 Metro
A palavra metro vem do gegro mtron e significa "o que mede". Foi estabelecido inicialmente que a medida do
metro seria a dcima milionsima parte da distncia do Plo Norte ao Equador, no meridiano que passa por Paris. No
Brasil o metro foi adotado oficialmente em 1928.
9.1.3

Mltiplos e Submltiplos do Metro

Alm da unidade fundamental de comprimento, o metro, existem ainda os seus mltiplos e submltiplos, cujos
nomes so formados com o uso dos prefixos: quilo, hecto, deca, deci, centi e mili. Observe o quadro:

Mltiplos
quilmetro

36

hectmetro

decmetro

Unidade
Fundamental
metro

Submltiplos
decmetro

centmetro

milmetro

__________________________________________________________________________________________________________________________

ATUALIZADO AT MAIO/2012

www.CARREIRAPUBLICA.com.br

(48) 4141-3220

4141-3222

____________

_____________________________ MATEMTICA _________________________________ Prof. Roberto

km
1.000m

hm
100m

dam
10m

m
1m

dm
0,1m

cm
0,01m

mm
0,001m

Os mltiplos do metro so utilizados para medir grandes distncias, enquanto os submltiplos, para pequenas
distncias. Para medidas milimtricas, em que se exige preciso, utilizamos:
-6

-10

mcron () = 10 m

angstrn () = 10

Para distncias astronmicas utilizamos o Ano-luz (distncia percorrida pela luz em um ano):
12

Ano-luz = 9,5 10 km
O p, a polegada, a milha e a jarda so unidades no pertencentes ao sistemas mtrico decimal, so utilizadas em
pases de lngua inglesa. Observe as igualdades abaixo:
P
Polegada
Jarda
Milha terrestre
Milha martima

=
=
=
=
=

30,48 cm
2,54 cm
91,44 cm
1.609 m
1.852 m

Observe que:
1 p = 12 polegadas 1 jarda = 3 ps
9.1.4 Leitura das Medidas de Comprimento
A leitura das medidas de comprimentos pode ser efetuada com o auxlio do quadro de unidades. Exemplos: Leia a
seguinte medida: 15,048 m.

Seqncia prtica
1)

Escrever o quadro de unidades:


km

hm

dam

dm

cm

mm

2) Colocar o nmero no quadro de unidades, localizando o ltimo algarismo da parte inteira sob a sua
respectiva.
km

hm

dam
1

m
5,

dm
0

cm
4

mm
8

3) Ler a parte inteira acompanhada da unidade de medida do seu ltimo algarismo e a parte decimal
acompanhada da unidade de medida do ltimo algarismo da mesma.
15 metros e 48 milmetros
Outros exemplos:
6,07 km

l-se "seis quilmetros e sete decmetros"

__________________________________________________________________________________________________________________________

ATUALIZADO AT MAIO/2012

www.CARREIRAPUBLICA.com.br

(48) 4141-3220

4141-3222

37

____________

82,107 dam
0,003 mm

_____________________________ MATEMTICA _________________________________ Prof. Roberto

l-se "oitenta e dois decmetros e cento e sete centmetros".


l-se "trs milmetros".

9.1.5 Transformao de Unidades

Observe as seguintes transformaes:

Transforme 16,584hm em m.
km

hm

dam

dm

cm

mm

Para transformar hm em m (duas posies direita) devemos multiplicar por 100 (10 x 10).
16,584 x 100 = 1.658,4
Ou seja:
16,584hm = 1.658,4m
Transforme 1,463 dam em cm.
km

hm

dam

dm

cm

mm

Para transformar dam em cm (trs posies direita) devemos multiplicar por 1.000 (10 x 10 x 10).
1,463 x 1.000 = 1,463
Ou seja:
1,463dam = 1.463cm.

Transforme 176,9m em dam.


km

hm

dam

dm

cm

mm

Para transformar dam em cm (trs posies esquerda) devemos dividir por 10.
176,9 : 10 = 17,69
Ou seja:
176,9m = 17,69dam

38

__________________________________________________________________________________________________________________________

ATUALIZADO AT MAIO/2012

www.CARREIRAPUBLICA.com.br

(48) 4141-3220

4141-3222

____________

_____________________________ MATEMTICA _________________________________ Prof. Roberto

Transforme 978m em km.


km

hm

dam

dm

cm

mm

Para transformar m em km (trs posies esquerda) devemos dividir por 1.000.


978 : 1.000 = 0,978
Ou seja:
978m = 0,978km.
Observao:
Para resolver uma expresso formada por termos com diferentes unidades, devemos inicialmente transformar
todos eles numa mesma unidade, para a seguir efetuar as operaes.
9.2 Medidas de superfcie

9.2.1 Introduo
As medidas de superfcie fazem parte de nosso dia a dia e respondem a nossas perguntas mais corriqueiras do
cotidiano:

Qual a rea desta sala?


Qual a rea desse apartamento?
Quantos metros quadrados de azulejos so necessrios para revestir essa piscina?
Qual a rea dessa quadra de futebol de salo?
Qual a rea pintada dessa parede?

9.2.2

Superfcie e rea

Superfcie uma grandeza com duas dimenses, enquanto rea a medida dessa grandeza, portanto, um nmero.
9.2.3 Metro Quadrado
2

A unidade fundamental de superfcie chama-se metro quadrado. O metro quadrado (m ) a medida


correspondente superfcie de um quadrado com 1 metro de lado.
Unidade
Fundamental

Mltiplos
quilmetros
quadrado
2
km
2
1.000.000m
2

hectmetro
quadrado
2
hm
2
10.000m

decmetro
quadrado
2
dam
2
100m

metro quadrado
2

m
2
1m

Submltiplos
decmetro
quadrado
2
dm
2
0,01m

centmetro
quadrado
2
cm
2
0,0001m
2

milmetro
quadrado
2
mm
2
0,000001m

O dam , o hm e km so utilizados para medir grandes superfcies, enquanto o dm , o cm e o mm so utilizados


para pequenas superfcies.
Exemplos:
1) Leia a seguinte medida: 12,56m
km

hm

dam

m
12,

dm
56

cm

mm

__________________________________________________________________________________________________________________________

ATUALIZADO AT MAIO/2012

www.CARREIRAPUBLICA.com.br

(48) 4141-3220

4141-3222

39

____________

_____________________________ MATEMTICA _________________________________ Prof. Roberto

L-se 12 metros quadrados e 56 decmetros quadrados. Cada coluna dessa tabela corresponde a uma unidade
de rea.

2) Leia a seguinte medida: 178,3 m


km

hm

dam
1

m
78,

dm
30

cm

mm

dm
70

cm

mm

L-se 178 metros quadrados e 30 decmetros quadrados


3) Leia a seguinte medida: 0,917 dam
km

hm

dam
0,

m
91

L-se 9.170 decmetros quadrados.


9.3 Medidas Agrrias

As medidas agrrias so utilizadas para medir superfcies de campo, plantaes, pastos, fazendas, etc. A principal
unidade destas medidas o are (a). Possui um mltiplo, o hectare (ha), e um submltiplo, o centiare (ca).
Unidade
agrria
Equivalncia
de valor

hectare (ha)

are (a)

centiare (ca)

100a

1a

0,01a

Lembre-se:
2

1 ha = 1hm
2
1a = 1 dam
2
1ca = 1m
9.3.1 Transformao de unidades

No sistema mtrico decimal, devemos lembrar que, na transformao de unidades de superfcie, cada unidade de
superfcie 100 vezes maior que a unidade imediatamente inferior:

Observe as seguintes transformaes:

Transformar 2,36 m em mm .
km

40

hm

dam

dm

cm

mm

__________________________________________________________________________________________________________________________

ATUALIZADO AT MAIO/2012

www.CARREIRAPUBLICA.com.br

(48) 4141-3220

4141-3222

____________

_____________________________ MATEMTICA _________________________________ Prof. Roberto


2

Para transformar m em mm (trs posies direita) devemos multiplicar por 1.000.000 (100x100x100).
2,36 x 1.000.000 = 2.360.000 mm
2

Transformar 580,2 dam em km .


km

hm

dam

dm

cm

mm

Para transformar dam em km (duas posies esquerda) devemos dividir por 10.000 (100x100).
580,2 : 10.000 = 0,05802 km

Pratique! Tente resolver esses exerccios:


2

1) Transforme 8,37 dm em mm
(R: 83.700 mm )
2
2
2
2) Transforme 3,1416 m em cm
(R: 31.416 cm )
2
2
2
3) Transforme 2,14 m em dam
(R: 0,0214 dam )
2
4) Calcule 40m x 25m (R: 1.000 m )

9.4 Medidas de volume

9.4.1 Introduo
Frequentemente nos deparamos com problemas que envolvem o uso de trs dimenses: comprimento, largura e
altura. De posse de tais medidas tridimensionais, poderemos calcular medidas de metros cbicos e volume.

9.4.2 Metro cbico


3

A unidade fundamental de volume chama-se metro cbico. O metro cbico (m ) medida correspondente ao
espao ocupado por um cubo com 1 m de aresta.
9.4.3 Mltiplos e submltiplos do metro cbico

Unidade
Fundamental

Mltiplos
quilmetro
cbico
3
km
3
1.000.000.000m

hectmetro
cbico
3
hm
3
1.000.000 m

decmetro
decmetro
metro cbico
cbico
cbico
3
3
3
dam
m
dm
3
3
3
1.000m
1m
0,001m

Submltiplos
centmetro
cbico
3
cm
3
0,000001m

milmetro cbico
3

mm
3
0,000000001 m

9.4.4 Leitura das medidas de volume


A leitura das medidas de volume segue o mesmo procedimento do aplicado s medidas lineares. Devemos utilizar
porem, trs algarismo em cada unidade no quadro. No caso de alguma casa ficar incompleta, completa-se com
zero(s). Exemplos.
Leia a seguinte medida: 75,84m

__________________________________________________________________________________________________________________________

ATUALIZADO AT MAIO/2012

www.CARREIRAPUBLICA.com.br

(48) 4141-3220

4141-3222

41

____________

km

_____________________________ MATEMTICA _________________________________ Prof. Roberto

hm

dam

dm
840

dm
006

m
75,

cm

mm

cm
400

mm

L-se "75 metros cbicos e 840 decmetros cbicos".


Leia a medida: 0,0064dm
km

hm

dam

m
0,

L-se "6400 centmetros cbicos


9.4.5 Transformao de unidades
Na transformao de unidades de volume, no sistema mtrico decimal, devemos lembrar que cada unidade de
volume 1.000 vezes maior que a unidade imediatamente inferior.

Observe a seguinte transformao:


3

Transformar 2,45 m para dm .


km

hm

dam

dm

cm

mm

Para transformar m em dm (uma posio direita) devemos multiplicar por 1.000.


2,45 x 1.000 = 2.450 dm

Pratique! Tente resolver esses exerccios:


3

1) Transforme 8,132 km em hm
(R: 8.132 hm )
3
3
3
2) Transforme 180 hm em km
(R: 0,18 km )
3
3
3
3) Transforme 1 dm em dam
(R: 0,000001 dam )
3
3
3
4) Expresse em metros cbicos o valor da expresso: 3.540dm + 340.000cm (R: 3,88 m )
9.5 Medidas de capacidade
A quantidade de lquido igual ao volume interno de um recipiente, afinal quando enchemos este recipiente, o
lquido assume a forma do mesmo. Capacidade o volume interno de um recipiente.
A unidade fundamental de capacidade chama-se litro.
9.5.1 Litro
a capacidade de um cubo que tem 1dm de aresta.
1l = 1dm

9.5.2 Mltiplos e submltiplos do litro


42 __________________________________________________________________________________________________________________________
ATUALIZADO AT MAIO/2012

www.CARREIRAPUBLICA.com.br

(48) 4141-3220

4141-3222

____________

_____________________________ MATEMTICA _________________________________ Prof. Roberto

Unidade
Fundamental
decalitro
litro
decilitro
dal
l
dl
10l
1l
0,1l

Mltiplos
quilolitro
kl
1000l

hectolitro
hl
100l

Submltiplos
centilitro
cl
0,01l

mililitro
ml
0,001l

Cada unidade 10 vezes maior que a unidade imediatamente inferior.


Relaes
1l = 1dm

1ml = 1cm

1kl = 1m

9.5.3 Leitura das medidas de capacidade


Exemplo: leia a seguinte medida: 2,478 dal
kl

hl

dal
2,

l
4

dl
7

cl
8

ml

L-se "2 decalitros e 478 centilitros".

9.5.4 Transformao de unidades


Na transformao de unidades de capacidade, no sistema mtrico decimal, devemos lembrar que cada unidade de
capacidade 10 vezes maior que a unidade imediatamente inferior.

Observe a seguinte transformao:


Transformar 3,19 l para ml.
kl

hl

dal

dl

cl

ml

Para transformar l para ml (trs posies direita) devemos multiplicar por 1.000 (10x10x10).
3,19 x 1.000 = 3.190 ml
Pratique! Tente resolver esses exerccios:
1) Transforme 7,15 kl em dl (R: 71.500 dl)
2) Transforme 6,5 hl em l (R: 650 l)
3) Transforme 90,6 ml em l (R: 0,0906 l)
3
4) Expresse em litros o valor da expresso: 0,6m + 10 dal + 1hl (R: 800 l)
9.6 Medidas de massa
__________________________________________________________________________________________________________________________

ATUALIZADO AT MAIO/2012

www.CARREIRAPUBLICA.com.br

(48) 4141-3220

4141-3222

43

____________

_____________________________ MATEMTICA _________________________________ Prof. Roberto

9.6.1 Introduo
Observe a distino entre os conceitos de corpo e massa:
9.6.2 Massa
a quantidade de matria que um corpo possui, sendo, portanto, constante em qualquer lugar da terra ou fora dela.
9.6.3 Peso
a fora com que esse corpo atrado (gravidade) para o centro da terra. Varia de acordo com o local em que o
corpo se encontra. Por exemplo:
A massa do homem na Terra ou na Lua tem o mesmo valor. O peso, no entanto, seis vezes maior na terra do
que na lua.
Explica-se esse fenmeno pelo fato da gravidade terrestre ser 6 vezes superior gravidade lunar.
Obs: A palavra grama, empregada no sentido de "unidade de medida de massa de um corpo", um substantivo
masculino. Assim 200g, l-se "duzentos gramas".
9.6.4 Quilograma
A unidade fundamental de massa chama-se quilograma.
3

O quilograma (Kg) a massa de 1dm de gua destilada temperatura


de 4C.
Apesar de o quilograma ser a unidade fundamental de massa, utilizamos na prtica o grama como unidade
principal de massa.
9.6.5 Mltiplos e Submltiplos do grama

Mltiplos
quilograma
kg
1.000g

hectograma
hg
100g

decagrama
dag
10g

Unidade
principal
grama
g
1g

Submltiplos
decigrama
dg
0,1g

centigrama
cg
0,01g

miligrama
mg
0,001g

Observe que cada unidade de volume dez vezes maior que a unidade imediatamente inferior. Exemplos:
1 dag = 10 g
1 g = 10 dg
9.6.6 Medidas de massa
Relaes Importantes
Podemos relacionar as medidas de massa com as medidas de volume e capacidade.
Assim, para a gua pura (destilada) a uma temperatura de 4C vlida a seguinte equivalncia:
3

1 kg <=> 1dm <=> 1l

44

__________________________________________________________________________________________________________________________

ATUALIZADO AT MAIO/2012

www.CARREIRAPUBLICA.com.br

(48) 4141-3220

4141-3222

____________

_____________________________ MATEMTICA _________________________________ Prof. Roberto

So vlidas tambm as relaes:


3

1m <=> 1 Kl <=> 1t

1cm <=> 1ml <=> 1g

Observao:
Na medida de grandes massas, podemos utilizar ainda as seguintes unidades especiais:
1 arroba = 15 kg
1 tonelada (t) = 1.000 kg
1 megaton = 1.000 t ou 1.000.000 kg
9.6.7 Leitura das Medidas de Massa
A leitura das medidas de massa segue o mesmo procedimento aplicado s medidas lineares. Exemplos:
Leia a seguinte medida: 83,732 hg
kg
8

hg
3,

dag
7

g
3

dg
1

cg

mg

g
0,

dg
0

cg
4

mg
3

L-se "83 hectogramas e 731 decigramas".


Leia a medida: 0,043g
kg

hg

dag

L-se " 43 miligramas".


9.6.8 Transformao de Unidades

Cada unidade de massa 10 vezes maior que a unidade imediatamente


inferior.

Observe as Seguintes transformaes:


Transforme 4,627 kg em dag.
kg

hg

dag

dg

cg

mg

__________________________________________________________________________________________________________________________

ATUALIZADO AT MAIO/2012

www.CARREIRAPUBLICA.com.br

(48) 4141-3220

4141-3222

45

____________

_____________________________ MATEMTICA _________________________________ Prof. Roberto

Para transformar kg em dag (duas posies direita) devemos multiplicar por 100

(10 x 10).

4,627 x 100 = 462,7


Ou seja:

4,627 kg = 462,7 dag

Observao:
Peso bruto: peso do produto com a embalagem.
Peso lquido: peso somente do produto.
9.8 Medidas de tempo

9.8.1 Introduo
comum em nosso dia-a-dia pergunta do tipo:
Qual a durao dessa partida de futebol?
Qual o tempo dessa viagem?
Qual a durao desse curso?
Qual o melhor tempo obtido por esse corredor?
Todas essas perguntas sero respondidas tomando por base uma unidade padro de medida de tempo.
A unidade de tempo escolhida como padro no Sistema Internacional (SI) o segundo.
9.8.2 Segundo
O Sol foi o primeiro relgio do homem: o intervalo de tempo natural decorrido entre as sucessivas passagens do Sol
sobre um dado meridiano d origem ao dia solar.

O segundo (s) o tempo equivalente a

do dia solar mdio.

As medidas de tempo no pertencem ao Sistema Mtrico Decimal.

9.8.3 Mltiplos e Submltiplos do Segundo


Quadro de unidades

minutos
min
60 s

Mltiplos
hora
h
60 min = 3.600 s

dia
d
24 h = 1.440 min = 86.400s

So submltiplos do segundo:

46

dcimo de segundo
centsimo de segundo

__________________________________________________________________________________________________________________________

ATUALIZADO AT MAIO/2012

www.CARREIRAPUBLICA.com.br

(48) 4141-3220

4141-3222

____________

_____________________________ MATEMTICA _________________________________ Prof. Roberto

milsimo de segundo

Cuidado: Nunca escreva 2,40h como forma de representar 2 h 40 min. Pois o sistema de medidas de tempo no
decimal.
Observe:

10 Equaes do primeiro grau

10.1 Introduo

Equao toda sentena matemtica aberta que exprime uma relao de igualdade. A palavra equao tem o prefixo
equa, que em latim quer dizer "igual". Exemplos:
2x + 8 = 0
5x - 4 = 6x + 8
3a - b - c = 0
No so equaes:
4 + 8 = 7 + 5 (No uma sentena aberta)
x - 5 < 3 (No igualdade)
(no sentena aberta, nem igualdade)
A equao geral do primeiro grau:
ax+b = 0
onde a e b so nmeros conhecidos e a > 0, se resolve de maneira simples: subtraindo b dos dois lados, obtemos:
ax = -b
dividindo agora por a (dos dois lados), temos:

__________________________________________________________________________________________________________________________

ATUALIZADO AT MAIO/2012

www.CARREIRAPUBLICA.com.br

(48) 4141-3220

4141-3222

47

____________

_____________________________ MATEMTICA _________________________________ Prof. Roberto

Considere a equao 2x - 8 = 3x -10

A letra a incgnita da equao. A palavra incgnita significa " desconhecida".


Na equao acima a incgnita x; tudo que antecede o sinal da igualdade denomina-se 1 membro, e o que
sucede, 2 membro.

Qualquer parcela, do 1 ou do 2 membro, um termo da equao.

Equao do 1 grau na incgnita x toda equao que pode ser escrita na forma ax=b, sendo a e b
nmeros racionais, com a diferente de zero.

10.2 Conjunto Verdade e Conjunto Universo de uma Equao


Considere o conjunto A = {0, 1, 2, 3, 4, 5} e a equao x + 2 = 5.
Observe que o nmero 3 do conjunto A denominado conjunto universo da equao e o conjunto {3} o
conjunto verdade dessa mesma equao.
Da, conclumos que:
Conjunto Universo o conjunto de todos os valores que varivel pode assumir. Indica-se
por U.

Conjunto verdade o conjunto dos valores de U, que tornam verdadeira a equao . Indica-

48

__________________________________________________________________________________________________________________________

ATUALIZADO AT MAIO/2012

www.CARREIRAPUBLICA.com.br

(48) 4141-3220

4141-3222

____________

_____________________________ MATEMTICA _________________________________ Prof. Roberto

se por V.

Observaes:
O conjunto verdade subconjunto do conjunto universo.

No sendo citado o conjunto universo, devemos considerar como conjunto universo o conjunto dos nmeros
racionais.

O conjunto verdade tambm conhecido por conjunto soluo e pode ser indicado por S.

10.3 Razes de uma equao


Os elementos do conjunto verdade de uma equao so chamados razes da equao.
Para verificar se um nmero raiz de uma equao, devemos obedecer seguinte seqncia:

Substituir a incgnita por esse nmero.


Determinar o valor de cada membro da equao.
Verificar a igualdade, sendo uma sentena verdadeira, o nmero considerado raiz da equao.
Exemplos:

Verifique quais dos elementos do conjunto universo so razes das equaes abaixo, determinando em
cada caso o conjunto verdade.

Resolva a equao x - 2 = 0, sendo U = {0, 1, 2, 3}.


Para x = 0 na equao x - 2 = 0 temos: 0 - 2 = 0 => -2 = 0. (F)
Para x = 1 na equao x - 2 = 0 temos: 1 - 2 = 0 => -1 = 0. (F)
Para x = 2 na equao x - 2 = 0 temos: 2 - 2 = 0 => 0 = 0. (V)
Para x = 3 na equao x - 2 = 0 temos: 3 - 2 = 0 => 1 = 0. (F)

Verificamos que 2 raiz da equao x - 2 = 0, logo V = {2}.

Resolva a equao 2x - 5 = 1, sendo U = {-1, 0, 1, 2}.

__________________________________________________________________________________________________________________________

ATUALIZADO AT MAIO/2012

www.CARREIRAPUBLICA.com.br

(48) 4141-3220

4141-3222

49

____________

_____________________________ MATEMTICA _________________________________ Prof. Roberto

Para x = -1 na equao 2x - 5 = 1 temos: 2 . (-1) - 5 = 1 => -7 = 1. (F)


Para x = 0 na equao 2x - 5 = 1 temos: 2 . 0 - 5 = 1 => -5 = 1. (F)
Para x = 1 na equao 2x - 5 = 1 temos: 2 . 1 - 5 = 1 => -3 = 1. (F)
Para x = 2 na equao 2x - 5 = 1 temos: 2 . 2 - 5 = 1 => -1 = 1. (F)

A equao 2x - 5 = 1 no possui raiz em U, logo V = .


10.4 Resoluo de uma equao

Resolver uma equao consiste em realizar uma espcie de operaes de operaes que nos conduzem a equaes
equivalentes cada vez mais simples e que nos permitem, finalmente, determinar os elementos do conjunto verdade
ou as razes da equao. Resumindo:
Resolver uma equao significa determinar o seu conjunto verdade, dentro do
conjunto universo considerado.

Na resoluo de uma equao do 1 grau com uma incgnita, devemos aplicar os princpios de equivalncia das
igualdades (aditivo e multiplicativo). Exemplos:
Sendo

, resolva a equao
.

m.m.c. (4, 6) = 12

-9x = 10

=> Multiplicador por (-1)

9x = -10

Como

Sendo

, ento

, resolva a equao
2 . (x - 2) - 3 . (1 - x) = 2 . (x - 4).

Iniciamos aplicando a propriedade distributiva da multiplicao:

2x - 4 - 3 + 3x = 2x - 8

50

__________________________________________________________________________________________________________________________

ATUALIZADO AT MAIO/2012

www.CARREIRAPUBLICA.com.br

(48) 4141-3220

4141-3222

____________

_____________________________ MATEMTICA _________________________________ Prof. Roberto

2x + 3x -2
x=-8+4+3
3x = -1

Como

, ento

10.5 Equaes impossveis e identidades

Sendo

, considere a seguinte equao: 2 . (6x - 4) = 3 . (4x - 1).

Observe, agora, a sua resoluo:


2 . 6x - 2 . 4 = 3 . 4x - 3 . 1

12x - 8 = 12x - 3

12x - 12x = - 3 + 8

0.x=5
Como nenhum nmero multiplicado por zero igual a 5, dizemos que a equao impossvel e, portanto, no tem
soluo. Logo, V = .
Assim, uma equao do tipo ax + b = 0 impossvel quando
Sendo

, considere a seguinte equao: 10 - 3x - 8 = 2 - 3x.


Observe a sua resoluo:
-3x + 3x = 2 - 10 + 8
0.x=0

Como todo nmero multiplicado por zero igual a zero, dizemos que a equao possui infinitas solues.
Equaes desse tipo, em que qualquer valor atribudo varivel torna a equao verdadeira, so denominadas
identidades.
10.7 Pares ordenados
Muitas vezes, para localizar um ponto num plano, utilizamos dois nmeros racionais, numa certa ordem.
Denominamos esses nmeros de par ordenado. Exemplos:

__________________________________________________________________________________________________________________________

ATUALIZADO AT MAIO/2012

www.CARREIRAPUBLICA.com.br

(48) 4141-3220

4141-3222

51

____________

_____________________________ MATEMTICA _________________________________ Prof. Roberto

Assim:
Indicamos por (x, y) o par ordenado formado pelos elementos x e y, onde x o 1
elemento e y o 2 elemento.

Observaes
1. De um modo geral, sendo x e y dois nmeros racionais quaisquer, temos:

2. Dois pares ordenados (x, y) e (r, s) so iguais somente se

x=r e

. Exemplos

y = s.

10.8 Representao grfica de um Par Ordenado


Podemos representar um par ordenado atravs de um ponto num plano.
Esse ponto chamado de imagem do par ordenado.
10.9 Coordenadas Cartesianas

Os nmeros do par ordenados so chamados de coordenadas cartesianas. Exemplos:

A (3, 5) ==> 3 e 5 so as coordenadas do ponto A.


Denominamos de abscissa o 1 nmero do par ordenado, e ordenada, o 2 nmero desse par. Assim:

10.10 Plano Cartesiano

52

__________________________________________________________________________________________________________________________

ATUALIZADO AT MAIO/2012

www.CARREIRAPUBLICA.com.br

(48) 4141-3220

4141-3222

____________

_____________________________ MATEMTICA _________________________________ Prof. Roberto

Representamos um par ordenado num plano cartesiano.


Esse plano formado por duas retas, x e y perpendiculares entre si.
A reta horizontal o eixo das abscissas (eixo x).
A reta vertical o eixo das ordenadas (eixo y).
O ponto comum dessas duas retas denominado origem, que
corresponde ao par ordenado (0, 0).

10.11 Localizao de um Ponto

Para localizar um ponto num plano cartesiano, utilizamos a seqncia prtica:


O 1 nmero do par ordenado deve ser localizado no eixo das abscissas.
O 2 nmero do par ordenado deve ser localizado no eixo das ordenadas.
No encontro das perpendiculares aos eixos x e y, por esses pontos, determinamos o ponto procurado.
10.12 Produto Cartesiano

Sejam os conjuntos A = {1, 2, 3} e B = {3, 4}.


Com auxlio do diagrama de flechas ao lado formaremos o
conjunto de todos os pares ordenados em que o 1
elemento pertena ao conjunto A e o 2 pertena ao
conjunto B.
Assim , obtemos o conjunto: {(1, 3), (1, 4), (2, 3), (2, 4), (3, 3), (3, 4)}
Esse conjunto denominado produto cartesiano de A por B, e indicado por:

Logo:
Dados dois conjuntos A e B, no-vazios, denominamos produtos cartesiano A x B o conjunto de todos os
pares ordenados (x, y) onde

10.13 Equaes de primeiro grau com duas variveis


Considere a equao: 2x - 6 = 5 - 3y
__________________________________________________________________________________________________________________________

ATUALIZADO AT MAIO/2012

www.CARREIRAPUBLICA.com.br

(48) 4141-3220

4141-3222

53

____________

_____________________________ MATEMTICA _________________________________ Prof. Roberto

Trata-se de uma equao com duas variveis, x e y, pode ser transformada numa equao equivalente mais
simples. Assim:
2x - 3y = 5 + 6
2x - 3y = 11 ==> Equao do 1 grau na forma ax + by = c .
Denominando equao de 1 grau com duas variveis, x e y, a toda equao que pode ser reproduzida
forma ax + by = c, sendo a e b nmeros diferentes de zero, simultaneamente.
Na equao ax + by = c, denominamos:
x + y - variveis ou incgnita

b - coeficiente de y

a - coeficiente de x

c - termo independente

Exemplos:
-3x - 7y = -48
x + y = 30
2x- 3y = 0
2x + 3y = 15
x-y=8
x - 4y = 10
10.13.1 Soluo de uma equao de 1 grau com duas variveis
Quais o valores de x e y que tornam a sentena x - 2y = 4 verdadeira?
Observe os pares abaixo:
x = 6, y = 1
x - 2y = 4
6-2.1=4
6-2=4
4 = 4 (V)
x = 8, y = 2
x - 2y = 4
8-2.2=4
8-4=4
4 = 4 (V)
x = -2, y = -3
x - 2y = 4
-2 - 2 . (-3) = 4

54

__________________________________________________________________________________________________________________________

ATUALIZADO AT MAIO/2012

www.CARREIRAPUBLICA.com.br

(48) 4141-3220

4141-3222

____________

_____________________________ MATEMTICA _________________________________ Prof. Roberto

-2 + 6 = 4
4 = 4 (V)

Verificamos que todos esses pares so solues da equao x - 2y = 4.


Assim, os pares (6, 1); (8, 2); (-2, -3) so algumas das solues dessa equao.
Uma equaes do 1 grau com duas variveis tem infinitas solues - infinitos (x, y) - , sendo, portanto, seu conjunto
.
universo
Podemos determinar essas solues, atribuindo-se valores quaisquer para uma das variveis, calculando a seguir o
valor da outra. Exemplo:
Determine uma soluo para a equao 3x - y = 8.
Atribumos para o x o valor 1, e calculamos o valor de y. Assim:

3x - y = 8
3 . (1) - y = 8
3-y=8
1

-y = 5 ==> Multiplicamos por -

y = -5

O par (1, -5) uma das solues dessa equao.


V = {(1, -5)}
Resumindo:
Um par ordenado (r, s) soluo de uma equao ax + by = c (a e b no-nulos
simultaneamente), se para x = r e y = s a sentena verdadeira.

10.14 Grfico de uma equao de 1 grau com duas variveis


Sabemos que uma equao do 1 grau com duas variveis possui infinitas solues.
Cada uma dessas solues pode ser representada por um par ordenado (x, y).
Dispondo de dois pares ordenados de uma equao, podemos represent-los graficamente num plano cartesiano,
determinando, atravs da reta que os une, o conjunto das soluo dessa equao. Exemplo:

Construir um grfico da equao x + y = 4.


Inicialmente, escolhemos dois pares ordenados que solucionam essa equao.
1 par: A (4, 0)
2 par: B (0, 4)

__________________________________________________________________________________________________________________________

ATUALIZADO AT MAIO/2012

www.CARREIRAPUBLICA.com.br

(48) 4141-3220

4141-3222

55

____________

_____________________________ MATEMTICA _________________________________ Prof. Roberto

A seguir, representamos esses pontos num plano cartesiano.

x
4
0

y
0
4

Finalmente, unimos os pontos A e B, determinando a reta r, que contm todos os pontos solues da equao.

A reta r chamada reta suporte do grfico da equao.

10.14 Sistemas de Equaes


Considere o seguinte problema:
Pipoca, em sua ltima partida, acertou x arremessos de 2 pontos e y arremessos de 3 pontos. Ele acertou 25
arremessos e marcou 55 pontos. Quantos arremessos de 3 pontos ele acertou?
Podemos traduzir essa situao atravs de duas equaes, a saber:

56

x + y = 25

(total de arremessos certo)

2x + 3y = 55

(total de pontos obtidos)

__________________________________________________________________________________________________________________________

ATUALIZADO AT MAIO/2012

www.CARREIRAPUBLICA.com.br

(48) 4141-3220

4141-3222

____________

_____________________________ MATEMTICA _________________________________ Prof. Roberto

Essas equaes contm um sistema de equaes.


Costuma-se indicar o sistema usando chave.

O par ordenado (20, 5), que torna ambas as sentenas verdadeiras, chamado soluo do sistema.
Um sistema de duas equaes com duas variveis possui uma nica soluo.

10.15 Resoluo de Sistemas

A resoluo de um sistema de duas equaes com duas variveis consiste em determinar um par ordenado que torne
verdadeiras, ao mesmo tempo, essas equaes.
Estudaremos a seguir alguns mtodos:

10.15.1 Mtodo de substituio

Soluo
Isolamos x na 1 equao.
x=4-y
Substitumos esse valor na 2 equao.
2 . (4 - y) -3y = 3
Resolvemos a equao formada.
8 - 2y -3y = 3

-2y -3y = 3

-5y = 5

Multiplicamos por -1

5y = -5

y=1

__________________________________________________________________________________________________________________________

ATUALIZADO AT MAIO/2012

www.CARREIRAPUBLICA.com.br

(48) 4141-3220

4141-3222

57

____________

_____________________________ MATEMTICA _________________________________ Prof. Roberto

Substitumos o valor encontrado de y, em qualquer das equaes, determinando x.


x +1= 4
x= 4-1
x=3
soluo do sistema o par ordenado (3, 1).
V = {(3, 1)}
10.15.2 Mtodo da adio
Sendo U =

, observe a soluo de cada um dos sistemas a seguir, pelo mtodo da adio.

Resolva o sistema abaixo:

Soluo
Adicionamos membros a membros as equaes:

2x = 16

x=8

Substitumos o valor encontrado de x, em qualquer das equaes, determinado y:


8 + y = 10

y = 10 - 8

y=2
A soluo do sistema o par ordenado (8, 2)
V = {(8, 2)}

10.17 INEQUAES DO 1 GRAU

10.17.1 Introduo

58

__________________________________________________________________________________________________________________________

ATUALIZADO AT MAIO/2012

www.CARREIRAPUBLICA.com.br

(48) 4141-3220

4141-3222

____________

_____________________________ MATEMTICA _________________________________ Prof. Roberto

Denominamos inequao toda sentena matemtica aberta por uma desigualdade.


As inequaes do 1 grau com uma varivel podem ser escritas numa das seguintes formas:
,

, como a e b reais

. Exemplos:

10.17.2 Repres. grfica de uma ineq. do 1 grau com duas variveis

Mtodo prtico

Substitumos a desigualdade por uma igualdade.


Traamos a reta no plano cartesiano.
Escolhemos um ponto auxiliar, de preferncia o ponto (0, 0) e verificamos se o mesmo satisfaz ou no a
desigualdade inicial.
Em caso positivo, a soluo da inequao corresponde ao semiplano ao qual pertence o ponto auxiliar.

Em caso negativo, a soluo da inequao corresponde ao semiplano oposto aquele ao qual pertence o ponto
auxiliar. Exemplos:

Representa graficamente a inequao

Tabela
x

(x, y)

(0, 4)

(2, 0)

Substituindo o ponto auxiliar (0, 0) na inequao


Verificamos:
__________________________________________________________________________________________________________________________

ATUALIZADO AT MAIO/2012

www.CARREIRAPUBLICA.com.br

(48) 4141-3220

4141-3222

59

____________

_____________________________ MATEMTICA _________________________________ Prof. Roberto

(Afirmativa positiva, o ponto auxiliar satisfaz a inequao)


A soluo da inequao corresponde ao semiplano ao qual pertence o ponto auxiliar (0, 0).

10.17.3 Inequaes de primeiro grau

Resoluo Grfica de um Sistema de Inequaes do 1 grau


Para resolver um sistema de inequaes do 1 grau graficamente, devemos:

traar num mesmo plano o grfico de cada inequao;


determinar a regio correspondente interseco dos dois semiplanos. Exemplos:

D a resoluo grfica do sistema:

Soluo
Traando as retas -x + y = 4 e 3x + 2y = 6.
Grfico
Tabela
x

(x, y)

(0, 4)

-4

(-4, 0)

Tabela
x

(x, y)

-1

(0, -1)

(1, 0)

10.18 Exerccios
1 - Calcule o valor de x nas equaes a seguir:
a)
b)
c)

60

3x + 4 = x + 16
5x - 3 + x = - 3 + 4x - 7 +15
2( 2 - 3x ) - 4( x + 1 ) = - (3 + 2x ) - 3
__________________________________________________________________________________________________________________________

ATUALIZADO AT MAIO/2012

www.CARREIRAPUBLICA.com.br

(48) 4141-3220

4141-3222

____________

d)
e)

f)
g)

_____________________________ MATEMTICA _________________________________ Prof. Roberto

7
=
3 .
x-2
- 6x + 12
( x - 60 ) + 3x + 2(3x + x ) = 0

x - 2
x-4

5
11

259 - 2x =
13

(Agente Administrativo - DASP)


x-7
11

(Exator da fazenda - SC)

2 - Calcule a raiz da equao abaixo


1 . x+1

2 .
x-1

1 .
x - 1

(ESA)

3 - Calcule x para que a equao abaixo seja solvel.


2 .
=
x2
1 . +
x-1
x-2
x - 3x + 2
(ESA)
4 - Um operrio, depois de receber o seu ordenado pagou na Cooperativa uma quantia igual a 1/4 do que recebeu, no
aougue uma quantia igual a 1/9 do resto e ainda ficou com UM$ 460,00. Calcule o salrio do operrio. (Exator Secretaria da Fazenda - SC)
5 - Depois de gastar 1/3 do seu salrio com alimentao e 1/4 com habitao, Joo fica com UM$ 259,00 para
outras despesas. Calcule o salrio de Joo. (Agente Administrativo - Secretaria da Fazenda)
R. UM$ 621,60
6 - Doze rapazes fizeram uma vaquinhapara comprar um barco. Como dois deles desistiram , cada um teve que
desembolsar mais UM$ 20,00. Qual o preo do barco? (ESA)
7 - Dizia um pastor: se eu tivesse mais duas ovelhas, poderia dar a meus trs filhos, respectivamente, 1/3, 1/4 e 1/6
daquele total e ficaria com as trs restantes, Quantas ovelhas o pastor possua? (ESA) R. 10
8 - Qual a condio para que a equao 5x + b = a tenha raiz nula? (ESA)

9 - A metade de um nmero aumentada de 6 igual ao triplo do nmero, diminudo de 4. Qual o


nmero?
10 - O nmero que somado aos 2/3 resulta 30
a) mpar; b) mltiplo de 9 c) divisor de 30 d) primo e) quadrado perfeito.
11- 3/5 de um nmero somados a 1/2 igual a 2/3 desse mesmo nmero. O nmero :
a) 0
b) 1 c) 20/33 d) 33/20 e) 15/2
12 - A soma de trs nmeros inteiros e consecutivos 60. Ento verdade que:
a) o quociente do maior pelo menor 2 b) o produto dos trs nmeros 8.000
c) no existem nmeros nesta condio d) falta informao para determinar os trs nmeros
e) o produto dos trs nmeros 7.980.
13 - A diferena entre o triplo da idade da Maria e doze igual soma entre a sua idade e dez. Qual a sua idade?
R.: 11
14 - O Vtor foi s compras e gastou metade do dinheiro que tinha num livro e a quinta parte do restante num
chocolate. Determine quanto dinheiro levava o Vtor se lhe sobraram R$ 880,00.
15 - Pretende-se trocar a quantia de 500$00 pelo mesmo nmero de moedas de 20$00 e de 100$00. Ser possvel?
Ser possvel com moedas de 25$00 em vez de 20$00?
16 - Dois capitais esto entre si na razo 8 para 3 e o maior deles excede o menor em UM$ 25.000,00. Calcule a
soma dos capitais. (Banco do Brasil)

__________________________________________________________________________________________________________________________

ATUALIZADO AT MAIO/2012

www.CARREIRAPUBLICA.com.br

(48) 4141-3220

4141-3222

61

____________

_____________________________ MATEMTICA _________________________________ Prof. Roberto

17 - Um atirador ganha UM$ 10,00 por tiro acertado e perde UM$ 15,00 por tiro errado. Se num total de 100 tiros
lucrou UM$ 250,00, quantos tiros errou?
(Banco do Brasil)
18 - Mauro cria galinhas e porcos em sua fazenda. So ao todo, 64 cabeas e 200 ps. Qual o nmero de galinhas?
(Banco do Brasil)
19 - Qual a frao equivalente a 2/3 cuja soma dos termos 40? (ESA)
20 - A diferena entre dois nmeros 15. Multiplicando-se o maior por 11, a diferena passa a ser 535, Qual o
nmero maior? (ESA)
21 - A soma de dois nmeros 180 e a sua diferena 120. Calcular o quociente entre o maior e o menor
nmero.
22 - A soma das idades de Eduardo e Cludia 26 anos. Daqui a 4 anos, Eduardo ter o dobro da idade que
Cludia tinha h 3 anos atrs. Calcule a idade de Cludia.
23- Carla e Fernando receberam R$ 6.000,00 de sua me para guardar. Carla aplicou a sua parte a 20% ao ms, e
Fernando gastou a metade e aplicou a outra metade a 30%. Ao fim de 30 dias, eles tero juntos R$ 5.000,00. Quanto
coube inicialmente a cada um ?
24- As idades de pai e filho somam hoje 52 anos; mas h 10 anos, a idade do pai era 15 vezes a idade do filho.
Determine as idades.
25 - Um pai tem hoje 54 anos e seus 4 filhos tem juntos 39 anos. Daqui a quantos anos a idade do pai ser igual
soma das idades de seus 4 filhos ?
26 - Em uma classe de 50 alunos, o nmero de moas excede em 5 o dobro do nmero de rapazes. Qual o
nmero de rapazes dessa classe ?
27 - Determine uma frao que se adicionarmos 4 unidades aos seus dois termos, ela ficar equivalente a 3/4, e
que se subtrairmos 2 unidades de ambos os termos, ela ficar equivalente a 1/2.
28 - misturando 2 litros de um xarope A com 3 litros de um xarope B, obtm-se um xarope de R$ 400,00 o litro.
Agora, se misturar 3 litros do xarope A com 2 litros do xarope B, obtm-se um produto de R$ 300,00 o litro. Qual o
preo de cada xarope ?
29 - Um livreiro vende, num dia, 3 exemplares de Lngua Portuguesa e 7 de Matemtica, recebendo R$ 3.240,00 No
dia seguinte, vende 2 de Lngua Portuguesa e 5 de Matemtica, e ento recebe R$ 2.260,00. Qual preo de cada
exemplar ?
30 - Certa quantidade de sacos precisam ser transportados e para isto dispe-se de jumentos. Se colocarmos dois
sacos em cada jumento, sobram treze sacos; se colocarmos trs sacos em cada jumento, sobram trs jumentos.
Quantos sacos precisam ser carregados? (TTN)
31 Em um nibus, se em cada banco for ocupado por 2 pessoas, ficam 5 pessoas em p. Se em cada banco
tomarem assento 3 pessoas, sobram 5 bancos vazios. Quantas pessoas esto sendo transportadas ?
32 - Eu tenho o dobro da idade que tu tinhas, quando eu tinha a idade que tens. Quando tiveres a idade que eu
tenho, juntos somaremos, hoje, 63 anos. Quais as nossas idades hoje?
33 - A um aluno propuseram o seguinte problema: "Um nmero tal que multiplicado por 3/4 diminui de 5 unidades;
dividido por 4/5 aumenta de 5 unidades; adicionando-se-lhe 10 unidades obtm-se outro nmero que 3/2 do
nmero dado. "
O aluno respondeu que o problema impossvel porque, embora as duas primeiras partes do problema fossem
possveis, o mesmo no se verifica em relao ao ltimo item.
Nestas condies, correto afirmar que:
a) o aluno acertou na resposta que deu;
b) o aluno errou porque o problema s se verifica em relao s duas ltimas partes;
c) o aluno errou porque o problema possvel;
d) o aluno errou porque o problema s possvel em relao primeira e a ltima parte.

62

__________________________________________________________________________________________________________________________

ATUALIZADO AT MAIO/2012

www.CARREIRAPUBLICA.com.br

(48) 4141-3220

4141-3222

____________

_____________________________ MATEMTICA _________________________________ Prof. Roberto

34 Lus e Maria resolveram comparar suas colees de compact disc . Descobriram que tm ao todo 104 CDs e
que se Maria tivesse 12 CDs a menos teria o triplo do nmero de CDs do Lus. possvel afirmar que a quantidade
de CDs que Lus possui :
a) 46 b) 40 c) 32 d) 23
35 Em um restaurante h 12 mesas, todas ocupadas. Algumas por 4 pessoas, outras por apenas 2 pessoas num
total de 38 fregueses. Qual o nmero de mesas ocupadas por apenas duas pessoas?
a) 4

b) 5

c) 6 d) 7

36 Um aluno ganha 5 pontos por exerccios que acerta e perde 3 por exerccio que erra. Ao fim de 50 exerccios,
tinha 130 pontos. Quantos exerccios acertou?
a) 35

b) 30 c) 25 d) 15

37 Em um restaurante existem mesas de 3, 4 e 6 cadeiras num total de 16 mesas. Ocupando todos os lugares nas
mesas de 3 e 4 cadeiras, 36 pessoas ficam perfeitamente acomodadas. Sabendo-se que o restaurante acomoda no
mximo 72 pessoas, quantas mesas de cada tipo ( 3, 4 e 6) , respectivamente, existem?
a) 6, 4 e 6 b) 6, 6 e 4 c) 4, 6 e 6

d) 3, 7 e 6

38 Um jogador de basquete fez o seguinte acordo com seu clube: cada vez que ele convertesse um arremesso,
receberia R$ 10,00 do clube e cada vez que ele errasse pagaria R$ 5,00 ao clube. Ao final de uma partida em que
arremessou 20 vezes, ele recebeu R$ 50,00. Pode-se afirmar que o nmero de arremessos convertidos pelo jogador
foi:
a) 0

b) 5 c) 10

d) 15

39 Um copo cheio tem massa de 385g; com 2/3 de gua tem massa de 310g. A massa do copo com 3/5 da gua
:
a) 160 g b) 225 g c) 260 g

d) 295 g

40 Num escritrio de advocacia trabalhavam apenas dois advogados e uma secretria. Como Dr. Andr e Dr.
Carlos sempre advogam em causas diferentes, a secretria, Cludia, coloca um grampo em cada processo do Dr.
Andr e dois grampos em cada processo do Dr. Carlos, para diferenci-los facilmente no arquivo. Sabendo-se que ao
todo so 78 processos, nos quais foram usados 110 grampos, podemos concluir que o nmero de processos do Dr.
Carlos igual a:
a) 64

b) 46 c) 40 d) 32

41 - Uma pessoa retira R$ 70,00 de um banco, recebendo 10 notas, algumas de R$ 10,00 e outras de R$ 5,00.
Calcule quantas notas de R$ 5,00 a pessoa recebeu.
a) 10 b) 6

c) 4 d) 2

42 Numa lanchonete, 2 copos de refrigerantes e 3 coxinhas custam R$ 5,70. O preo de 3 copos de refrigerantes
e 5 coxinhas R$ 9,30. Nessas condies, verdade que cada copo de refrigerante custa:
a) R$ 0,70 a menos que cada coxinha.
c)R$ 0,90 a menos que cada coxinha.

b) R$ 0,80 a menos que cada coxinha.


d) R$ 0,80 a mais que cada coxinha.

43 Carlos e sua irm Andria foram com seu cachorro Bidu farmcia de seu av. L encontraram uma velha
balana com defeito que s indicava corretamente pesos superiores a 60kg. Assim eles se pesam dois a dois e
obtiveram as seguintes marcas:

Carlos e o co pesam juntos 87kg;


Carlos e Andra pesam 123kg e

__________________________________________________________________________________________________________________________

ATUALIZADO AT MAIO/2012

www.CARREIRAPUBLICA.com.br

(48) 4141-3220

4141-3222

63

____________

_____________________________ MATEMTICA _________________________________ Prof. Roberto

Andria e Bidu pesam 66kg.

Podemos afirmar que:


a.
b.
c.
d.

Cada um deles pesa menos que 60kg


Dois deles pesam mais de 60kg
Andria a mais pesada dos trs
Carlos mais pesado que Andria e Bidu juntos.

44 -

x + y = 5

x y =1

45 -

2 x y = 3

3 x + y = 7

46 -

3x + 2 y = 6

2 x + y = 3
2

47 - Uma empreiteira destinou originalmente alguns operrios para a construo de uma obra de 72m . Como 4 deles
2
foram demitidos antes do incio da obra, os demais tiveram que trabalhar 9m a mais cada um para compensar.
a) Qual o nmero de operrios originalmente designados para a obra?
b) Qual a porcentagem de operrios demitidos?
48 - Helena tem 3 anos a mais que Ana. Sabendo que a soma das idades 57 anos, qual a idade de Ana?
49 - ( FEPESE-2007)Numa caixa o nmero de bolas vermelhas o triplo de bolas brancas. Se tirarmos 2 bolas
brancas e 26 bolas vermelhas, o nmero de bolas de cada cor ficar igual. A quantidade de bolas brancas encontrada
:
a) 42 b) 24 c) 21 d) 13 e) 12
50 - Os alunos de uma turma resolveram comprar um presente custando R$ 48,00 para o professor de Matemtica,
dividindo igualmente o gasto entre eles. Depois que 6 alunos recusaram-se a participar da diviso, cada um dos
alunos restantes teve que contribuir com mais R$ 0,40 para a compra do presente. Qual a percentagem de alunos da
turma que contriburam para a compra do presente?
a) 85%
b) 65%
c) 60%
d) 80%
e) 75%
51 - (FUVEST 84) Um copo cheio de gua pesa 325g. Se jogarmos metade da gua fora, seu peso cai para 180g. O
peso do copo vazio :
a) 20g b) 25g c) 35g d) 40g e) 45g
52 - A soma de trs nmeros racionais igual a 521. O maior nmero igual ao dobro do menor deles e o outro
nmero tem 5 unidades a mais que o nmero menor. Qual o valor desses trs nmeros?
53 - Um pai tinha 27 anos quando seu filho nasceu. Hoje, a idade do pai o qudruplo da idade do filho.
A atual idade do pai :
a) 40 anos b) 36 anos c) 32 anos d) 44 anos
54 - A soma das idades de um pai e um filho 42 anos. H 3 anos passados, a idade do pai era onze vezes a idade
do filho. Determine a idade atual do pai
55 - (Soldado-PM) Duas amigas foram juntas feira. Uma delas comprou meia dzia de bananas e dois meles,
gastando R$ 10,80. A outra gastou R$ 11,40, comprando meia dezena de bananas e trs meles. A razo entre os
preos de uma banana e de um melo :
a)

3
5

64

b)

5
3
2
c)
d)
3
2
3

__________________________________________________________________________________________________________________________

ATUALIZADO AT MAIO/2012

www.CARREIRAPUBLICA.com.br

(48) 4141-3220

4141-3222

____________

_____________________________ MATEMTICA _________________________________ Prof. Roberto

1
1
e z = 1 assinale a opo que representa o valor de z.
x
y
2
1
1
a) z = x b) z =
c) z =
d) z = 1 x e) z =
x
1 x
x

56 - ( FEPESE )Seja y = 1

57 - Em agosto de 2000, Zuza gastou R$192,00 na compra de algumas peas de certo artigo. No ms seguinte, o
preo unitrio desse artigo aumentou R$8,00 e, com a mesma quantia que gastou em agosto, ele pode comprar duas
peas a menos. Em setembro, o preo de cada pea de tal artigo era
a) R$ 24,00 b) R$ 25,00 c) R$ 28,00 d) R$ 30,00 e) R$ 32,00
58 - (FEPESE -2006) Uma papelaria vende cadernos fabricados por duas empresas, denominadas por A e B. Ao se
analisar as vendas destes produtos durante dois meses consecutivos, obteve-se o seguinte resultado:
1 dia: foram vendidas 20 cadernos da marca A e 30 cadernos da marca B, resultando num total de vendas de R$
310,00.
2 dia: foram vendidas 10 cadernos da marca A e 20 cadernos da marca B, resultando num total de vendas de R$
190,00.
Assinale a alternativa que indica a razo entre o preo do caderno da marca B e o preo do caderno da marca A.
a) 0,7 b) 1 c) 2 d) 1,4 e) 1,5
59 - ( ACAFE) Uma concessionria est fazendo uma promoo de 51 automveis de trs marcas diferentes, a, b e c.
O nmero de veculos da marca b igual a 8/5 da a e o dobro da c. A diferena entre o nmero de automveis das
marcas a e c :
a) 12 b) 3 c) 9 d) 7 e) 1
60 - ( UNICAMP) Uma transportadora entrega, com caminhes, 60 toneladas de acar por dia. Devido a problemas
operacionais, em um certo dia cada caminho foi carregado com 500kg a menos que o usual, tendo sido necessrio,
naquele dia, alugar mais 4 caminhes.
a) Quantos caminhes foram necessrios naquele dia?
b) Quantos quilos transportou cada caminho naquele dia?
61 - O grfico da funo f(x) = ax + b est representado na figura.

O valor de a + b :
a) -1 b) 2/5 c) 3/2 d) 2
62 - ( CEFET-SC ) Uma agncia de aluguel de automveis cobra R$ 40,00 por dia mais R$ 0,50 por Km rodado.
Quanto custa alugar um carro para uma viagem de um dia, percorrendo 250 Km ? Uma pessoa pagou R$ 89,00 pelo
aluguel de um dia quantos Km rodou ?
a) R$ 125,00 e 89Km b) R$ 125,00 e 98Km
c) R$ 165,00 e 89Km d) R$ 165,00 e 98Km
63 - O grfico a seguir fornece o perfil do lucro de uma empresa agrcola ao longo do tempo, sendo 1969 o ano zero,
ou seja, o ano de sua fundao. Analisando o grfico, podemos afirmar que:

__________________________________________________________________________________________________________________________

ATUALIZADO AT MAIO/2012

www.CARREIRAPUBLICA.com.br

(48) 4141-3220

4141-3222

65

____________

(
(
(
(
(

)
)
)
)
)

_____________________________ MATEMTICA _________________________________ Prof. Roberto

10 foi o nico ano em que ela foi deficitria.


20 foi o ano de maior lucro.
25 foi um ano deficitrio.
15 foi um ano de lucro.
5 foi o ano de maior lucro no perodo que vai da fundao at o ano 15.

64 - Na figura a seguir tem-se o grfico da funo f, onde f(x) representa o preo pago em reais por x cpias de um
mesmo original, na Copiadora Reprodux

De acordo com o grfico, verdade que o preo pago nessa Copiadora por
a) 228 cpias de um mesmo original R$22,50.
b) 193 cpias de um mesmo original R$9,65.
c) 120 cpias de um mesmo original R$7,50.
d) 100 cpias de um mesmo original R$5,00
e) 75 cpias de um mesmo original R$8,00.
65 - (Unb) Cada bilhete vendido em um parque de diverses d direito utilizao de apenas um brinquedo, uma
nica vez. Esse parque oferece aos usurios trs opes de pagamento:
I. R$ 2,00 por bilhete;
II. valor fixo de R$ 10,00 por dia, acrescido de R$ 0,40 por bilhete;
III. valor fixo de R$ 16,00 por dia, com acesso livre aos brinquedos.
Com base nessa situao, julgue os itens a seguir.
(1) Se uma criana dispe de R$ 14,00, a opo I a que lhe permite utilizar o maior nmero de brinquedos.
(2) Se x representa o nmero de vezes que uma pessoa utiliza os brinquedos do parque, a funo f que descreve a
despesa diria efetuada, em reais, ao se utilizar a opo III, dada por f(x)=16x.
(3) possvel a um usurio utilizar determinado nmero de brinquedos em um nico dia, de modo que a sua despesa
total seja a mesma, independente da opo de pagamento escolhida.
66 - A promoo de uma mercadoria em um supermercado est representada, no grfico a seguir, por 6 pontos de
uma mesma reta.

66

__________________________________________________________________________________________________________________________

ATUALIZADO AT MAIO/2012

www.CARREIRAPUBLICA.com.br

(48) 4141-3220

4141-3222

____________

_____________________________ MATEMTICA _________________________________ Prof. Roberto

Quem comprar 20 unidades dessa mercadoria, na promoo, pagar por unidade, em reais, o equivalente a:
a) 4,50 b) 5,00 c) 5,50 d) 6,00
67 - Em uma partida, Vasco e Flamengo levaram ao Maracan 90.000 torcedores. Trs portes foram abertos s 12
horas e at as 15 horas entrou um nmero constante de pessoas por minuto. A partir desse horrio, abriram-se mais
3 portes e o fluxo constante de pessoas aumentou.
Os pontos que definem o nmero de pessoas dentro do estdio em funo do horrio de entrada esto contidos no
grfico a seguir:

Quando o nmero de torcedores atingiu 45.000, o relgio estava marcando 15 horas e:


a) 20 min b) 30 min c) 40 min d) 50 min
68 - ( TCNICO-INSS ) Seu Manoel comprou uma saca que ele pensava conter 100 Kg de feijo por R$ 81,00.
Depois de empacotar o feijo em sacos de 2 Kg, seu Manoel contou apenas 45 sacos, ou seja, havia na saca menos
feijo do que ele pensava. Na realidade, quanto ele pagou, em reais, por cada quilo de feijo ?
a) 0,80 b) 0,81
c)0,90 d) 1
69 - Para publicar certo livro, h um investimento inicial de R$200.000,00 e depois um gasto de R$5,00 por exemplar.
Calculando-se o custo por exemplar, numa tiragem de 4000 exemplares e numa tiragem de 16.000 exemplares,
obtm-se respectivamente.
a) R$ 55,00 e R$ 22,00 b) R$ 55,00 e R$ 13,75
c) R$ 105,00 e R$ 30,00 d) R$ 55,00 e R$ 17,50
e) R$ 105,00 e R$ 26,25
70 - (PRF-1998) Num determinado Estado, quando um veculo rebocado por estacionar em Local proibido, o
motorista paga uma taxa fixa de R$ 76,88 e mais R$ 1,25 por hora de permanncia no estacionamento da polcia. Se
o valor pago foi de R$ 101,88 o total de horas que o veculo ficou estacionado na polcia corresponde a :
a) 20 b) 21 c) 22 d) 23 e) 24
71 - Sabedoria egpcia
H mais de 5.000 anos os egpcios observaram que a sombra no cho provocada pela incidncia dos raios solares
de um gnmon (um tipo de vareta) variava de tamanho e de direo. Com medidas feitas sempre ao meio dia,
notaram que a sombra, com o passar dos dias, aumentava de tamanho. Depois de chegar a um comprimento
mximo, ela recuava at perto da vareta. As sombras mais longas coincidiam com dias frios. E as mais curtas, com
dias quentes.
(Adaptado de Revista "Galileu", janeiro de 2001.)
__________________________________________________________________________________________________________________________

ATUALIZADO AT MAIO/2012

www.CARREIRAPUBLICA.com.br

(48) 4141-3220

4141-3222

67

____________

_____________________________ MATEMTICA _________________________________ Prof. Roberto

Um estudante fez uma experincia semelhante descrita no texto, utilizando uma vareta OA de 2 metros de
comprimento. No incio do inverno, mediu o comprimento da sombra OB, encontrando 8 metros.
Utilizou, para representar sua experincia, um sistema de coordenadas cartesianas, no qual o eixo das ordenadas (y)
e o eixo das abscissas (x) continham, respectivamente, os segmentos de reta que representavam a vareta e a
sombra que ela determinava no cho.
Esse estudante pde, assim, escrever a seguinte equao da reta que contm o segmento AB:
a) y = 8 - 4x b) x = 6 - 3y c) x = 8 - 4y d) y = 6 - 3x
72 - Os dados experimentais da tabela a seguir correspondem s concentraes de uma substncia qumica medida
em intervalos de 1 segundo. Assumindo que a linha que passa pelos trs pontos experimentais uma parbola, temse que a concentrao (em moles) aps 2,5 segundos :
Tempo (s)
Concentrao (moles)
1
3,00
2
5,00
3
1,00
a) 3,60 b) 3,65 c) 3,70 d) 3,75 e) 3,80
73 - Usando uma unidade monetria conveniente, o lucro obtido com a venda de uma unidade de certo produto x10, sendo x o preo de venda e 10 o preo de custo. A quantidade vendida, a cada ms, depende do preo de venda
e , aproximadamente, igual a 70-x.
Nas condies dadas, o lucro mensal obtido com a venda do produto , aproximadamente, uma funo quadrtica de
x, cujo valor mximo, na unidade monetria usada,
a) 1200 b) 1000 c) 900 d) 800 e) 600
74 - Observe o grfico:
Crepsculo da garrafa azul Os brasileiros esto trocando o vinho branco alemo por produto de melhor qualidade (em
milhes de litros).

("Veja", 13/09/1999) Se o consumo de vinho branco alemo, entre 1994 e 1998, sofreu um decrscimo linear, o
volume total desse consumo em 1995, em milhes de litros, corresponde a:
a) 6,585 b) 6,955 c) 7,575 d) 7,875
75 - Para produzir um objeto, um arteso gasta R$ 1,20 por unidade. Alm disso, ele tem uma despesa fixa de
R$123,50 , independente da quantidade de objetos produzidos. O preo de venda de R$ 2,50 por unidade. O

68

__________________________________________________________________________________________________________________________

ATUALIZADO AT MAIO/2012

www.CARREIRAPUBLICA.com.br

(48) 4141-3220

4141-3222

____________

_____________________________ MATEMTICA _________________________________ Prof. Roberto

nmero mnimo de objetos que o arteso deve vender, para que recupere o capital empregado na produo dos
mesmos, :
76 - O balano de clcio a diferena entre a quantidade de clcio ingerida e a quantidade excretada na urina e nas
fezes. usualmente positivo durante o crescimento e a gravidez e negativo na menopausa, quando pode ocorrer a
osteoporose, uma doena caracterizada pela diminuio da absoro de clcio pelo organismo. A baixa concentrao
++
de on clcio (Ca ) no sangue estimula as glndulas paratireides a produzirem hormnio paratireideo (HP). Nesta
situao, o hormnio pode promover a remoo de clcio dos ossos, aumentar sua absoro pelo intestino e reduzir
sua excreo pelos rins.
(Adaptado de ALBERTS, B. et al., "Urologia Molecular da Clula." Porto Alegre: Artes Mdicas, 1997.)
Admita que, a partir dos cinqenta anos, a perda da massa ssea ocorra de forma linear conforme mostra o grfico
abaixo.

(Adaptado de "Galileu", janeiro de 1999.)


Aos 60 e aos 80 anos, as mulheres tm, respectivamente, 90% e 70% da massa ssea que tinham aos 30 anos. O
percentual de massa ssea que as mulheres j perderam aos 76 anos, em relao massa aos 30 anos, igual a:
a) 14 b) 18 c) 22 d) 26
77 - Muitos restaurantes adotam o sistema de comida a kilo, isto , o fregus paga pela quantidade de alimentos
que consome. Num desses restaurantes o preo do quilo R$ 8,00. Uma pessoa que consome 340 gramas de
alimento e toma 2 latas de refrigerante, no valor de R$ 0,80 cada uma, pagar pela conta, em reais:
a) 2,72 b) 4,32 c) 3,62 d) 3,52 e) 4,52
78 - (SOLDADO-PM) A cidade de Campos de Jordo fica numa regio cujas temperaturas so as mais baixas do
pas. O grfico a seguir descreve a temperatura nessa regio, das 2 s 8 horas da noite de um dia de inverno. Em
que horrio, a temperatura atingiu 0C, nesse dia?

a) 3 horas e 30 minutos b) 3 horas

c) 4 horas

d) 2horas

79 - Um fabricante vende um produto por R$ 0,80 a unidade. O custo total do produto consiste numa taxa fixa de R$
40,00, mais o custo de produo, de R$ 0,30 por unidade. Se o fabricante vender 200 unidades desse produto, ele
ter:
a) um lucro de R$ 100,00. b) um prejuzo de R$ 60,00.
c) nem lucro nem prejuzo. d) um lucro de R$ 60,00.
e) um prejuzo de R$ 100,00.

__________________________________________________________________________________________________________________________

ATUALIZADO AT MAIO/2012

www.CARREIRAPUBLICA.com.br

(48) 4141-3220

4141-3222

69

____________

_____________________________ MATEMTICA _________________________________ Prof. Roberto

80 - ( TCNICO INSS) Um motorista parou num posto para abastecer seu caminho com leo diesel, ele pagou
com uma nota de R$ 100,00 e recebeu R$ 5,75 de troco. Se o litro do leo diesel custava R$ 1,450, quantos litros ele
comprou ?
a) 55 b) 58 c) 65 d) 75 e) 78
81 - Um recipiente, contendo uma barra de gelo que se encontra a -30C, colocado para aquecer sobre a chama de
um fogo. O grfico abaixo indica a evoluo da temperatura (T) em funo do tempo (t), em minutos. Assinale a
alternativa correta.

a) T= 25t - 175 para 7 t < 11.


b) T= 0 para 0 t < 7.
c) T= -10t - 30 para 0 t < 3.
d) T= 100 para t > 7.
e) T= -10 para t = 1.
82 - O preo total cobrado por um eletricista A inclui uma parte fixa, referente visita, e outra que depende da
quantidade de metros de fio utilizada no servio. O grfico abaixo apresenta o valor do servio efetuado pelo
eletricista A em funo do nmero de metros de fio utilizados. O preo cobrado por um outro eletricista B depende
unicamente do nmero de metros de fio utilizado, no sendo cobrada a visita. O preo do servio de R$ 3,50 por
metro de fio utilizado. Com base no exposto, est correta a afirmao da alternativa:

a) Se forem utilizados 40 metros de fio, o preo cobrado pelos eletricistas A e B ser o mesmo.
b) O eletricista A cobra R$ 2,50 por metro de fio utilizado.
c) A parte fixa cobrada pelo eletricista A de R$ 30,00.
d) Por 50m de fio, o eletricista A cobrar R$ 190,00.
e) Sendo necessrios 60 metros de fio, convm contratar o eletricista B.
83 - Um comerciante paga R$ 7,00 por 3 unidades de uma mercadoria, e revende por R$ 18,00 cada 5 unidades. Na
comercializao dessa mercadoria, ele obtm um lucro de R$ 342,00 quando vende um total de unidades igual a
a) 210. b) 240. c) 270. d) 300. e) 330.
84 - O grfico a seguir descreve o crescimento em determinada populao de uma doena infecto contagiosa em
certo vilarejo desde 1910 at 1990. No eixo das ordenadas, a populao infectada dada em milhares de habitantes.

70

__________________________________________________________________________________________________________________________

ATUALIZADO AT MAIO/2012

www.CARREIRAPUBLICA.com.br

(48) 4141-3220

4141-3222

____________

_____________________________ MATEMTICA _________________________________ Prof. Roberto

a) Determine em que dcada a populao infectada atingiu a marca de 5.000 habitantes.


b) Observe que a partir de 1960 o crescimento da populao infectada em cada dcada tem se mantido constate.
Suponha que esta taxa se mantenha inalterada no futuro.
Determine em que dcada o vilarejo ter 20.000 habitantes infectados.
85 - (ACAFE) Dois atletas A e B fazem teste de Cooper numa pista retilnea, ambos correndo com velocidade
constante. A distncia (d) que cada um percorre mostrada no grfico abaixo.

d(m)
500
400
300
200
100
0

10 20 30

x
t(min)

Com base no grfico, a alternativa correta :


a) A mais veloz que B, pois percorre 600m em 20 min.
b) B percorre 1km em 20 min.
c) B mais veloz que A, pois percorre 400m em 5 min.
d) A e B correm na mesma velocidade.
e) A percorre 400m em 30 min.

__________________________________________________________________________________________________________________________

ATUALIZADO AT MAIO/2012

www.CARREIRAPUBLICA.com.br

(48) 4141-3220

4141-3222

71

____________

_____________________________ MATEMTICA _________________________________ Prof. Roberto

Gabarito
1) a) 6 b) 4 c) 3/4
d) 2 e) 5 f) 1/3 g)
84
2) 4
3)
4) 690
5) 621,60
6) 1200
7) 10
8) a = b
9) 4
10) b
11) e
12) e
13) 11
14) 2200
15) No/Sim
16) 55000
17) 30
18) 28

23) 200 e 4000

45) (2;1)

67) b

24) 40 e 12
25) 5
26) 15
27) 5/8
28) 100 e 600
29)300 e 380
30) 57
31) 45
32) 36 e 27
33) c
34) d
35) b
36) a
37) c
38) c
39) d
40) d

46) (0;3)
47) a) 8 b) 50%
48) 27
49) e
50) d
51) c
52) 258; 134; 129
53) b
54) 36
55) d
56) c
57) e
58) d
59) b
60) a) 28 b) 2000
61) c
62) d

19) 16/24
20) 52
21) 5
22) 12

41) b
42) 1
43) d
44)(3;2)

63) FVFFV
64) b
65) FFF
66) a

68) c
69) d
70) a
71) c
72) d
73) c
74) d
75) 95
76) d
77) b
78) c
79) d
80) c
81) a
82) a
83) c
84) a) 40
b) entre 2040 e 2050
85)b

11 Equaes de 2 grau

11.1 Definies
Denomina-se equao do 2 grau na incgnita x, toda equao da forma:
2

ax + bx + c = 0; a, b, c

IR e

Exemplo:

x - 5x + 6 = 0 um equao do 2 grau com a = 1,


2
6x - x - 1 = 0 um equao do 2 grau com a = 6,
2
7x - x = 0
um equao do 2 grau com a = 7,
2
um equao do 2 grau com a = 1,
x - 36 = 0

b = -5 e c = 6.
b = -1 e c = -1.
b = -1 e c = 0.
b = 0 e c = -36.

Nas equaes escritas na forma ax + bx + c = 0 (forma normal ou forma reduzida de uma equao do 2 grau
na incgnita x) chamamos a, b e c de coeficientes.
a

sempre o coeficiente de x;

sempre o coeficiente de x,

o coeficiente ou termo independente.

11.2 Equao completas e Incompletas

72

__________________________________________________________________________________________________________________________

ATUALIZADO AT MAIO/2012

www.CARREIRAPUBLICA.com.br

(48) 4141-3220

4141-3222

____________

_____________________________ MATEMTICA _________________________________ Prof. Roberto

Uma equao do 2 grau completa quando b e c so diferentes de zero. Exemplos: x - 9x + 20 = 0 e -x + 10x


- 16 = 0 so equaes completas.
Uma equao do 2 grau incompleta quando b ou c igual a zero, ou ainda quando ambos so iguais a zero.
Exemplos:

x - 36 = 0
(b = 0)

x - 10x = 0
(c = 0)

4x = 0
(b = c = 0)

11.3 Razes de uma equao do 2 grau


Resolver uma equao do 2 grau significa determinar suas razes.
Raiz o nmero real que, ao substituir a incgnita de uma equao, transforma-a numa
sentena verdadeira.
O conjunto formado pelas razes de uma equao denomina-se conjunto verdade ou conjunto soluo.
Exemplos:

Dentre os elementos do conjuntos A= {-1, 0, 1, 2}, quais so razes da equao x - x - 2 = 0 ?

Soluo
Substitumos a incgnita x da equao por cada um dos elementos do conjunto e verificamos quais as
sentenas verdadeiras.

Para x = -1

Para x = 0

Para x = 1

Para x = 2

(-1) - (-1) - 2 = 0
1+1-2=0
0=0
0 - 0 - 2 = 0
0 - 0 -2 = 0
-2 = 0
1 - 1 - 2 = 0
1-1-2=0
-2 = 0
2 - 2 - 2 = 0
4-2-2=0
0=0

(V)

(F)

(F)

(V)

Logo, -1 e 2 so razes da equao.

Determine p sabendo que 2 raiz da equao (2p - 1) x - 2px - 2 = 0.


Soluo
Substituindo a incgnita x por 2, determinamos o valor de p.

__________________________________________________________________________________________________________________________

ATUALIZADO AT MAIO/2012

www.CARREIRAPUBLICA.com.br

(48) 4141-3220

4141-3222

73

____________

_____________________________ MATEMTICA _________________________________ Prof. Roberto

Logo, o valor de p

11.4 Resoluo de equaes incompletas


Resolver uma equao significa determinar o seu conjunto verdade.
Utilizamos na resoluo de uma equao incompleta as tcnicas da fatorao e duas importantes propriedades
dos nmeros reais:
1 Propriedade:
2 Propriedade:
1 Caso: Equao do tipo

Exemplo:

Determine as razes da equao

, sendo

Soluo
Inicialmente, colocamos x em evidncia:

Para o produto ser igual a zero, basta que um dos fatores tambm o seja. Assim:

Obtemos dessa maneira duas razes que formam o conjunto verdade:

De modo geral, a equao do tipo

tem para solues

2 Caso: Equao do tipo


Exemplos:

Determine as razes da equao

, sendo U = IR.

Soluo

74

__________________________________________________________________________________________________________________________

ATUALIZADO AT MAIO/2012

www.CARREIRAPUBLICA.com.br

(48) 4141-3220

4141-3222

____________

_____________________________ MATEMTICA _________________________________ Prof. Roberto

De modo geral, a equao do tipo


raiz real caso

possui duas razes reais se

for um nmero positivo, no tendo

seja um nmero negativo.

11.5 Resoluo de equaes completas


Para solucionar equaes completas do 2 grau utilizaremos a frmula de Bhaskara.
, em que a, b, c
A partir da equao
da frmula de Bhaskara (ou frmula resolutiva).

IR e

, desenvolveremos passo a passo a deduo

1 passo: multiplicaremos ambos os membros por 4a.

2 passo: passar 4ac par o 2 membro.

3 passo: adicionar

aos dois membros.

4 passo: fatorar o 1 elemento.

5 passo: extrair a raiz quadrada dois membros.

6 passo: passar b para o 2 membro.

7 passo: dividir os dois membros por

Assim, encontramos a frmula resolutiva da equao do 2 grau:

__________________________________________________________________________________________________________________________

ATUALIZADO AT MAIO/2012

www.CARREIRAPUBLICA.com.br

(48) 4141-3220

4141-3222

75

____________

_____________________________ MATEMTICA _________________________________ Prof. Roberto

Podemos representar as duas razes reais por x' e x", assim:

Exemplos:

resoluo a equao:
Temos

11.6 Discriminante
2

Denominamos discriminante o radical b - 4ac que representado pela letra grega

(delta).

Podemos agora escrever deste modo a frmula de Bhaskara:

De acordo com o discriminante, temos trs casos a considerar:

76

__________________________________________________________________________________________________________________________

ATUALIZADO AT MAIO/2012

www.CARREIRAPUBLICA.com.br

(48) 4141-3220

4141-3222

____________

_____________________________ MATEMTICA _________________________________ Prof. Roberto

1 Caso: O discriminante positivo


O valor de

real e a equao tem duas razes reais diferentes, assim representadas:

Exemplo:

Para quais valores de k a equao x - 2x + - 2 = 0 admite razes reais e desiguais?


Soluo
Para que a equao admita razes reais e desiguais, devemos ter

Logo, os valores de k devem ser menores que 3.


2 Caso: O discriminante nulo
O valor de

nulo e a equao tem duas razes reais e iguais, assim representadas:

Exemplo:

Determine o valor de p, para que a equao x - (p - 1) x + p = 0


Soluo
Para que a equao admita razes iguais necessrio que

Logo, o valor de p 3.
__________________________________________________________________________________________________________________________

ATUALIZADO AT MAIO/2012

www.CARREIRAPUBLICA.com.br

(48) 4141-3220

4141-3222

77

____________

_____________________________ MATEMTICA _________________________________ Prof. Roberto

3 Caso: O discriminante negativo

no existe em IR, no existindo, portanto, razes reais. As razes da equao so nmero

O valor de
complexos.

Exemplo:

Para quais valores de m a equao 3x + 6x +m = 0 no admite nenhuma raiz real?


Soluo
Para que a equao no tenha raiz real devemos ter

Logo, os valores de m devem ser maiores que 3.


Resumindo
Dada a equao ax + bx + c = 0, temos:
Para

, a equao tem duas razes reais diferentes.

Para

, a equao tem duas razes reais iguais.

Para

, a equao no tem razes reais.

11.7 Equaes literais incompletas


A resoluo de equaes literais incompletas segue o mesmo processo das equaes numricas.
Observe os exemplos:

Resolva a equao literal incompleta 3x - 12m =0, sendo x a varivel.


Soluo
2

3x - 12m = 0
2

3x = 12m
2

x = 4m

x=

78

__________________________________________________________________________________________________________________________

ATUALIZADO AT MAIO/2012

www.CARREIRAPUBLICA.com.br

(48) 4141-3220

4141-3222

____________

_____________________________ MATEMTICA _________________________________ Prof. Roberto

Logo, temos:
2

Resolva a equao literal incompleta my - 2aby=0,com m

0, sendo y a varivel.

Soluo
2

my - 2aby = 0
y(my - 2ab)=0
Temos, portanto, duas solues:
y=0
ou

my - 2ab = 0

my = 2ab

y=

Assim:

Na soluo do ltimo exemplo, teramos cometido um erro grave se tivssemos assim resolvido:
2

my - 2aby= 0
2

my = 2aby
my = 2ab

Desta maneira, obteramos apenas a soluo

O zero da outra soluo foi "perdido" quando dividimos ambos os termos por y.
Esta uma boa razo para termos muito cuidado com os cancelamentos, evitando desta maneira a diviso por zero,
que um absurdo.
11.8 Equaes literais completas
As equaes literais completas podem ser tambm resolvidas pela frmula de Bhaskara:
Exemplo:
2

2 2

Resolva a equao: x - 2abx - 3a b , sendo x a varivel.


Soluo
2 2

Temos a=1, b = -2ab e c=-3a b

__________________________________________________________________________________________________________________________

ATUALIZADO AT MAIO/2012

www.CARREIRAPUBLICA.com.br

(48) 4141-3220

4141-3222

79

____________

_____________________________ MATEMTICA _________________________________ Prof. Roberto

Portanto:

Assim, temos: V= { - ab, 3ab}.

11.9 Relaes entre os coeficientes e as razes


2

Considere a equao ax + bx + c = 0, com a

0 e sejam x'e x'' as razes reais dessa equao.

Logo:

Observe as seguintes relaes:

80

Soma das razes (S)

Produto das razes (P)

__________________________________________________________________________________________________________________________

ATUALIZADO AT MAIO/2012

www.CARREIRAPUBLICA.com.br

(48) 4141-3220

4141-3222

____________

_____________________________ MATEMTICA _________________________________ Prof. Roberto

Como

,temos:

Denominamos essas relaes de relaes de Girard. Verifique alguns exemplos de aplicao dessas relaes.
2

Determine a soma e o produto das razes da equao 10x + x - 2 = 0.

Soluo
Nesta equao, temos: a=10, b=1 e c=-2.

A soma das razes igual a

Assim:

O produto das razes igual a

Assim:

Determine o valor de k na equao x + ( 2k - 3)x + 2 = 0, de modo que a soma de suas razes seja igual a 7.

Soluo
Nesta equao, temos: a=1, b=2k e c=2.
S= x1 + x2 = 7

Logo, o valor de k -2.

Determine o valor de m na equao 4x - 7x + 3m = 0, para que o produto das razes seja igual a -2.

Soluo
Nesta equao, temos: a=4, b=-7 e c=3m.
P= x1. x2= -2

Logo, o valor de m

__________________________________________________________________________________________________________________________

ATUALIZADO AT MAIO/2012

www.CARREIRAPUBLICA.com.br

(48) 4141-3220

4141-3222

81

____________

_____________________________ MATEMTICA _________________________________ Prof. Roberto


2

Determine o valor de k na equao 15x + kx + 1 = 0, para que a soma dos diversos de suas razes seja
igual a 8.

Soluo
Considere x1 e x2 as razes da equao.

A soma dos inversos das razes corresponde a

Assim:

Logo, o valor de k -8.


2

Determine os valores de m para os quais a equao ( 2m - 1) x + ( 3m - 2) x + m + 2 = 0 admita:


a) razes simtricas;
b) razes inversas.

Soluo
Se as razes so simtricas, ento S=0.

Se as razes so inversas, ento P=1.

11.10 Composio de uma equao do 2 grau, conhecidas as razes


2

Considere a equao do 2 grau ax + bx + c = 0.

82

__________________________________________________________________________________________________________________________

ATUALIZADO AT MAIO/2012

www.CARREIRAPUBLICA.com.br

(48) 4141-3220

4141-3222

____________

_____________________________ MATEMTICA _________________________________ Prof. Roberto

Dividindo todos os termos por a

Como

, obtemos:

, podemos escrever a equao desta maneira.


2

x - Sx + P= 0

Exemplos:

Componha a equao do 2 grau cujas razes so -2 e 7.

Soluo
A soma das razes corresponde a:
S= x1 + x2 = -2 + 7 = 5
O produto das razes corresponde a:
P= x1 . x2 = ( -2) . 7 = -14
2

A equao do 2 grau dada por x - Sx + P = 0, onde S=5 e P= -14.


2

Logo, x - 5x - 14 = 0 a equao procurada.

Formar a equao do 2 grau, de coeficientes racionais, sabendo-se que uma das razes

Soluo
Se uma equao do 2 grau, de coeficientes racionais, tem uma raiz

, a outra raz ser

Assim:

Logo, x - 2x - 2 = 0 a equao procurada.


FORMA FATORADA
__________________________________________________________________________________________________________________________

ATUALIZADO AT MAIO/2012

www.CARREIRAPUBLICA.com.br

(48) 4141-3220

4141-3222

83

____________

_____________________________ MATEMTICA _________________________________ Prof. Roberto


2

Considere a equao ax + bx + c = 0.
Colocando a em evidncia, obtemos:

Ento, podemos escrever:

Logo, a forma fatorada da equao ax + bx + c = 0 :


a.(x - x') . (x - x'') = 0
Exemplos:

Escreva na forma fatorada a equao x - 5x + 6 = 0.

Soluo
2

Calculando as razes da equao x - 5x + 6 = 0, obtemos x1= 2 e x2= 3.


2

Sendo a= 1, x1= 2 e x2= 3, a forma fatorada de x - 5x + 6 = 0 pode ser assim escrita:


(x-2).(x-3) = 0

Escreva na forma fatorada a equao 2x - 20x + 50 = 0.

Soluo
2

Calculando as razes da equao 2x - 20x + 50 = 0, obtemos duas razes reais e iguais a 5.


2

Sendo a= 2, x1=x2= 5, a forma fatorada de 2x - 20x + 50 = 0 pode ser assim escrita:


2

2.(x - 5) (x - 5) = 0 ou 2. (x - 5) =0
2

Escreva na forma fatorada a equao x + 2x + 2 = 0.


Soluo
Como o

84

, a equao no possui razes reais.

__________________________________________________________________________________________________________________________

ATUALIZADO AT MAIO/2012

www.CARREIRAPUBLICA.com.br

(48) 4141-3220

4141-3222

____________

_____________________________ MATEMTICA _________________________________ Prof. Roberto

Logo, essa equao no possui forma fatorada em IR.


11.11

FUNO DO 2 GRAU

Uma funo f de IR em IR chamada quadrtica ou do 2 grau, se a cada x IR, associa o elemento (ax+bx+c) IR
(a 0).
f: IR IR
x ax+bx+c (a 0)
11.11.1 GRFICO DA FUNO DO 2 GRAU
O grfico dessa funo uma parbola.
Para construirmos o grfico da funo quadrtica devemos primeiramente encontrar os zeros(razes) da funo em
seguida fazer uma anlise grfica. Devemos considerar 3 possveis casos.
I)

Se >0, ento teremos duas razes distintas.

a)Se a>0 o a parbola fica voltada para cima:


y

Ex.: y = x 3x + 2
2

0
-2

-1

b) Se a<0 o grfico a parbola passa a ter concavidade para baixo:


Ex.: y = x+1
2 y

0
-2

II)

-1

-2
Se =0, ento teremos duas
razes iguais (razes duplas)

a) Se a>0 o a parbola fica voltada para cima:


Ex.: y = x

2 y

0
-2

-1

-2

b) Se a<0 o grfico a parbola passa a ter concavidade para baixo:


2 y

Ex.: y = - x

0
-2

-1

-2

III)

Se < 0, ento a funo no possui razes reais, isto , o grfico da parbola no toca o eixo dos x.

a) Se a>0

Ex.: y = x+1
y
4

__________________________________________________________________________________________________________________________

ATUALIZADO AT MAIO/2012

www.CARREIRAPUBLICA.com.br
x

0
-2

-1

(48) 4141-3220

4141-3222

85

____________

b) Se a<0

_____________________________ MATEMTICA _________________________________ Prof. Roberto

Ex.: y = x1
y
x

0
-2

-1

-2

-4

11.11.2 COORDENADAS DO VRTICE DA PARBOLA


O vrtice formado pela abscissa xv e a ordenada yv:

V
,
2a 4a

xv
yv

Obs.: Para a>0, o ponto V do vrtice ponto de mnimo da funo.


Para a<0 o ponto V do vrtice ponto de mximo da funo.

11.11.3

Imagem da funo quadrtica

A imagem dessa funo obtida projetando-se ortogonalmente os pontos da parbola no eixo y. Desse modo a
ordenada yv ser sempre um dos extremos do intervalo do conjunto imagem, observe:
a>0
Im = [yv, + )

y
f(X2)
f(X1)

xv

a<0
Im =(+ , yv]

yv

X1

X2
0

11.12 Exerccios
1 - Calcule as razes das equaes abaixo:
a) x - 7x + 10 = 0
b) 2x - 20x + 18 = 0

c) - x + 10x - 21 = 0

d) (x + 3)(x - 5) = 0

e) (x - 1)(x - 2) = 0

f) (x + 5) = 0
2 - Calcule a soma das razes da equao 2x - 6x - 5 = 0

( Ag. Adm. - SC)

3 - Qual o valor de p para que a equao 3x - 6x + p = 0 tenha razes reais e iguais? ( ESA)
4 - Calcule a soma e o produto das razes da equao 5x + 3x - 4 = 0 (ESA)
5 - Qual a equao, cujas razes so 7 e - 2 ? ( Exator - SC)

86

__________________________________________________________________________________________________________________________

ATUALIZADO AT MAIO/2012

www.CARREIRAPUBLICA.com.br

(48) 4141-3220

4141-3222

____________

_____________________________ MATEMTICA _________________________________ Prof. Roberto

6 - Calcule m para que a equao 3x - 2x + 2m = 0 tenha uma raiz igual a 2. (ESA)


7 - Calcule o valor de m para que uma das razes da equao mx + (m - 1)x + 2m - 3/4 = 0 seja igual a 1. (AMAN)
8 - Quais os valores de b na equao x - bx + 48 = 0 , para que uma raiz seja o triplo da outra? ( AMAN)
9 - Qual o valor de m para que a equao x - 14x + m = o tenha razes reais e iguais ?
10-Quais os valores de b e c para que a equao ax + bx + c = 0 ( a>0), tenha duas razes reais e simtricas? (
AMAN)
11-Calcule as razes da equao (x - 3)(x + 10) . (x - 5)(x + 2) = 0
(x - 5)(x + 2)
12- Determine a de modo que a diferena entre as razes da equao x + ax + 40 = 0 seja igual a 6.
13- Na equao 4x - 4mx + (m 16) = 0, determine o valor de m de modo que uma das razes seja o inverso da
outra.
14- Determine o valor de p na equao x - 7x + p= 0, de modo que a soma dos inversos das razes seja 7/10.
15- Qual deve ser a relao entre a e b para que a equao ax + (2a + 1)x + (a + b)/a = 0 tenha:
a) Razes iguais

b) Razes reais e diferentes c) Razes no-reais

16- Qual o nmero cujo quadrado excede em 30 unidades o prprio nmero ? R. 6 ou 5


17- Determine trs nmeros inteiros, positivos e consecutivos, tais que o quadrado do menor seja igual diferena
entre os quadrados dos outros dois.
18- A diferena entre o quadrado e o dobro de um nmero 195. Determine esse nmero.
19 - Para pagar as despesas mentais de um condomnio, ficou combinado que todos contribuiriam com a mesma
quantia. Num certo ms, em que as despesas totalizaram R$ 10.800,00, deviado inadimplncia de dois dos
condminos, cada um dos demais foi obrigado a pagar, alm da sua cota normal, um adicional de R$ 32,00. Qual o
nmero de condminos?
20 - Um pai tinha 30 anos quando seu filho nasceu. Se multiplicarmos as idades que possuem hoje, obtm-se um
produto que igual a trs vezes o quadrado da idade do filho. Quais so as suas idades? R.: 15 e 45
21 - Os elefantes de um zoolgico esto de dieta juntos, num perodo de 10 dias devem comer uma quantidade de
cenouras igual ao quadrado da quantidade que um coelho come em 30 dias. Em um dia os elefantes e o coelho
comem juntos1.444 kg de cenoura. Quantos Kg de cenoura os elefantes comem em 1 dia?
22 - Se tirarmos 8 cm ao lado de um quadrado e adicionarmos 12 cm ao outro obtm-se um retngulo de rea 96
2
cm . Qual o valor do lado do quadrado?

23 - Interrogado sobre a sua idade, disse o Paulo: "o quadrado de metade dos anos que j fiz igual ao seu
quntuplo". Quantos anos tem o Paulo?
24 - A soma de dois quadrados perfeitos consecutivos 265. Encontre-os.
25 - O nmero de diagonais D de um polgono com n lados dado pela frmula:

__________________________________________________________________________________________________________________________

ATUALIZADO AT MAIO/2012

www.CARREIRAPUBLICA.com.br

(48) 4141-3220

4141-3222

87

____________

_____________________________ MATEMTICA _________________________________ Prof. Roberto

a) Existir algum polgono que tenha 44 diagonais?


b) Existir um polgono com 30 diagonais?
26 - Assinale a NICA proposio CORRETA. A figura a seguir representa o grfico de uma parbola cujo vrtice o
ponto V. A equao da reta r

a) y = -2x + 2.
b) y = x + 2.
c) y = 2x + 1.
d) y = 2x + 2.
e) y = -2x - 2.
27 - Observe a figura

Nessa figura, est representada a parbola de vrtice V, grfico da funo de segundo grau cuja expresso
2
a) y = (x /5) - 2x
2
b) y = x - 10x
2
c) y = x + 10x
2
d) y = (x /5) - 10x
2
e) y = (x /5) + 10x
28 - Um supermercado fez campanha publicitria para vender o estoque de determinado produto. Suponha que x dias
2
aps o trmino da campanha as vendas dirias foram calculadas segundo a funo y = - x + 10x + 75. Conforme o
grfico abaixo, as vendas se reduziram a zero depois de:

a) 15 dias

88

__________________________________________________________________________________________________________________________

ATUALIZADO AT MAIO/2012

www.CARREIRAPUBLICA.com.br

(48) 4141-3220

4141-3222

____________

_____________________________ MATEMTICA _________________________________ Prof. Roberto

b) 10 dias
c) 25 dias
d) 75 dias
e) 50 dias
2

29 - Sobre o grfico da funo, definida por f(x) = -x + 4x - 5, de IR em IR, a alternativa correta :


a) Todo ponto pertencente ao grfico possui ordenada negativa.
b) O grfico uma parbola com a concavidade voltada para baixo e vrtice V (2, 1).
c) O ponto (0, 5) pertence ao grfico.
d) A parbola tangencia o eixo x.
e) Todo ponto da parbola pertence ao primeiro ou segundo quadrante.
30 - O custo C, em reais, para se produzir n unidades de determinado produto dado por:
2
C = 2510 - 100n + n . Quantas unidades devero ser produzidas para se obter o custo mnimo?
2

31 - A funo real f, de varivel real, dada por f(x)= -x + 12x - 20, tem um valor
a) mximo, igual a 16, para x = 6
b) mnimo, igual a -16, para x = -12
c) mximo, igual a 56, para x = 6
d) mnimo, igual a 72, para x = 12
e) mximo, igual a 240, para x = 20
Resposta: a
32 - Uma bola largada do alto de um edifcio e cai em direo ao solo. Sua altura h em relao ao solo, t segundos
2
aps o lanamento, dada pela expresso h=-25t +625. Aps quantos segundos do lanamento a bola atingir o
solo?
a) 2,5
b) 5
c) 7
d) 10
e) 25
2

33 - A funo L(x) = x + 12x 27 representa o lucro de uma empresa, em milhes de reais, onde x a quantidade
de unidades vendidas. Nesse contexto, considere as seguintes afirmaes:
I. Se vender apenas 2 unidades, a empresa ter lucro.
II. Se vender exatamente 6 unidades, a empresa ter lucro mximo.
III. Se vender 15 unidades, a empresa ter prejuzo.
Est(o) correta(s) apenas:
a) I
b) II
c) III
d) I e I
e) II e III
34 - ( B.B 2007 ) Um grupo de amigos fez, em conjunto, um jogo em determinada loteria, tendo sido premiado com
a importncia de R$ 2.800.000,00 que deveria ser dividida igualmente entre todos eles. No momento da partilha,
constatou-se que 3 deles no haviam pago a parcela correspondente ao jogo, e, dessa forma, no faziam juz ao
quinho do prmio. Com a retirada dos 3 amigos que no pagaram oi jogo , coube a cada um dos restantes mais R$
120.000,00 . Considerando a situao hipottica apresentada, julgue os itens que se seguem.
1 - Se x a quantidade de elementos do grupo de amigos , ento

2.800.000
2.800.000
+ 120.000 =
x3
x

2. - Considerando que, em uma funo da forma f(x) = ax + bx +c . em que a , b e c so constantes bem


determinadas, a equao f(x) = 0 determina a quantidade de elementos do grupo de amigosento correto afirmar
que, para essa funo, o ponto de mnimo atingido quando x =

3
.
2

3 - A quantidade de elementos do grupo de amigos que fizeram juz ao prmio superior a 11.
4 - Cada um dos elementos do grupo de amigos que efetivamente pagou a parcela correspondente ao jogo recebe
uma quantia superior a R$ 250.000,00.
__________________________________________________________________________________________________________________________

ATUALIZADO AT MAIO/2012

www.CARREIRAPUBLICA.com.br

(48) 4141-3220

4141-3222

89

____________

_____________________________ MATEMTICA _________________________________ Prof. Roberto

Gabarito
1a) (5 ;2) b) (9; 1)
c) (3; 7) d) (- 3; 5 )
e) (1; 2) f) (5; 5)

2- (3)

3 (3)

4 (4/5)

5 (x - 5x 14)

6 (- 4)
11 (3; -10)

7 (7/16)
12 (b = 0 e c<0)

8 ( 16)
13 12

9 (49)
14 (10)

10 (b = 0 e c<0)
15 (8a +4a+4b)/a=0
(8a +4a+4b)/a>0
(8a +4a+4b)/a<0

16 (6;10)
21- (1440)

17 93; 4; 5)
22- 12

18 (15; - 13)
23- (20)

26 d
31 a

27 a
32 b

28 a
33 a

19 (27)
24- (11; 12; - 11; 12)
29 a
34 (F; V; F; V)

20 (15;45)
25 Sim/No
30 50

12 Razes
Vamos considerar um carro de corrida com 4m de comprimento e um kart com 2m de comprimento. Para
compararmos as medidas dos carros, basta dividir o comprimento de um deles pelo outro. Assim:

(o tamanho do carro de corrida duas vezes o tamanho do kart).

Podemos afirmar tambm que o kart tem a metade


do comprimento do carro de corrida.
A comparao entre dois nmeros racionais, atravs de uma diviso, chama-se razo.

A razo
de corrida.

pode tambm ser representada por 1:2 e significa que cada metro do kart corresponde a 2m do carro

Denominamos de razo entre dois nmeros a e b (b diferente de zero)


o quociente

ou a:b.

A palavra razo, vem do latim ratio, e significa "diviso". Como no exemplo anterior, so diversas as situaes
em que utilizamos o conceito de razo. Exemplos:

Dos 1200 inscritos num concurso, passaram 240 candidatos.


Razo dos candidatos aprovados nesse concurso:

(de cada 5 candidatos inscritos, 1 foi aprovado).

90

__________________________________________________________________________________________________________________________

ATUALIZADO AT MAIO/2012

www.CARREIRAPUBLICA.com.br

(48) 4141-3220

4141-3222

____________

_____________________________ MATEMTICA _________________________________ Prof. Roberto

Para cada 100 convidados, 75 eram mulheres.


Razo entre o nmero de mulheres e o nmero de convidados:

(de cada 4 convidados, 3 eram mulheres).

Observaes:
1) A razo entre dois nmeros racionais pode ser apresentada de trs formas. Exemplo:
Razo entre 1 e 4:

1:4 ou

ou 0,25.

2) A razo entre dois nmeros racionais pode ser expressa com sinal negativo, desde que seus termos
tenham sinais contrrios. Exemplos:

A razo entre 1 e -8

A razo entre

12.1 Termos de uma razo


Observe a razo:

(l-se "a est para b" ou "a para b").

Na razo a:b ou
Veja o exemplo:

, o nmero a denominado antecedente e o nmero b denominado consequente.

3:5 =
Leitura da razo: 3 est para 5 ou 3 para 5.
12.2 Razes inversas

Considere as razes

__________________________________________________________________________________________________________________________

ATUALIZADO AT MAIO/2012

www.CARREIRAPUBLICA.com.br

(48) 4141-3220

4141-3222

91

____________

_____________________________ MATEMTICA _________________________________ Prof. Roberto

Observe que o produto dessas duas razes igual a 1, ou seja,

Nesse caso, podemos afirmar que

so razes inversas.

Duas razes so inversas entre si quando o produto delas igual a 1.


Exemplo:
so razes inversas, pois

Verifique que nas razes inversas o antecedente de uma o consequente da outra, e vice-versa.

Observaes:
1) Uma razo de antecedente zero no possui inversa.
2) Para determinar a razo inversa de uma razo dada, devemos permutar (trocar) os seus termos.
Exemplo: O inverso de

12.3 Razes equivalentes


Dada uma razo entre dois nmeros, obtemos uma razo equivalente da seguinte maneira:
Multiplicando-se ou dividindo-se os termos de uma razo por um mesmo nmero
racional (diferente de zero), obtemos uma razo equivalente.
Exemplos:

so razes equivalentes.

so razes equivalentes.

12.4 Razes entre grandezas da mesma espcie


O conceito o seguinte:
Denomina-se razo entre grandezas de mesma espcie o quociente entre os nmeros que

92

__________________________________________________________________________________________________________________________

ATUALIZADO AT MAIO/2012

www.CARREIRAPUBLICA.com.br

(48) 4141-3220

4141-3222

____________

_____________________________ MATEMTICA _________________________________ Prof. Roberto

expressam as medidas dessas grandezas numa mesma unidade.


Exemplos:
1) Calcular a razo entre a altura de dois anes, sabendo que o primeiro possui uma altura h1= 1,20m e o
segundo possui uma altura h2= 1,50m. A razo entre as alturas h1 e h2 dada por:

2) Determinar a razo entre as reas das superfcies das quadras de vlei e basquete, sabendo que a quadra de
2
2
vlei possui uma rea de 162m e a de basquete possui uma rea de 240m .

Razo entre as rea da quadra de vlei e basquete:

12.5 Razes entre grandezas de espcies diferentes


O conceito o seguinte:
Para determinar a razo entre duas grandezas de espcies diferentes, determina-se o quociente
entre as medidas dessas grandezas. Essa razo deve ser acompanhada da notao que
relaciona as grandezas envolvidas.
Exemplos:
1) Consumo mdio:
Beatriz foi de So Paulo a Campinas (92Km) no seu carro. Foram gastos nesse percurso 8 litros de combustvel. Qual
a razo entre a distncia e o combustvel consumido? O que significa essa razo? Soluo:

Razo =
Razo =

(l-se "11,5 quilmetros por litro").

Essa razo significa que a cada litro consumido foram percorridos em mdia 11,5 km.

2) Velocidade mdia:
Moacir fez o percurso Rio-So Paulo (450Km) em 5 horas. Qual a razo entre a medida dessas grandezas? O que
significa essa razo?
Soluo:

Razo =
Razo = 90 km/h (l-se "90 quilmetros por hora").
Essa razo significa que a cada hora foram percorridos em mdia 90 km.

__________________________________________________________________________________________________________________________

ATUALIZADO AT MAIO/2012

www.CARREIRAPUBLICA.com.br

(48) 4141-3220

4141-3222

93

____________

_____________________________ MATEMTICA _________________________________ Prof. Roberto

3) Densidade demogrfica:
O estado do Cear no ltimo censo teve uma populao avaliada em 6.701.924 habitantes. Sua rea de 145.694
2
km . Determine a razo entre o nmero de habitantes e a rea desse estado. O que significa essa razo?
Soluo:

Razo =
Razo = 46 hab/km2 (l-se "46 habitantes por quilmetro quadrado").
Essa razo significa que em cada quilmetro quadrado existem em mdia 46 habitantes.

4) Densidade absoluta ou massa especfica:


Um cubo de ferro de 1cm de aresta tem massa igual a 7,8g. Determine a razo entre a massa e o volume desse
corpo. O que significa essa razo?
Soluo:
Volume = 1cm . 1cm . 1cm = 1cm

Razo =
Razo = 7,8 g/cm3 (l-se "7,8 gramas por centmetro cbico").
3

Essa razo significa que 1cm de ferro pesa 7,8g.

13 Propores

13.1 Introduo
Rogerio e Claudinho passeiam com seus cachorros. Rogerio pesa 120kg, e seu co, 40kg. Claudinho, por sua vez,
pesa 48kg, e seu co, 16kg.
Observe a razo entre o peso dos dois rapazes:

Observe, agora, a razo entre o peso dos cachorros:

94

__________________________________________________________________________________________________________________________

ATUALIZADO AT MAIO/2012

www.CARREIRAPUBLICA.com.br

(48) 4141-3220

4141-3222

____________

_____________________________ MATEMTICA _________________________________ Prof. Roberto

Verificamos que as duas razes so iguais. Nesse caso, podemos afirmar que a

igualdade

uma proporo. Assim:


Proporo uma igualdade entre duas razes.

13.2 Elementos de uma proporo


Dados quatro nmeros racionais a, b, c, d, no-nulos, nessa ordem, dizemos que eles formam uma proporo
quando a razo do 1 para o 2 for igual razo do 3 para o 4. Assim:

ou a:b=c:d
(l-se "a est para b assim como c est para d")
Os nmeros a, b, c e d so os termos da proporo, sendo:

b e c os meios da proporo.
a e d os extremos da proporo.

Exemplo:

, temos:
Dada a proporo
Leitura: 3 est para 4 assim como 27 est para 36.
Meios: 4 e 27
Extremos: 3 e 36
13.3 Propriedade fundamental das propores
Observe as seguintes propores:
Produto dos meios = 4.30 = 120
Produto dos extremos = 3.40 = 120
Produto dos meios = 9.20 = 180
Produto dos extremos = 4.45 = 180
Produto dos meios = 8.45 = 360
Produto dos extremos = 5.72 = 360
__________________________________________________________________________________________________________________________

ATUALIZADO AT MAIO/2012

www.CARREIRAPUBLICA.com.br

(48) 4141-3220

4141-3222

95

____________

_____________________________ MATEMTICA _________________________________ Prof. Roberto

De modo geral, temos que:

Da podemos enunciar a propriedade fundamental das propores:


Em toda proporo, o produto dos meios igual ao produto dos extremos.
13.4 Aplicaes da propriedade fundamental
Determinao do termo desconhecido de uma proporo
Exemplos:

Determine o valor de x na proporo:

Soluo:
5 . x = 8 . 15
5 . x = 120

(aplicando a propriedade fundamental)

x = 24
Logo, o valor de x 24.

Determine o valor de x na proporo:

Soluo:
5 . (x-3) = 4 . (2x+1)
5x - 15 = 8x + 4
5x - 8x = 4 + 15
-3x = 19
3x = -19
x=

(aplicando a propriedade fundamental)

Logo, o valor de x

Os nmeros 5, 8, 35 e x formam, nessa ordem, uma proporo. Determine o valor de x.


Soluo:

(aplicando a propriedade fundamental)


5 . x = 8 . 35
5x = 280

96

__________________________________________________________________________________________________________________________

ATUALIZADO AT MAIO/2012

www.CARREIRAPUBLICA.com.br

(48) 4141-3220

4141-3222

____________

_____________________________ MATEMTICA _________________________________ Prof. Roberto

x = 56
Logo, o valor de x 56.

13.5 Resoluo de problemas envolvendo propores


Exemplo:

Numa salina, de cada metro cbico (m ) de gua salgada, so retirados 40 dm de sal. Para obtermos 2 m
de sal, quantos metros cbicos de gua salgada so necessrios?

Soluo:
A quantidade de sal retirada proporcional ao volume de gua salgada.
Indicamos por x a quantidade de gua salgada a ser determinada e armamos a proporo:

Lembre-se que 40dm = 0,04m .

(aplicando a propriedade fundamental)

1 . 2 = 0,04 . x

0,04x = 2

x = 50 m

Logo, so necessrios 50 m de gua salgada.


13.6 Quarta proporcional
Dados trs nmeros racionais a, b e c, no-nulos, denomina-se quarta proporcional desses nmeros um nmero x
tal que:

Exemplo:

Determine a quarta proporcional dos nmeros 8, 12 e 6.


Soluo: Indicamos por x a quarta proporcional e armamos a proporo:

(aplicando a propriedade fundamental)


8 . x = 12 . 6
8 . x = 72
__________________________________________________________________________________________________________________________

ATUALIZADO AT MAIO/2012

www.CARREIRAPUBLICA.com.br

(48) 4141-3220

4141-3222

97

____________

_____________________________ MATEMTICA _________________________________ Prof. Roberto

x = 9
Logo, a quarta proporcional 9.
13.7 Proporo contnua

Considere a seguinte proporo:


Observe que os seus meios so iguais, sendo, por isso, denominada proporo contnua. Assim:
Proporo contnua toda a proporo que apresenta os meios iguais.
De um modo geral, uma proporo contnua pode ser representada por:

13.8 Terceira proporcional


Dados dois nmeros naturais a e b, no-nulos, denomina-se terceira proporcional desses nmeros o nmero
x tal que:

Exemplo:
Determine a terceira proporcional dos nmeros 20 e 10.
Soluo
Indicamos por x a terceira proporcional e armamos a proporo:

(aplicando a propriedade fundamental)


20 . x = 10 . 10
20x = 100

x=5

Logo, a terceira proporcional 5.

13.9 Mdia geomtrica ou mdia proporcional

Dada uma proporo contnua


entre a e c. Exemplo:

98

, o nmero b denominado mdia geomtrica ou mdia proporcional

__________________________________________________________________________________________________________________________

ATUALIZADO AT MAIO/2012

www.CARREIRAPUBLICA.com.br

(48) 4141-3220

4141-3222

____________

_____________________________ MATEMTICA _________________________________ Prof. Roberto

Determine a mdia geomtrica positiva entre 5 e 20.


Soluo:

5 . 20 = b . b

100 = b2

b2 = 100

b=

b = 10

Logo, a mdia geomtrica positiva 10.

13.10 Propriedades das propores


1 propriedade:
Numa proporo, a soma dos dois primeiros termos est para o 2 (ou 1) termo, assim como a
soma dos dois ltimos est para o 4 (ou 3).
Demonstrao
Considere as propores:

Adicionando 1 a cada membro obtemos:

Exemplo:

Determine x e y na proporo
Soluo:

, sabendo que x+y=84.

Assim:

x+y = 84 => x = 84-y =>

x = 84-48 => x=36.

__________________________________________________________________________________________________________________________

ATUALIZADO AT MAIO/2012

www.CARREIRAPUBLICA.com.br

(48) 4141-3220

4141-3222

99

____________

_____________________________ MATEMTICA _________________________________ Prof. Roberto

Logo, x=36 e y=48.

2 propriedade:
Numa proporo, a diferena dos dois primeiros termos est para o 2 (ou 1) termo, assim
como a diferena dos dois ltimos est para o 4 (ou 3).
Demonstrao
Considere as propores:

Subtraindo 1 a cada membro obtemos:

(Mult. os 2 membros por -1)

Exemplo:

Sabendo-se que x-y=18, determine x e y na proporo


Soluo:

Pela 2 propriedade temos que:

x-y = 18 => x=18+y => x = 18+12


Logo, x=30 e y=12.

=> x=30.

3 propriedade:
Numa proporo, a soma dos antecedentes est para a soma dos consequentes, assim como
cada antecedente est para o seu consequente.
Demonstrao
Considere a proporo:

100__________________________________________________________________________________________________________________________
ATUALIZADO AT MAIO/2012

www.CARREIRAPUBLICA.com.br

(48) 4141-3220

4141-3222

____________

_____________________________ MATEMTICA _________________________________ Prof. Roberto

Permutando os meios, temos:

Aplicando a 1 propriedade, obtemos:

Permutando os meios, finalmente obtemos:

4 propriedade:
Numa proporo, a diferena dos antecedentes est para a diferena dos consequentes, assim
como cada antecedente est para o seu consequente.
Demonstrao
Considere a proporo:

Permutando os meios, temos:

Aplicando a 2 propriedade, obtemos:

Permutando os meios, finalmente obtemos:

Exemplo:

Sabendo que a-b = -24, determine a e b na proporo


Soluo:

Pela 4 propriedade, temos que:

__________________________________________________________________________________________________________________________

ATUALIZADO AT MAIO/2012

www.CARREIRAPUBLICA.com.br

(48) 4141-3220

101

4141-3222

____________

_____________________________ MATEMTICA _________________________________ Prof. Roberto

5 propriedade:
Numa proporo, o produto dos antecedentes est para o produto dos consequentes, assim como o
quadrado de cada antecedente est para quadrado do seu consequente.
Demonstrao
Considere a proporo:

Multiplicando os dois membros por

, temos:

Assim:

Observao: a 5 propriedade pode ser estendida para qualquer nmero de razes. Exemplo:

13.11 Proporo mltipla


Denominamos proporo mltipla uma srie de razes iguais. Assim:

uma proporo mltipla.

Dada a srie de razes iguais

, de acordo com a 3 e 4 propriedade, podemos escrever:

102__________________________________________________________________________________________________________________________
ATUALIZADO AT MAIO/2012

www.CARREIRAPUBLICA.com.br

(48) 4141-3220

4141-3222

____________

_____________________________ MATEMTICA _________________________________ Prof. Roberto

14 Grandezas
14.1

Introduo

Entendemos por grandeza tudo aquilo que pode ser medido, contado. As grandezas podem ter suas medidas
aumentadas ou diminudas.
Alguns exemplos de grandeza: o volume, a massa, a superfcie, o comprimento, a capacidade, a velocidade, o tempo,
o custo e a produo.
comum ao nosso dia-a-dia, ituaes em que relacionamos duas ou mais grandezas. Por exemplo:
Em uma corrida de "quilmetros contra o relgio", quanto maior for a velocidade, menor ser o tempo gasto
nessa prova. Aqui as grandezas so a velocidade e o tempo.
Num forno utilizado para a produo de ferro fundido comum, quanto maior for o tempo de uso, maior ser a
produo de ferro. Nesse caso, as grandezas so o tempo e a produo.
14.2 Grandezas diretamente proporcionais
Um forno tem sua produo de ferro fundido de acordo com a tabela a seguir.
Tempo (minutos)

Produo (Kg)

100

10

200

15

300

20

400

Observe que uma grandeza varia de acordo com a outra. Essas grandezas so variveis dependentes. Observe
que:
Quando duplicamos o tempo, a produo tambm duplica.
5 min ----> 100Kg
10 min ----> 200Kg
Quando triplicamos o tempo, a produo tambm triplica.
5 min ----> 100Kg
15 min ----> 300Kg
Assim:
Duas grandezas variveis dependentes so diretamente proporcionais quando a razo entre os
valores da 1 grandeza igual a razo entre os valores correspondentes da 2
Verifique na tabela que a razo entre dois valores de uma grandeza igual a razo entre os dois valores
correspondentes da outra grandeza.

__________________________________________________________________________________________________________________________

ATUALIZADO AT MAIO/2012

www.CARREIRAPUBLICA.com.br

(48) 4141-3220

103

4141-3222

____________

_____________________________ MATEMTICA _________________________________ Prof. Roberto

14.3 Grandezas inversamente proporcionais


Um ciclista faz um treino para a prova de "1000 metros contra o relgio", mantendo em cada volta uma velocidade
constante e obtendo, assim, um tempo correspondente, conforme a tabela a seguir

Velocidade (m/s)

Tempo (s)

200

125

10

100

16

62,5

20

50

Observe que uma grandeza varia de acordo com a outra. Essas grandezas so variveis dependentes. Observe
que:
Quando duplicamos a velocidade, o tempo fica reduzido metade.
5 m/s ----> 200s
10 m/s ----> 100s
Quando quadriplicamos a velocidade, o tempo fica reduzido quarta parte.
5 m/s ----> 200s
20 m/s ----> 50s
Assim:
Duas grandezas variveis dependentes so inversamente proporcionais quando a razo entre
os valores da 1 grandeza igual ao inverso da razo entre os valores correspondentes da 2.
Verifique na tabela que a razo entre dois valores de uma grandeza igual ao inverso da razo entre os dois valores
correspondentes da outra grandeza.

14.4 Exerccios

1.
Dividindo-se 840 em partes diretamente proporcionais a 2, 3, 4, 6, qual a parte
correspondente ao nmero 4?

104__________________________________________________________________________________________________________________________
ATUALIZADO AT MAIO/2012

www.CARREIRAPUBLICA.com.br

(48) 4141-3220

4141-3222

____________

_____________________________ MATEMTICA _________________________________ Prof. Roberto

2.
Dividir 540 em partes proporcionais aos nmeros 1, 2, 3.
3.

Dividir 840 em partes proporcionais aos nmeros 2/3, 1/2 e 5/6

4.
Dividir 15.000 em trs partes , tais que a primeira esteja para a 2 como 3 est para
5 e a segunda para a terceira , como 5 para 7
5.
Um nmero foi dividido em partes diretamente proporcionais a 3 1 ; 0,25 e 2 Nesta
diviso o maior nmero obtido foi 126 qual o nmero inicial?
2
3
6.
Dividindo-se um nmero N em partes inversamente proporcionais aos nmeros 0,4;
3,2 e 6,4 foi encontrado 800 como sendo a parte correspondente ao nmero 3,2 . Qual o valor de
N?
7.

Dividindo-se 1720 em partes proporcionais a 0,2; 1/3 e 2/7, qual a segunda parte?

8.
Na diviso do nmero 840 em partes diretamente proporcionais aos nmeros 4, 5 e
6 qual a parte correspondente ao nmero 5 ?
9.
Divida 372 em
segunda parte ?

5 partes, sendo cada uma metade da anterior, Quanto vale a

10.
Divida 4774 em 5 partes proporcionais de modo que cada uma seja o qudrupulo
da anterior. Qual o valor da segunda parte ?
11.
Divida 4,93 em partes diretamente proporcionais a 0,3 1 e 8/5. Qual o valor da
segunda parte?
12.

Divida 14,3 em 3 partes, sendo cada uma o triplo da anterior. A maior parte vale?

13.

Dividindo 286 em partes inversamente proporcionais a 0,2, 3/4 e 1, a menor parte ?

14.
Dividindo 3720 proporcionalmente a 4, 3, e 5 e ao mesmo tempo a 5, 2 e 1, qual a
maior parte ?
15.
Dividindo 620 em partes proporcionais a 7, 3 e 2 e inversamente a 1, 5 e 3 , qual a
segunda parte?
16.
Dividindo-se 48.400 entre duas funcionrias
respectivamente, qual a gratificao da menos assdua ?

que

faltaram

dias

17.
Dividindo uma quantia em partes inversamente proporcionais a 2,5 e 1/3, a diferena
entre a metade da 3 e o dobro da 1 95. Qual o dobro da segunda parte?
18.
maiores?

Dividi 49.220 proporcionalmente a 0,2 ; 1/3 e 1. Qual a diferena entre as 2 partes

19.
Dividi uma quantia proporcionalmente s idades 22 anos, 31 anos, 42 anos e 53
anos. A diferena entre as duas primeiras R$ 81.900,00. Qual a soma das partes das duas
ltimas idades?

__________________________________________________________________________________________________________________________

ATUALIZADO AT MAIO/2012

www.CARREIRAPUBLICA.com.br

(48) 4141-3220

105

4141-3222

____________

_____________________________ MATEMTICA _________________________________ Prof. Roberto

20.
Divida R$ 123.000,00 inversamente proporcional a 2, 3 e 7 anos . Qual a parte do
mais velho?
21.

Divida R$ 34.750,00 em trs partes diretamente proporcionais a 2, 5 e 1/5 e


inversamente a 3, 4 e 1/2. Qual o valor de menor parte?

22
Dividiu-se R$ 63.270,00 entre trs pessoas. Se cada pessoa recebeu sete vezes o
que recebeu a anterior, qual a parte que tocou terceira pessoa?
23.
Dividindo-se R$ 28,60 em duas partes inversamente proporcionais a 0,6 e 1 e
diretamente a 8 e 4, a segunda parte ser?
24.
Divida uma herana em partes inversamente proporcionais s idades: 20 anos e 12
anos. Se a do mais novo de R$ 243.000,00, qual a do mais velho?
25.
Um nmero foi dividido proporcionalmente a 3, 8 e 6. O triplo do segundo menos o
dobro do terceiro e menos o primeiro de R$ 11.700,00. Qual o valor da soma das 2 primeiras
partes?
26.
Divida R$ 24.600,00 entre trs pessoas, inversamente s suas idades que so 28,
12 e 8 anos. Qual a importncia que toca ao mais novo?
27.
Divida R$ 34.400,00 em partes proporcionais a 1/5, 2/7 e 0,333... Qual o valor da
terceira parte?
28.
Se eu dividir meu ordenado em partes proporcionais a 3, 5 e 9, a terceira parte R$
5.400,00. Qual a segunda parte , se eu dividi-lo em partes proporcionais a 2,5 e 13?
29.
Divida 148,8 em 5 partes de modo que cada uma seja a metade de seguinte. Qual a
ltima parte?
30.
Divida R$ 104.000,00 em 4 partes, de modo que cada uma seja 2/3 da anterior. A
primeira parte ?
31.
Dividi um nmero proporcionalmente a 5, 7, 11 e 4. O triplo do ltimo menos o dobro
do primeiro d R$ 446,00. Qual o segundo?
32.
33.
parte ...

A 13/17 de B. C a metade de B. O total 1232. Qual o valor de A?


Divida R$ 2.280,00 em 3 partes, cada uma sendo 0,666... da seguinte. A terceira

34.
Trs nmeros so proporcionais a 5, 7 e 9. O dobro do ltimo menos a soma dos
dois primeiros 66. Qual o menor deles?

GABARITO
106__________________________________________________________________________________________________________________________
ATUALIZADO AT MAIO/2012

www.CARREIRAPUBLICA.com.br

(48) 4141-3220

4141-3222

____________

_____________________________ MATEMTICA _________________________________ Prof. Roberto

1- 224

2- 90, 180, 270

3- 280, 210, 350

4- 3.000, 5000, 7000

5- 159

6- 7.600

7- 700

8- 280

9- 96

10- 56

11- 1,7

12- 9,9

13- 39

14- 2400

15- 45

16- R$ 17.600,00

17- 76

18- R$ 21.400,00

19- R$ 864.500,00

20- R$ 18.000,00

21- 6.000

22- R$ 54.390,00

23- R$ 6,60

24- R$ 145.800,00

25- 14.300

26- R$ 12.600,00

27- 14.000

28- R$ 2.550,00

29- 4,8

30- 43200

31- 1.561

32- 416

33- 1.080

34- 55

PROBLEMAS DE CONCURSOS
1 - Trs crianas tem, 3 anos, 5 anos e 8 anos, respectivamente. Quer-se distribuir um prmio de R$ 340,00 em
partes diretamente proporcionais s idades. Calcule quanto recebeu a mais velha. (Ag. Adm. - SC)
2 - Joo tem 6 anos mais que Pedro. Jos tem 5 anos menos que Pedro. Antnio tem 9 anos mais que Jos. A
soma das idades 37 anos.
Dividiu-se uma herana de R$ 952.750,00 em partes inversamente proporcionais s idades de cada um. Diante de
tais informaes, podemos afirmar que:
a) Antnio e Joo receberam, juntos, R$ 515.000,00
b) Pedro e Jos receberam, juntos, R$ 629.000,00
c) Jos recebeu R$ 83.250,00 a mais que os outros trs juntos.
d) Joo recebeu R$ 231.750,00 a menos que os outros trs juntos.
(Fiscal de mercadorias em trnsito - SC)
3 - Dividindo-se 728 em partes diretamente proporcionais a 2, 5 e 6, calcule a maior parte. (Exator-SC)
4 - 165 balas foram distribudas entre trs irmos, cujas idades totalizam 33 anos. Sabendo-se que a distribuio foi
diretamente proporcional idade de cada um, que o irmo mais moo recebeu 40 balas e o do meio, 50, calcule as
idades de cada um. ( Banco do Brasil)
5 - Certa herana foi dividida de forma diretamente proporcional s idades dos herdeiros, que tinham 35, 32 e 23
anos. Se o mais velho recebeu R$ 525,00, calcule quanto coube ao mais novo. (Banco do Brasil )
6 - Destinam-se 24 Fiscais do ICM para as cidades A, B e C, com respectivamente, 30, 48 e 66 estabelecimentos
comerciais. Segundo uma distribuio proporcional, calcule o nmero de fiscais do ICM destinados para a cidade B.
(Fiscal do ICM- RS )
7 - Os amigos A, B e C compraram um bilhete de Loteria, sendo que B gastou o dobro de A e C gastou o dobro de B.
Dividindo-se o prmio do bilhete, proporcionalmente ao que cada um gastou, C ganharia R$ 18.150,00. Calcule o
prmio do bilhete. (Fiscal do ICM RS)
8 - A quantia de R$ 477.900 foi repartida entre trs irmos, de modo que o primeiro recebeu o dobro do que coube
ao segundo, e este, o triplo do que recebeu o terceiro. Calcule quanto o primeiro irmo recebeu. (Banco do Brasil )
9 - Deseja-se dividir R$ 3.780,00 entre dois (2) homens, 4 mulheres e 6 crianas, dentro do seguinte critrio: cada
mulher dever receber 50% a mais do que cada criana e cada homem dever receber 60% a mais do que cada
__________________________________________________________________________________________________________________________

ATUALIZADO AT MAIO/2012

www.CARREIRAPUBLICA.com.br

(48) 4141-3220

107

4141-3222

____________

_____________________________ MATEMTICA _________________________________ Prof. Roberto

mulher. A partir de tal posicionamento, calcule as quantias recebidas por cada homem e cada criana,
respectivamente. ( Fiscal de Tributos - SC )
10 - Dividir R$ 7.200,00 depositados em RDB (Recebidos de Depsitos Bancrios) do Banco do Brasil em trs partes
proporcionais a 3, 6 e 7.
11- Cinco pessoas desejam repartir um prmio no valor de R$ 7.765.000,00 em partes proporcionais aos nmeros 1,
2, 3, 4 e 5. Quanto recebeu cada uma?
12- Duas pessoas repartem a importncia de R$ 350,00 proporcionalmente aos nmeros 3 e 4. Quanto recebe cada
uma?
13- A importncia de R$ 36.500 foi dividida entre trs pessoas em partes diretamente proporcionais s suas idades e
inversamente proporcionais, ao mesmo tempo, ao nmero de filhos. Sabendo-se que o primeira pessoa tem 30 anos
e 2 filhos, a Segunda 36 anos e 3 filhos e a terceira 38 anos e que a primeira recebeu R$15.000. Calcule quantos
filhos tem a terceira pessoa.

GABARITO
1) 170 2) c 3) 336 4) 8; 10 e 15 5) 345 6) 8 7) 31.762,50 8)286.740 9) 540 e 225
10) 1.350,00; 2.700,00 e 3.150,00 11) 517.650; 1.035.350 ; 1.553.000; 2.070.650; 2.588.350
12)150 2 200 13) 4

14.5 Regra de trs simples


Regra de trs simples um processo prtico para resolver problemas que envolvam quatro valores dos quais
conhecemos trs deles. Devemos, portanto, determinar um valor a partir dos trs j conhecidos.

Passos utilizados numa regra de trs simples:


1) Construir uma tabela, agrupando as grandezas da mesma espcie em colunas e mantendo na mesma linha
as grandezas de espcies diferentes em correspondncia.
2) Identificar se as grandezas so diretamente ou inversamente proporcionais.
3) Montar a proporo e resolver a equao.
Exemplos:
2

1) Com uma rea de absoro de raios solares de 1,2m , uma lancha com motor movido a energia solar
2
consegue produzir 400 watts por hora de energia. Aumentando-se essa rea para 1,5m , qual ser a energia
produzida?
Soluo: montando a tabela:
2

rea (m )
1,2
1,5

Energia (Wh)
400
x

Identificao do tipo de relao:

108__________________________________________________________________________________________________________________________
ATUALIZADO AT MAIO/2012

www.CARREIRAPUBLICA.com.br

(48) 4141-3220

4141-3222

____________

_____________________________ MATEMTICA _________________________________ Prof. Roberto

Inicialmente colocamos uma seta para baixo na coluna que contm o x (2 coluna).
Observe que: Aumentando a rea de absoro, a energia solar aumenta.
Como as palavras correspondem (aumentando - aumenta), podemos afirmar que as grandezas so diretamente
proporcionais. Assim sendo, colocamos uma outra seta no mesmo sentido (para baixo) na 1 coluna. Montando a
proporo e resolvendo a equao temos:

Logo, a energia produzida ser de 500 watts por hora.

2) Um trem, deslocando-se a uma velocidade mdia de 400Km/h, faz um determinado percurso em 3 horas. Em
quanto tempo faria esse mesmo percurso, se a velocidade utilizada fosse de 480km/h?
Soluo: montando a tabela:
Velocidade (Km/h)
400
480

Tempo (h)
3
x

Identificao do tipo de relao:

Inicialmente colocamos uma seta para baixo na coluna que contm o x (2 coluna).
Observe que: Aumentando a velocidade, o tempo do percurso diminui.
Como as palavras so contrrias (aumentando - diminui), podemos afirmar que as grandezas so inversamente
proporcionais. Assim sendo, colocamos uma outra seta no sentido contrrio (para cima) na 1 coluna. Montando a
proporo e resolvendo a equao temos:

Logo, o tempo desse percurso seria de 2,5 horas ou 2 horas e 30 minutos.

3) Bianca comprou 3 camisetas e pagou R$120,00. Quanto ela pagaria se comprasse 5 camisetas do mesmo
tipo e preo?
__________________________________________________________________________________________________________________________

ATUALIZADO AT MAIO/2012

www.CARREIRAPUBLICA.com.br

(48) 4141-3220

109

4141-3222

____________

_____________________________ MATEMTICA _________________________________ Prof. Roberto

Soluo: montando a tabela:


Camisetas
3
5

Preo (R$)
120
x

Observe que: Aumentando o nmero de camisetas, o preo aumenta.


Como as palavras correspondem (aumentando - aumenta), podemos afirmar que as grandezas so diretamente
proporcionais. Montando a proporo e resolvendo a equao temos:

Logo, a Bianca pagaria R$200,00 pelas 5 camisetas.

4) Uma equipe de operrios, trabalhando 8 horas por dia, realizou determinada obra em 20 dias. Se o nmero de
horas de servio for reduzido para 5 horas, em que prazo essa equipe far o mesmo trabalho?
Soluo: montando a tabela:

Horas por dia


8
5

Prazo para trmino (dias)


20
x

Observe que: Diminuindo o nmero de horas trabalhadas por dia, o prazo para trmino aumenta.
Como as palavras so contrrias (diminuindo - aumenta), podemos afirmar que as grandezas so inversamente
proporcionais. Montando a proporo e resolvendo a equao temos:

14.6 Regra de trs composta


A regra de trs composta utilizada em problemas com mais de duas grandezas, direta ou inversamente
proporcionais.
3

Exemplos:
1) Em 8 horas, 20 caminhes descarregam 160m de areia. Em 5 horas, quantos caminhes
3
sero necessrios para descarregar 125m ?
Soluo: montando a tabela, colocando em cada coluna as grandezas de mesma espcie e, em cada linha, as
grandezas de espcies diferentes que se correspondem:

110__________________________________________________________________________________________________________________________
ATUALIZADO AT MAIO/2012

www.CARREIRAPUBLICA.com.br

(48) 4141-3220

4141-3222

____________

_____________________________ MATEMTICA _________________________________ Prof. Roberto

Horas
8
5

Caminhes
20
x

Volume
160
125

Identificao dos tipos de relao:


Inicialmente colocamos uma seta para baixo na coluna que contm o x (2 coluna).

A seguir, devemos comparar cada grandeza com aquela onde est o x.


Observe que:
Aumentando o nmero de horas de trabalho, podemos diminuir o nmero de caminhes. Portanto a relao
inversamente proporcional (seta para cima na 1 coluna).
Aumentando o volume de areia, devemos aumentar o nmero de caminhes. Portanto a relao diretamente
proporcional (seta para baixo na 3 coluna). Devemos igualar a razo que contm o termo x com o produto das
outras razes de acordo com o sentido das setas.
Montando a proporo e resolvendo a equao temos:

Logo, sero necessrios 25 caminhes.

2) Numa fbrica de brinquedos, 8 homens montam 20 carrinhos em 5 dias. Quantos carrinhos sero montados
por 4 homens em 16 dias?
Soluo: montando a tabela:

Homens
8
4

Carrinhos
20
x

Dias
5
16

Observe que:
Aumentando o nmero de homens, a produo de carrinhos aumenta. Portanto a relao diretamente
proporcional (no precisamos inverter a razo).
Aumentando o nmero de dias, a produo de carrinhos aumenta. Portanto a relao tambm diretamente
proporcional (no precisamos inverter a razo). Devemos igualar a razo que contm o termo x com o produto das
outras razes.
Montando a proporo e resolvendo a equao temos:
__________________________________________________________________________________________________________________________

ATUALIZADO AT MAIO/2012

www.CARREIRAPUBLICA.com.br

(48) 4141-3220

111

4141-3222

____________

_____________________________ MATEMTICA _________________________________ Prof. Roberto

Logo, sero montados 32 carrinhos.

3) Dois pedreiros levam 9 dias para construir um muro com 2m de altura. Trabalhando 3 pedreiros e
aumentando a altura para 4m, qual ser o tempo necessrio para completar esse muro?
Inicialmente colocamos uma seta para baixo na coluna que contm o x. Depois colocam-se flechas
concordantes para as grandezas diretamente proporcionais com a incgnita e discordantes para as inversamente
proporcionais, como mostra a figura abaixo:

Montando a proporo e resolvendo a equao temos:

Logo, para completar o muro sero necessrios 12 dias.

14.7 Exerccios
Agora chegou a sua vez de tentar. Pratique tentando fazer esses exerccios:
1) Trs torneiras enchem uma piscina em 10 horas. Quantas horas levaro 10 torneiras
piscinas? Resposta: 6 horas.

para encher 2

2) Uma equipe composta de 15 homens extrai, em 30 dias, 3,6 toneladas de carvo. Se for aumentada para 20
homens, em quantos dias conseguiro extrair 5,6 toneladas de carvo? Resposta: 35 dias.
3) Vinte operrios, trabalhando 8 horas por dia, gastam 18 dias para construir um muro de 300m. Quanto tempo
levar uma turma de 16 operrios, trabalhando 9 horas por dia, para construir um muro de 225m? Resposta: 15 dias.
4) Um caminhoneiro entrega uma carga em um ms, viajando 8 horas por dia, a uma velocidade mdia de 50 km/h.
Quantas horas por dia ele deveria viajar para entregar essa carga em 20 dias, a uma velocidade mdia de 60 km/h?
Resposta: 10 horas por dia.
5) Com uma certa quantidade de fio, uma fbrica produz 5400m de tecido com 90cm de largura em 50 minutos.
Quantos metros de tecido, com 1 metro e 20 centmetros de largura, seriam produzidos em 25 minutos? Resposta:
2025 metros.
14.8

Exerccios complementares

112__________________________________________________________________________________________________________________________
ATUALIZADO AT MAIO/2012

www.CARREIRAPUBLICA.com.br

(48) 4141-3220

4141-3222

____________

1.
produto?

_____________________________ MATEMTICA _________________________________ Prof. Roberto

Se 4 kg de um produto custou R$ 72,00, Quanto custaro 5,5 kg deste mesmo

2.
Se 100 kg de trigo fornecem 85 kg de farinha que quantidade de farinha se obter
com 150 sacos de 75 kg de trigo cada?
3.
Se 14 pedreiros levam 180 dias para construir uma casa, quanto tempo levaro para
faz-la 10 pedreiros?
4.

Um automvel percorre 240 km em 3 h. Quanto tempo levar para percorrer 400

km?
5.
Quanto tempo levar um automvel para se deslocar de A a B, andando a 80 km / h,
se andando a 60 km/ h leva 2 horas para percorrer a mesma distncia?
6.

Uma roda d 2376 voltas em 9 minutos. Quantas voltas dar em 1 h e 27 min?

7.
Duas rodas dentadas que esto engrenadas uma na outra tm, respectivamente, 12
e 54 dentes. Quantas voltas dar a menor, enquanto a maior d 8?
8.

Se um relgio adianta 18 minutos em 1 dia, quanto adiantar em 6 3/4 horas?

9.
Num internato, 35 alunos gastam R$ 15,40 pelas refeies de 22 dias. Quanto
gastariam 100 alunos pelas refeies de 83 dias no mesmo internato?
10.
Se com 36 kg de fio foram feitos 126 m de tecido de 0,60 m de largura, pergunta-se
quantos metros de 0,72 m se podem tecer com 48 kg do mesmo fio?
11.
Uma adega de vinho abastece 35 homens por um ms, dando a cada um deles 3/5
de litro por dia. Se os homens ficassem reduzidos a 20 e se cada um deles recebesse 3/4 de litro,
quantos dias a adega poderia abastec-los?
12.

Um andarilho percorre certa distncia em 2 dias e 8 h, andando 10 horas por dia.


Aumentando a sua velocidade em 2/5, pergunta-se quantas horas dirias deve andar
para vencer a mesma distncia em 2 dias?
13.
Para alimentar 15 cavalos durante 11 dias so necessrios 2.200 kg de alfafa.
Retirando-se 7 cavalos, em quanto tempo sero consumidos 1.280 kg de alfafa?
14.
Se 10 operrios, trabalhando 8 h dirias, levam 5 1/2 dias para levantar uma parede
de
22 m de comprimento por 0,45 m de espessura, em quanto tempo 16 operrios,
trabalhando 8 h dia levantam outra parede de 18 m de comprimento, 0,30 m de espessura e de
altura duas vezes maior que a primeira?
15.
Um bloco de mrmore de 3 m de comprimento, 1,50 m de largura e 0,60 de altura
pesa 4.350kg. Quanto pesar um outro bloco do mesmo mrmore cujas dimenses so:
comprimento 2,20 m, largura 0,75 m e altura 1,20 m?
GABARITO
1) R$ 99,00

2) 9.562,50 kg

3) 252 dias

4) 5 h

5) 1 h 30min

6) 22.968 voltas

__________________________________________________________________________________________________________________________

ATUALIZADO AT MAIO/2012

www.CARREIRAPUBLICA.com.br

(48) 4141-3220

113

4141-3222

____________

_____________________________ MATEMTICA _________________________________ Prof. Roberto

7) 36 voltas

8) 5 min e 3,75 segundos.

9) R$ 166,00

10) 140 m

11) 42 dias

12) 8 h e 20 minutos

13) 12 dias

14) 3 3/4 dias = 3 d 18 h

15) 3.190 kg

PROBLEMAS DE CONCURSOS
1

- 15 operrios, em 9 dias de trabalho, ganham R$ 10.800,00, trabalhando 8 horas dirias. Quanto ganhariam 23
operrios, em 12 dias, trabalhando 6 horas por dia ?

- Um operrio levou 10 dias, trabalhando 8 horas por dia, para tecer 1.000 m de fazenda. Quantos dias levaria
para tecer 2.000 m de outra fazenda, trabalhando 6 horas dirias, sabendo-se que esta apresenta uma
dificuldade igual aos 3/4 da primeira. ?

- Com 30 operrios, trabalhando 8 horas por dia, certo engenheiro poder concluir uma construo em 45 dias.
Desejando termin-la 15 dias antes, contrata mais 6 operrios. Quantas horas dirias devero todos trabalhar,
para que a obra seja terminada nesse prazo ?

Se 8 lmpadas de certa potncia, permanecendo acesas 13 noites e 3 horas por noite, consomem 78 Kw,
quantos quilowatts consumiro 5 lmpadas de dupla potncia, permanecendo acesas 16 noites e 4 horas por
noite ? R. 160 Kw

- Um livro tem 144 pginas de 25 linhas cada pgina e 66 letras cada linha. Reimprimindo-se esse livro, com os
mesmos caracteres, porem fazendo pginas de 30 linhas cada uma e com 60 letras por linha, quantas pginas
ter o novo livro ?

- 18 operrios, em 24 dias, fizeram 3.240 m de tecido. Quantos dias levaro 20 operrios, para fazerem 2.040 m,
se os primeiros so trs vezes mais ativos que os segundos ?

- Um grupo de 10 trabalhadores pode fazer uma estrada em 96 dias, trabalhando 6 horas por dia. Quantos dias
levaro o mesmo grupo, se trabalharem 8 horas dirias ? ( TTN )

- Duas mquinas imprimem 12.000 folhetos em 18 dias. Em quantos dias 4 mquinas imprimem 16.000 folhetos,
trabalhando o mesmo nmero de horas por dia ?

- Um pintor gasta 10 latas de tinta para pintar um muro de 40 m de comprimento e 2 m de altura. Quantas latas
de tinta gastar para pintar um muro de 24 m de comprimento e 3 m de altura ?

10 - Trabalhando 6 horas por dia, durante 10 dias, 16 pedreiros construram um muro de 140 m de comprimento.
Quantas horas por dias devero trabalhar 24 pedreiros, para construrem 210 m de um muro, de mesma altura,
em 15 dias ?
11 - Um navio possui provises para uma viagem de 15 dias. Devido a um contratempo qualquer, esta sofre um
atraso de 5 dias. De quanto deve ser reduzida a rao de cada tripulante ?
12 - Uma torneira enche um tanque em 3 horas e outra o esvazia em 5 horas. Mantendo-se as duas torneiras
abertas, simultaneamente. Calcule o tempo necessrio para encher o tanque. R.: 7h 30m
13 - Na fabricao de pes, juntou-se 36g de fermento a 80 g de farinha. Para a obteno da massa, a cada poro
de fermento e farinha, acrescentou-se 50% de gua. Quando levada ao forno, a cada 168 Kg de massa, obtevese 140 Kg de pes. Um padeiro usa 400,32 Kg de fermento e:
a) Dever usar 889,6 g de farinha
b) Precisar de 1.290 l de gua
c) Obter 1.935 Kg de massa
* d) Produzir, ao final, 1.612,4 Kg de pes.
( Fiscal de Tributos - SC)
14 - Quantos litros de gua so desperdiados por uma torneira que d um pingo por segundo, em 6 horas, se cada
gota tem 1 ml?

114__________________________________________________________________________________________________________________________
ATUALIZADO AT MAIO/2012

www.CARREIRAPUBLICA.com.br

(48) 4141-3220

4141-3222

____________

_____________________________ MATEMTICA _________________________________ Prof. Roberto

15 - O passo de um homem cerca de 8 dm. Se andar a 3 passos a cada 2 segundos, quanto tempo levar para
percorrer 4,68 Km ? ( Exator - SC)

15 Progresso aritmtica

15.1 Introduo
Chama-se seqncia ou sucesso numrica, a qualquer conjunto ordenado de nmeros reais ou complexos.
Assim, por exemplo, o conjunto ordenado A = ( 3, 5, 7, 9, 11, ... , 35) uma seqncia cujo primeiro termo
3, o segundo termo 5, o terceiro termo 7 e assim sucessivamente.
Uma seqncia pode ser finita ou infinita.
O exemplo dado acima de uma seqncia finita.
J a seqncia P = (0, 2, 4, 6, 8, ... ) infinita.
Uma seqncia numrica pode ser representada genericamente na forma:
(a1, a2, a3, ... , ak, ... , an, ...) onde a1 o primeiro termo, a2 o segundo termo, ... , ak o k-simo termo, ... ,
an o n-simo termo. (Neste caso, k < n).
Por exemplo, na seqncia Y = ( 2, 6, 18, 54, 162, 486, ... ) podemos dizer que
a3 = 18, a5 = 162, etc.
So de particular interesse, as seqncias cujos termos obedecem a uma lei de formao, ou seja possvel
escrever uma relao matemtica entre eles.
Assim, na seqncia Y acima, podemos observar que cada termo a partir do segundo igual ao anterior
multiplicado por 3.
A lei de formao ou seja a expresso matemtica que relaciona entre si os termos da seqncia,
denominada termo geral.
Considere por exemplo a seqncia S cujo termo geral seja dado por an = 3n + 5, onde n um nmero
natural no nulo.
Observe que atribuindo-se valores para n, obteremos o termo an (n - simo termo) correspondente.
Assim por exemplo, para n = 20, teremos
an = 3.20 + 5 = 65, e portanto o vigsimo termo dessa seqncia (a20) igual a 65.
Prosseguindo com esse raciocnio, podemos escrever toda a seqncia S que seria:
S = ( 8, 11, 14, 17, 20, ... ).
Dado o termo geral de uma seqncia, sempre fcil determin-la.
2
Seja por exemplo a seqncia de termo geral an = n + 4n + 10, para n inteiro e positivo.
Nestas condies, podemos concluir que a seqncia poder ser escrita como:
(15, 22, 31, 42, 55, 70, ... ).
Por exemplo:
2
a6 = 70 porque a6 = 6 + 4.6 + 10 = 36 + 24 + 10 = 70.
15.2 Conceito de Progresso Aritmtica
Chama-se Progresso Aritmtica PA toda seqncia numrica cujos termos a partir do segundo, so
iguais ao anterior somado com um valor constante denominado razo.
Exemplos:
A = ( 1, 5, 9, 13, 17, 21, ... ) razo = 4 (PA crescente)
B = ( 3, 12, 21, 30, 39, 48, ... ) razo = 9 (PA crescente)
C = ( 5, 5, 5, 5, 5, 5, 5, ... ) razo = 0 (PA constante)
D = ( 100, 90, 80, 70, 60, 50, ... ) razo = -10 ( PA decrescente)

__________________________________________________________________________________________________________________________

ATUALIZADO AT MAIO/2012

www.CARREIRAPUBLICA.com.br

(48) 4141-3220

115

4141-3222

____________

_____________________________ MATEMTICA _________________________________ Prof. Roberto

15.3 Termo Geral de uma PA


Seja a PA genrica (a1, a2, a3, ... , an, ...) de razo r.
De acordo com a definio podemos escrever:
a2 = a1 + 1.r
a3 = a2 + r = (a1 + r) + r = a1 + 2r
a4 = a3 + r = (a1 + 2r) + r = a1 + 3r
.....................................................
Podemos inferir (deduzir) das igualdades acima que: .............. an = a1 + (n 1) . r
A expresso an = a1 + (n 1) . r denominada termo geral da PA.
Nesta frmula, temos que an o termo de ordem n (n-simo termo) , r a razo e a1 o primeiro termo da
Progresso Aritmtica PA.
Exemplos:
Qual o milsimo nmero mpar positivo?
Temos a PA: ( 1, 3, 5, 7, 9, ... ) onde o primeiro termo a1= 1, a razo r = 2 e queremos calcular o milsimo
termo a1000. Nestas condies, n = 1000 e poderemos escrever:
a1000 = a1 + (1000 - 1).2 = 1 + 999.2 = 1 + 1998 = 1999.
Portanto, 1999 o milsimo nmero mpar.
Qual o nmero de termos da PA: ( 100, 98, 96, ... , 22) ?
Temos a1 = 100, r = 98 -100 = - 2 e an = 22 e desejamos calcular n.
Substituindo na frmula do termo geral, fica: 22 = 100 + (n - 1). (- 2) ;
logo, 22 - 100 = - 2n + 2 e, 22 - 100 - 2 = - 2n de onde conclui-se que - 80 = - 2n ,
de onde vem n = 40.
Portanto, a PA possui 40 termos.
Atravs de um tratamento simples e conveniente da frmula do termo geral de uma PA, podemos generalizala da seguinte forma:
Sendo aj o termo de ordem j (j-simo termo) da PA e ak o termo de ordem k ( k-simo termo) da PA,
poderemos escrever a seguinte frmula genrica:

aj = ak + (j - k).r
Exemplos:
Se numa PA o quinto termo 30 e o vigsimo termo 60, qual a razo?
Temos a5 = 30 e a20 = 60.
Pela frmula anterior, poderemos escrever:
a20 = a5 + (20 - 5) . r e substituindo fica: 60 = 30 + (20 - 5).r ;
60 - 30 = 15r ; logo, r = 2.
Numa PA de razo 5, o vigsimo termo vale 8. Qual o terceiro termo?
Temos r = 5, a20 = 8.
Logo, o termo procurado ser: a3 = a20 + (3 20).5
a3 = 8 17.5 = 8 85 = - 77.

15.4 Propriedades das Progresses Aritmticas


Numa PA, cada termo (a partir do segundo) a mdia aritmtica dos termos vizinhos deste.
Exemplo:
PA : ( m, n, r ) ; portanto, n = (m + r) / 2
Assim, se lhe apresentarem um problema de PA do tipo:
Trs nmeros esto em PA, ... , a forma mais inteligente de resolver o problema considerar que a PA do

116__________________________________________________________________________________________________________________________
ATUALIZADO AT MAIO/2012

www.CARREIRAPUBLICA.com.br

(48) 4141-3220

4141-3222

____________

_____________________________ MATEMTICA _________________________________ Prof. Roberto

tipo:
(x - r, x, x + r), onde r a razo da PA.
Numa PA, a soma dos termos eqidistantes dos extremos constante.
Exemplo:
PA : ( m, n, r, s, t); portanto, m + t = n + s = r + r = 2r
Estas propriedades facilitam sobremaneira a soluo de problemas.
5 - Soma dos n primeiros termos de uma PA
Seja a PA ( a1, a2, a3, ..., an-1, an).
A soma dos n primeiros termos Sn = a1 + a2 + a3 + ... + an-1 + an , pode ser deduzida facilmente, da aplicao
da segunda propriedade acima.
Temos:
Sn = a1 + a2 + a3 + ... + an-1 + an
claro que tambm poderemos escrever a igualdade acima como:
Sn = an + an-1 + ... + a3 + a2 + a1
Somando membro a membro estas duas igualdades, vem:
2. Sn = (a1 + an) + (a2 + an-1) + ... + (an + a1)
Logo, pela segunda propriedade acima, as n parcelas entre parnteses possuem o mesmo valor ( so iguais
soma dos termos extremos a1 + an ) , de onde conclumos inevitavelmente que:
2.Sn = (a1 + an).n , onde n o nmero de termos da PA.
Da ento, vem finalmente que:

Exemplo:
Calcule a soma dos 200 primeiros nmeros mpares positivos.
Temos a PA: ( 1, 3, 5, 7, 9, ... )
Precisamos conhecer o valor de a200 .
Mas, a200 = a1 + (200 - 1).r = 1 + 199.2 = 399
Logo, Sn = [(1 + 399). 200] / 2 = 40.000
Portanto, a soma dos duzentos primeiros nmeros mpares positivos igual a 40000.

15.5 Exerccios resolvidos


1 - Qual o nmero mnimo de termos que se deve somar na P.A. :( 7/5 , 1 , 3/5 , ... ) , a partir do primeiro
termo, para que a soma seja negativa?
a) 9 *
b) 8
c) 7
d)6
e) 5
SOLUO:
Temos: a1 = 7/5 e r = 1 7/5 = 5/5 7/5 = -2/5, ou seja: r = -2/5.
Poderemos escrever ento, para o n-simo termo an:
an = a1 + (n 1).r = 7/5 + (n 1).(-2/5)
an = 7/5 2n/5 + 2/5 = (7/5 + 2/5) 2n/5 = 9/5 2n/5 = (9 2n)/5

__________________________________________________________________________________________________________________________

ATUALIZADO AT MAIO/2012

www.CARREIRAPUBLICA.com.br

(48) 4141-3220

117

4141-3222

____________

_____________________________ MATEMTICA _________________________________ Prof. Roberto

A soma dos n primeiros termos, pela frmula vista anteriormente ser ento:
Sn = (a1 + an). (n/2) = [(7/5) + (9 2n)/5].(n/2) = [(16 2n)/5].(n/2)
2
Sn = (16n 2n ) / 10
Ora, ns queremos que a soma Sn seja negativa; logo, vem:
2
(16n 2n ) / 10 < 0
Como o denominador positivo, para que a frao acima seja negativa, o numerador deve ser negativo.
Logo, deveremos ter:
2
16n 2n < 0
Portanto, n(16 2n ) < 0
Ora, como n o nmero de termos, ele um nmero inteiro e positivo. Portanto, para que o produto acima
seja negativo, deveremos ter:
16 2n < 0, de onde vem 16 < 2n ou 2n > 16 ou n > 8.
Como n um nmero inteiro positivo, deduzimos imediatamente que n = 9.
Portanto, a alternativa correta a letra A.
2

2 - As medidas dos lados de um tringulo so expressas por x + 1, 2x , x - 5 e esto em P.A. , nesta ordem.
O permetro do tringulo vale:
a) 8 b) 12 c) 15 *d) 24 e) 33
SOLUO:
2
Ora, se x + 1, 2x , x 5 formam uma P.A. , podemos escrever:
2
2x (x + 1) = (x 5) 2x
2
2x x 1 + 5 x + 2x = 0
2
3x + 4 x = 0
Multiplicando por (-1) ambos os membros da igualdade acima, fica:
2
x 3x 4 = 0
Resolvendo a equao do segundo grau acima encontraremos x = 4 ou x = - 1.
Assim, teremos:
2
x = 4: os termos da P.A . sero: x+1, 2x, x 5 ou substituindo o valor de x encontrado: 5, 8, 11, que so as
medidas dos lados do tringulo. Portanto, o permetro do tringulo (soma das medidas dos lados) ser igual a
5+8+11 = 24.
O valor negativo de x no serve ao problema, j que levaria a valores negativos para os lados do tringulo, o
que uma impossibilidade matemtica, pois as medidas dos lados de um tringulo so necessariamente
positivas. Portanto, a alternativa correta a letra D.
3 - UFBA - Um relgio que bate de hora em hora o nmero de vezes correspondente a cada hora, bater , de
zero s 12 horas x vezes. Calcule o dobro da tera parte de x.
Resp: 60
SOLUO:
Teremos que:
0 hora o relgio bater 12 vezes. (Voc no acha que bateria 0 vezes, no ?).
1 hora o relgio bater 1 vez
2 horas o relgio bater 2 vezes
3 horas o relgio bater 3 vezes
....................................................
....................................................
12 horas o relgio bater 12 vezes.
Logo, teremos a seguinte seqncia:
(12, 1, 2, 3, 4, 5, ... , 12)
A partir do segundo termo da seqncia acima, temos uma PA de 12 termos, cujo primeiro termo igual a 1,
a razo 1 e o ltimo termo 12.

118__________________________________________________________________________________________________________________________
ATUALIZADO AT MAIO/2012

www.CARREIRAPUBLICA.com.br

(48) 4141-3220

4141-3222

____________

_____________________________ MATEMTICA _________________________________ Prof. Roberto

Portanto, a soma dos termos desta PA ser:


S = (1 + 12).(12/2) = 13.6 = 78
A soma procurada ser igual ao resultado anterior (a PA em vermelho acima) mais as 12 batidas da zero
hora. Logo, o nmero x ser igual a x = 78 + 12 = 90.
Logo, o dobro da tera parte de x ser: 2. (90/3) = 2.30 = 60, que a resposta do problema proposto.
4 - UFBA - Numa progresso aritmtica, o primeiro termo 1 e a soma do n-simo termo com o nmero de
termos 2. Calcule a razo dessa progresso.
Resp: r = -1
SOLUO:
Temos: a1 = 1 e an + n = 2, onde an o n-simo termo.
Fazendo n = 2, vem: a2 + 2 = 2, de onde vem imediatamente que a2 = 0.
Da, r = a2 a1 = 0 1 = -1, que a resposta procurada.
5 - A soma dos mltiplos positivos de 8 formados por 3 algarismos :
a) 64376 b) 12846 c) 21286 d) 112 *e) 61376
SOLUO:
Nmeros com 3 algarismos: de 100 a 999.
Primeiro mltiplo de 8 maior do que 100 = 104 (que igual a 8x13)
Maior mltiplo de 8 menor do que 999 = 992 (que igual a 8x124)
Temos ento a PA: (104, 112, 120, 128, 136, ... , 992).
Da frmula do termo geral an = a1 + (n 1) . r poderemos escrever:
992 = 104 + (n 1).8, j que a razo da PA 8.
Da vem: n = 112
Aplicando a frmula da soma dos n primeiros termos de uma PA, teremos finalmente:
Sn = S112 = (104 + 992).(112/2) = 61376
A alternativa correta portanto, a letra E.
6 Determinar o centsimo termo da progresso aritmtica na qual a soma do terceiro termo com o stimo
igual a 30 e a soma do quarto termo com o nono igual a 60.
Resp: 965
SOLUO:
Podemos escrever:
a3 + a7 = 30
a4 + a9 = 60
Usando a frmula do termo geral, poderemos escrever:
a1 + 2r + a1 + 6r = 30 ou 2.a1 + 8r = 30
a1 + 3r + a1 + 8r = 60 ou 2.a1 + 11r = 60
Subtraindo membro a membro as duas expresses em negrito, vem:
3r = 30 , de onde conclumos que a razo igual a r = 10.
Substituindo numa das equaes em negrito acima, vem:
2.a1 + 8.10 = 30, de onde tiramos a1 = - 25.
Logo, o centsimo termo ser:
a100 = a1 + 99r = - 25 + 99.10 = 965

15.6 Exerccios propostos


1. (MACK-SP) O trigsimo primeiro termo de uma progresso aritmtica de primeiro termo 2 e razo 3 :
__________________________________________________________________________________________________________________________

ATUALIZADO AT MAIO/2012

www.CARREIRAPUBLICA.com.br

(48) 4141-3220

119

4141-3222

____________

a.
b.
c.
d.
e.

_____________________________ MATEMTICA _________________________________ Prof. Roberto

63
65
92
95
98

2. (FEI-SP) A razo de uma PA de 10 termos, onde o primeiro termo 42 e o ltimo 12, vale:
a.
b.
c.
d.
e.

-5
-9
-6
-7
0

3. O termo geral de uma PA dado por an = 2n 1. Ento o terceiro termo da PA vale:


a.
b.
c.
d.
e.

2
3
5
6
4

4. (PUC PR) Calculando o nmero de termos de uma PA, onde o primeiro termo 0,5 , o ltimo termo 45,5 e a
razo 1,5, obtm-se:
a.
b.
c.
d.
e.

45
38
43
31
57

5. (FEI-SP) O 10 termo da PA (a, 3a/2, ...) igual a :


a.
b.
c.
d.
e.

11a/2
9a/2
7a/2
13a/2
15a/2

6. (UFPA) Numa progresso aritmtica, temos a7 = 5 e a15 = 61. Ento, a razo pertence ao intervalo:
a.
b.
c.
d.
e.

[8,10]
[6,8[
[4,6[
[2,4[
[0,2[

7. (MACK-SP) O produto das razes da equao x + 2x 3 = 0 a razo de uma PA de primeiro termo 7. O 100
termo dessa PA :
a.
b.
c.
d.
e.

-200
-304
-290
-205
-191

8. (UFRS) O nmero de mltiplos de 7 entre 50 e 1206 :


a. 53
b. 87
c. 100

120__________________________________________________________________________________________________________________________
ATUALIZADO AT MAIO/2012

www.CARREIRAPUBLICA.com.br

(48) 4141-3220

4141-3222

____________

_____________________________ MATEMTICA _________________________________ Prof. Roberto

d. 165
e. 203
9. A razo de uma PA, na qual a3 + a5 = 20 e a4 + a7= 29, vale:
a.
b.
c.
d.
e.

3
5
7
9
11

10. (FAAT) A quantidade de nmeros compreendidos entre 1 e 5000, que so divisveis por 3 e 7, :
a.
b.
c.
d.
e.

138
238
137
247
157

11. (FGV-SP) A soma do 4 e 8 termos de PA 20; o 31 termo o dobro do 16 termo. Determine a PA:
a.
b.
c.
d.
e.

(-5, -2, 1, ...)


(5, 6, 7, ...)
(0, 2, 4, ...)
(0, 3, 6, 9, ...)
(1, 3, 5, ...)

12. (PUC-PR) Se em uma PA de 7 termos, de razo K, retirarmos o segundo, terceiro, quinto e sexto termos, a
sucesso restante uma PA de razo:
a.
b.
c.
d.
e.

k
2k
k/2
3k
5k

13. O nmero de termos n de uma PA finita, na qual o primeiro termo 1, o ltimo 17 e a razo r = n 1, vale:
a.
b.
c.
d.
e.

4
5
7
8
12

14. Numa PA de n termos e razo r, temos a1= -2/15, an = 2/3 e r . n = 1. Ento r e n valem, respectivamente:
a.
b.
c.
d.
e.

1/5 e 5
1/3 e 3
1/6 e 6
1/7 e 7
1/9 e 9

15. A soma do 2 e do 4 termos de uma PA 15 e a soma do 5 e 6 termos 25. Ento o 1 termo e a razo valem,
respectivamente:
a.
b.
c.
d.
e.

7/3 e 3
7/4 e 4
7/2 e 2
7/5 e 5
7/6 e 6

__________________________________________________________________________________________________________________________

ATUALIZADO AT MAIO/2012

www.CARREIRAPUBLICA.com.br

(48) 4141-3220

121

4141-3222

____________

_____________________________ MATEMTICA _________________________________ Prof. Roberto

16. (MACK-SP) O n-simo termo da progresso aritmtica 1,87; 3,14; 4,41; ... :
a.
b.
c.
d.
e.

1,27n + 0,6
1,27n + 0,6
1,27 + 0,6 n
1,27 + 0,6
2
0,6n + 1,27

17. Interpolando-se 6 meios aritmticos entre 100 e 184, a razo encontrada vale:
a.
b.
c.
d.
e.

11
12
15
17
19

18. ( POLI ) Inscrevendo-se nove meios aritmticos entre 15 e 45, o sexto termo da PA ser igual :
a.
b.
c.
d.
e.

18
24
36
27
30

19. A quantidade de meios aritmticos que se pode inserir ente 15 e 30, tal que a razo tenha valor 3, :
a.
b.
c.
d.
e.

3
2
4
5
9

20. ( UFPI ) A soma dos nmeros pares de 2 a 400 igual :


a.
b.
c.
d.
e.

7432
8200
40200
80200
20400

21. Em uma PA, a soma dos termos 70, o primeiro termo 10 e a razo 5. O nmero de termos :
a.
b.
c.
d.
e.

10
8
4
12
16

22. ( FATEC - SP ) Se o termo geral de uma PA an = 5n - 13, com n


termos :
a.
b.
c.
d.
e.

IN , ento a soma de seus 50 primeiros

5850
5725
5650
5225
5150
2

23. ( PUC ) A soma dos n primeiros termos de uma PA n + 2n. O 10 termo dessa PA vale:

122__________________________________________________________________________________________________________________________
ATUALIZADO AT MAIO/2012

www.CARREIRAPUBLICA.com.br

(48) 4141-3220

4141-3222

____________

a.
b.
c.
d.
e.

_____________________________ MATEMTICA _________________________________ Prof. Roberto

17
18
19
20
21

24. A soma dos termos de uma PA, cujo primeiro termo 4, o ltimo termo 46 e a razo igual ao nmero de
termos :
a.
b.
c.
d.
e.

50
100
175
150
195

25. ( CEFET - PR ) Inserindo-se K meios aritmticos entre 1 e K , obtm - se uma progresso aritmtica de razo:
a.
b.
c.
d.
e.

1
k
k-1
k+1
2
k

26. O nmero de termos que devemos tomar na PA ( -7, -3, ...) a fim de que a soma valha 3150 :
a.
b.
c.
d.
e.

38
39
40
41
42

27. ( PUC - RS ) Um teatro tm 18 poltronas na primeira fila, 24 na segunda, 30 na terceira e assim na mesma
seqncia , at a vigsima fila que a ltima .O nmero de poltronas desse teatro :
a.
b.
c.
d.
e.

92
150
1500
132
1320

28. ( FATEC ) A soma de todos os nmeros naturais, no nulos, no maiores que 600 e no mltiplos de 5,:
a.
b.
c.
d.
e.

180300
141770
144000
136415
147125

29. ( FGV - SP ) Sabendo que a soma do segundo e do quarto termos de uma progresso aritmtica 40 e que a
razo do primeiro termo , a soma dos dez primeiros temos ser:
a.
b.
c.
d.
e.

350
270
400
215
530

__________________________________________________________________________________________________________________________

ATUALIZADO AT MAIO/2012

www.CARREIRAPUBLICA.com.br

(48) 4141-3220

123

4141-3222

____________

_____________________________ MATEMTICA _________________________________ Prof. Roberto

30. ( MACK - SP) Se soma dos 10 primeiros termos de uma progresso aritmtica 50 e a soma dos 20 primeiros
termos 50, ento a soma dos 30 primeiros termos :
a.
b.
c.
d.
e.

0
50
150
25
100

Numa estrada que liga a entrada de uma fazenda at a sua sede existem duas palmeiras, uma a 8 metros da entrada
e outra a 260 metros. O proprietrio deseja plantar, entre elas, outras cinco palmeiras, com a mesma distncia entre
elas.
A distncia, em metros, entre as palmeiras, de:
a) 54
b) 42
c) 50,4
d) 36
e) n.d.a
31. Uma progresso aritmtica de n termos tem razo igual a 3. Se retirarmos os termos de ordem mpar, os de
ordem par formaro uma progresso
a) aritmtica de razo 2
b) aritmtica de razo 6
c) aritmtica de razo 9
d) geomtrica de razo 3
e) geomtrica de razo 6
Resposta: b
32. Para todo n natural no nulo, sejam as seqncias
(3, 5, 7, 9, ..., an, ...)
(3, 6, 9, 12, ..., bn, ...)
(c1, c2, c3, ..., cn, ...)
com cn = an + bn.
Nessas condies, c20 , igual a
a) 25
b) 37
c) 101
d) 119
e) 149
33. Se as medidas dos ngulos internos de um tringulo esto em progresso aritmtica e a medida do maior ngulo
o qudruplo do menor, ento, a diferena entre o maior e o menor ngulo, em graus, :
a) 36
b) 96
c) 60
d) 24
e)72
34. Um veculo parte de uma cidade A em direo a uma cidade B, distante 500km. Na 1 hora do trajeto ele percorre
20km, na 2 hora 22,5km, na 3 hora 25km e assim sucessivamente. Ao completar a 12 hora do percurso, a
distncia esse veculo estar de B?
a) 95 km
b) 115 km
c) 125 km
d) 135 km
e) 155 km

124__________________________________________________________________________________________________________________________
ATUALIZADO AT MAIO/2012

www.CARREIRAPUBLICA.com.br

(48) 4141-3220

4141-3222

____________

_____________________________ MATEMTICA _________________________________ Prof. Roberto

35. Num programa de condicionamento fsico um atleta corre sempre 300 metros a mais do que correu no dia
anterior. Sabe-se que no segundo dia ele correu um quilmetro. Ento, no dcimo dia, ele correr em metros:
a) 3700.
b) 3100
c) 3400
d) 4000
e) 2800
36. Uma criana anmica pesava 8,3 kg. Iniciou um tratamento mdico que fez com que engordasse 150 g por
semana durante 4 meses. Quanto pesava ao trmino da 15 semana de tratamento?
a) 22,50 kg
b) 15 kg
c) 10,7 kg
d) 10,55 kg
e) 10,46 kg
37. Sobre Progresso Aritmtica, propriedades e generalidades. Analise as afirmaes a seguir.
1. Existem 81 mltiplos de 11 entre 100 e 1000.
2. Sabendo que 1, (3 + x) e (17 - 4x) so termos consecutivos de uma P.A., o valor de x 2.
3. O quarto termo da P.A.(a - b, 5a - 2b, ...) a4 = 13a - 4b.
4. Dada a P.A. (82, 76, 70, ...), o nmero 22 ocupa a 11a posio.
Marque a alternativa correta:

38. Eddie Sortudo no deseja contar com a sorte e espera ganhar um pouco de tempo, acreditando que a munio do
inimigo acabe. Suponha ento que, a partir do primeiro nmero falado por Eddie, ele dir, cada um dos demais,
exatamente 3 segundos aps ter falado o anterior, at que chegue ao nmero determinado pelo seu comandante.
Assim, com sua estratgia, Eddie conseguir ganhar um tempo, em segundos, igual a:

a) 177
b) 188
c) 237
d) 240
Resposta: c
39. Uma criana anmica pesava 8,3 kg. Iniciou um tratamento mdico que fez com que engordasse 150 g por
semana durante 4 meses. Quanto pesava ao trmino da 15 semana de tratamento?
__________________________________________________________________________________________________________________________

ATUALIZADO AT MAIO/2012

www.CARREIRAPUBLICA.com.br

(48) 4141-3220

125

4141-3222

____________

_____________________________ MATEMTICA _________________________________ Prof. Roberto

a) 22,50 kg
b) 15 kg
c) 10,7 kg
d) 10,55 kg
e) 10,46 kg
Resposta: d
40. Um agricultor estava perdendo a sua plantao, em virtude da ao de uma praga. Ao consultar um especialista,
foi orientado para que pulverizasse, uma vez ao dia, uma determinada quantidade de um certo produto, todos os dias,
da seguinte maneira:
primeiro dia: 1,0 litro;
segundo dia: 1,2 litros;
terceiro dia: 1,4 litros; ... e assim sucessivamente.
Sabendo-se que o total de produto pulverizado foi de 63 litros, o nmero de dias de durao deste tratamento nesta
plantao foi de:
a) 21
b) 22
c) 25
d) 27
e) 30
Gabarito
1 (c)
9(a)
17(b)
25(d)
33(e)
16

2(c)
10(b)
18(e)
26(e)
34(a)

3(c)
11(c)
19(c)
27(c)
35(c)

4(d)
12(d)
20(c)
28(c)
36(d)

5(a)
13(b)
21(c)
29(a)
37(b)

6(b)
14(a)
22(b)
30(b)
38(c)

7(c)
15(c)
23(e)
31(b)
39(d)

8(d)
16(b)
24(c)
32(c)
40(a)

LOGARITMOS E EXPONENCIAIS

16.1 INTRODUO
O conceito de logaritmo foi introduzido pelo matemtico escocs John Napier (1550-1617) e aperfeioado pelo ingls
Henry Briggs (1561-1630). A descoberta dos logaritmos deveu-se sobretudo grande necessidade de simplificar os
clculos excessivamente trabalhosos para a poca, principalmente na rea da astronomia, entre outras. Atravs dos
logaritmos, pode-se transformar as operaes de multiplicao em soma, de diviso em subtrao, entre outras
transformaes possveis, facilitando os clculos. Na verdade, a idia de logaritmo muito simples, e pode-se dizer
que o nome logaritmo uma nova denominao para expoente, conforme veremos a seguir.
2

Assim, por exemplo, como sabemos que 4 = 16 , onde 4 a base, 2 o expoente e 16 a potncia, na linguagem dos
logaritmos, diremos que 2 o logaritmo de 16 na base 4. Simples, no ?
Nestas condies, escrevemos simbolicamente: log416 = 2.
Outros exemplos:
2
15 = 225, logo: log15225 = 2
4
5 = 625, logo: log5625 = 4

16.2

6 = 216, logo: log6216 = 3


0
7 = 1, logo: log71 = 0

DEFINIO

Dados os nmeros reais b (positivo e diferente de 1), N (positivo) e x , que satisfaam a relao b = N, dizemos que x
o logaritmo de N na base b. Isto expresso simbolicamente da seguinte forma: logbN = x. Neste caso, dizemos que
b a base do sistema de logaritmos, N o logaritmando ou antilogaritmo e x o logaritmo.
Exemplos:
3
a) log28 = 3 porque 2 = 8.
1
d) log55 = 1 porque 5 = 5.

b) log41 = 0 porque 4 = 1. c) log39 = 2 porque 3 = 9.

126__________________________________________________________________________________________________________________________
ATUALIZADO AT MAIO/2012

www.CARREIRAPUBLICA.com.br

(48) 4141-3220

4141-3222

____________

_____________________________ MATEMTICA _________________________________ Prof. Roberto

Notas:
1 - quando a base do sistema de logaritmos igual a 10 , usamos a expresso logaritmo decimal e na representao
simblica escrevemos somente logN ao invs de log10N. Assim que quando escrevemos logN = x , devemos
x
concluir pelo que foi exposto, que 10 = N.
Existe tambm um sistema de logaritmos chamado neperiano (em homenagem a John Napier - matemtico escocs
do sculo XVI, inventor dos logaritmos), cuja base o nmero irracional
e = 2,7183... e indicamos este logaritmo pelo smbolo ln. Assim,
logeM = ln M. Este sistema de logaritmos, tambm conhecido como sistema de logaritmos naturais, tem grande
aplicao no estudo de diversos fenmenos da natureza.
Exemplos:
2
a) log100 = 2 porque 10 = 100.
0,3010
c) log2 = 0,3010 porque 10
= 2.
1
e) ln e = 1 porque e = e = 2,7183...

b) log1000 = 3 porque 10 = 1000.


0,4771
d) log3 = 0,4771 porque 10
= 3.
f) ln 7 = loge7

2 - Os logaritmos decimais (base 10) normalmente so nmeros decimais onde a parte inteira denominada
caracterstica e a parte decimal denominada mantissa .
Assim por exemplo, sendo log20 = 1,3010, 1 a caracterstica e 0,3010 a mantissa.
As mantissas dos logaritmos decimais so tabeladas.
Consultando a tbua de logaritmo (qualquer livro de Matemtica traz) , podemos escrever por exemplo que log45 =
1,6532. As tbuas de logaritmos decimais foram desenvolvidas por Henry Briggs, matemtico ingls do sculo XVI.
Observe que do fato de termos log45 = 1,6532 , podemos concluir pela definio de logaritmo que
1,6532
10
= 45.
3) Da definio de logaritmo, infere-se (conclui-se) que somente os nmeros reais positivos possuem logaritmo.
Assim, no tm sentido as expresses log3(-9) , log20 , etc.
4) fcil demonstrar as seguintes propriedades imediatas dos logaritmos, todas decorrentes da definio:
P1) O logaritmo da unidade em qualquer base nulo, ou seja:
0
logb1 = 0 porque b = 1.
1

P2) O logaritmo da base sempre igual a 1, ou seja: logbb = 1 , porque b = b.


k

P3) logbb = k , porque b = b .


P4) Se logbM = logbN ento podemos concluir que M = N. Esta propriedade muito utilizada na soluo de
exerccios envolvendo equaes onde aparecem logaritmos (equaes logartmicas).
P5) b
16.3

logbM

= M ou seja: b elevado ao logaritmo de M na base b igual a M.

PROPRIEDADES OPERATRIAS DOS LOGARITMOS

P1 - LOGARITMO DE UM PRODUTO
O logaritmo de um produto igual a soma dos logaritmos dos fatores, ou seja:
logb(M.N) = logbM + logbN
Exemplo: log20 =log(2.10) = log2 + log10 = 0,3010 + 1 = 1,3010. Observe que como a base no foi especificada,
sabemos que ela igual a 10.
P2 - LOGARITMO DE UM QUOCIENTE

__________________________________________________________________________________________________________________________

ATUALIZADO AT MAIO/2012

www.CARREIRAPUBLICA.com.br

(48) 4141-3220

127

4141-3222

____________

_____________________________ MATEMTICA _________________________________ Prof. Roberto

O logaritmo de uma frao ordinria igual a diferena entre os logaritmos do numerador da frao e do
denominador, ou seja:
logb(M/N) = logbM - logbN
Exemplo: log0,02 = log(2/100) = log2 - log100 = 0,3010 - 2,0000 = -1,6990. Do exposto anteriormente, podemos
-1,6990
concluir que, sendo log0,02 = -1,6990 ento 10
= 0,02.
Da mesma forma podemos exemplificar:
log5 = log(10/2) = log10 - log2 = 1 - 0,3010 = 0,6990.
Observao: a no indicao da base, subtende-se logaritmos decimal (base 10).
Nota: Chamamos de cologaritmo de um nmero positivo N numa base b, ao logaritmo do inverso multiplicativo de N,
tambm na base b. Ou seja:
cologbN = logb(1/N) = logb1 - logbN = 0 - logbN = - logbN.
(menos log de N na base b).
Exemplo: colog10 = -log10 = -1.
P3 - LOGARITMO DE UMA POTNCIA
k

Temos a seguinte frmula, facilmente demonstrvel: logbM = k.logbM.


6
Exemplo: log525 = 6.log525 = 6.2 = 12.
P4 - MUDANA DE BASE
s vezes, para a soluo de problemas, temos necessidade de mudar a base de um sistema de logaritmos, ou seja,
conhecemos o logaritmo de N na base b e desejamos obter o logaritmo de N numa base a . Esta mudana de base,
muito importante na soluo de exerccios, poder ser feita de acordo com a frmula a seguir, cuja demonstrao no
apresenta dificuldades, aplicando-se os conhecimentos aqui expostos.

Exemplos:
a) log416 = log216 / log24 (2 = 4:2) b) log864 = log264 / log28 (2 = 6:3)
1,5
c) log25125 = log5125 / log525 = 3 / 2 = 1,5. Temos ento que 25 = 125.
Notas:
1 - na resoluo de problemas, sempre muito mais conveniente mudar um log de uma base maior para uma base
menor, pois isto simplifica os clculos.
2 - Duas conseqncias importantes da frmula de mudana de base so as seguintes:
a) logbN = logN / logb (usando a base comum 10, que no precisa ser indicada).
b) logba . logab = 1
Exemplos:
a) log37 . log73 = 1
b) log23 = log3 / log2 = 0,4771 / 0,3010 = 1,5850
16.4 A FUNO LOGARTIMICA E A FUNO EXPONENCIAL

Considere a funo y = a , denominada funo exponencial, onde a base a um nmero positivo e diferente de 1,
definida para todo x real.
x

Observe que nestas condies, a um nmero positivo, para todo x R, onde R o conjunto dos nmeros reais.
*

Denotando o conjunto dos nmeros reais positivos por R+ , poderemos escrever a funo exponencial como segue:
*
x
f: R
R+ ; y = a , 0 < a 1

128__________________________________________________________________________________________________________________________
ATUALIZADO AT MAIO/2012

www.CARREIRAPUBLICA.com.br

(48) 4141-3220

4141-3222

____________

_____________________________ MATEMTICA _________________________________ Prof. Roberto

Esta funo bijetora, pois:


a) injetora, ou seja: elementos distintos possuem imagens distintas.
b) sobrejetora, pois o conjunto imagem coincide com o seu contradomnio.
Assim sendo, a funo exponencial BIJETORA e, portanto, uma funo inversvel, OU SEJA, admite uma funo
inversa.
x

Vamos determinar a funo inversa da funo y = a , onde 0 < a 1.


Permutando x por y, vem:
y
x = a y = logax
Portanto, a funo logartmica ento:
*
f: R+ R ; y = logax , 0 < a 1.
x

Mostramos a seguir, os grficos das funes exponencial ( y = a ) e logartmica


( y = logax ), para os casos a > 1 e 0 < a 1. Observe que, sendo as funes, inversas, os seus grficos so curvas
simtricas em relao bissetriz do primeiro e terceiro quadrantes, ou seja, simtricos em relao reta y = x.

Da simples observao dos grficos acima, podemos concluir que:


1 - para a > 1, as funes exponencial e logartmica so CRESCENTES.
2 - para 0 < a 1, elas so DECRESCENTES.
*
3 - o domnio da funo y = logax o conjunto R+ .
4 - o conjunto imagem da funo y = logax o conjunto R dos nmeros reais.
x
5 - o domnio da funo y = a o conjunto R dos nmeros reais.
x
*
6 - o conjunto imagem da funo y = a o conjunto R+ .
7 - observe que o domnio da funo exponencial igual ao conjunto imagem da funo logartmica e que o domnio
da funo logartmica igual ao conjunto imagem da funo exponencial. Isto ocorre porque as funes so inversas
entre si.
16.5 EXERCCIOS RESOLVIDOS
2

1 - Se S a soma das razes da equao log x - logx - 2 = 0 , ento calcule o valor de 1073 - 10S.
SOLUO:
Faamos logx = y; vem:
2
y -y-2=0
Resolvendo a equao do segundo grau acima, encontramos: y = 2 ou y = -1.
Portanto, logx = 2 OU logx = -1
Como a base igual a 10, teremos:
2
x = 10 = 100
log10x = 2
-1
log10x = -1 x = 10 = 1/10
__________________________________________________________________________________________________________________________

ATUALIZADO AT MAIO/2012

www.CARREIRAPUBLICA.com.br

(48) 4141-3220

129

4141-3222

____________

_____________________________ MATEMTICA _________________________________ Prof. Roberto

As razes procuradas so, ento, 100 e 1/10.


Conforme enunciado do problema, teremos:
S = 100 + 1/10 = 1000/10 + 1/10 = 1001/10
Logo, o valor de 1073 - 10S ser:
1073 - 10(1001/10) = 1073 - 1001 = 72

Resp: 72

2 - Calcule o valor de y = 6 onde x = log32 . log63 .


SOLUO:
Substituindo o valor de x, vem:
log 2 . log 3
log 3 log 2
log 2
y=6 3
6 = (6 6 ) 3 = 3 3 = 2
log M
Na soluo acima, empregamos a propriedade b b = M , vista anteriormente.
Resp: 2
3 - UEFS - Sendo log 2 = 0,301, o nmero de algarismos de 5
a) 13 b) 14 c) 19 d) 20 e) 27

20

SOLUO:
20
Seja n = 5 . Podemos escrever, usando logaritmo decimal:
20
log n = log 5 = 20.log5
Para calcular o valor do logaritmo decimal de 5, ou seja, log5, basta lembrar que podemos escrever:
log 5 = log (10/2) = log 10 - log 2 = 1 - 0,301 = 0,699
Portanto, log n = 20 . 0,699 = 13,9800
Da teoria vista acima, sabemos que se log n = 13,9800, isto significa que a caracterstica do log decimal vale 13 e,
portanto, o nmero n possui 13 + 1 , ou seja 14 algarismos.
Portanto, a resposta correta a letra B.
4 - UFBA - Considere a equao 10
log 3,68 = 0,5658 , calcule 10x.

x + 0,4658

= 368. Sabendo-se que

SOLUO:
x + 0,4658
= 368
Temos: 10
Da, podemos escrever:
log 368 = x + 0,4658 x = log 368 - 0,4658
Ora, dado que: log 3,68 = 0,5658, ou seja:
log(368/100) = 0,5658
Logo, log 368 - log 100 = 0,5658 log 368 - 2 = 0,5658 , j que
2
log 100 = 2 (pois 10 = 100).
Da, vem ento:
log 368 = 2,5658
Ento, x = log 368 - 0,4658 = 2,5658 - 0,4658 = 2,1
Como o problema pede o valor de 10x, vem: 10.2,1 = 21
Resp: 21
5 - Se log N = 2 + log 2 - log 3 - 2log 5 , calcule o valor de 30N.
SOLUO:
Podemos escrever:
2
logN = 2 + log2 - log3 - log5
logN = 2 + log2 - log3 - log25
logN = 2 + log2 - (log3 + log25)
Como 2 = log100, fica:
logN = (log100 + log2) - (log3 + log25)
logN = log(100.2) - log(3.25)
logN = log200 - log75
logN = log(200/75)

130__________________________________________________________________________________________________________________________
ATUALIZADO AT MAIO/2012

www.CARREIRAPUBLICA.com.br

(48) 4141-3220

4141-3222

____________

_____________________________ MATEMTICA _________________________________ Prof. Roberto

Logo, conclumos que N = 200/75


Simplificando, fica:
N = 40/15 = 8/3
Logo, 30N = 30(8/3) = 80 Resp: 30N = 80

16.6

EXERCCIOS PROPOSTOS
3

1 - UFBA - Sendo log2 = 0,301 e x = 5 .


, ento o logx :
*a) 2,997 b) 3,398 c) 3,633 d) 4,398 e) 5,097
2

2 - UEFS - O produto das razes da equao log(x -7x + 14) = 2log2 :


01) 5 02) 7 *03) 10 04) 14 05) 35
n+1

n+1

3 - UCSal - Se 12 = 3 . 8 , ento log2 n igual a:


a) -2 *b) -1 c) 1/2 d) 1 e) 2
4 - UEFS - O domnio da funo y = log [(2x-3)/(4-x)] :
a) (-3/2,4) b) (-4,3/2) c) (-4,2) *d) (3/2,4) e) (3/2,10)
5 - UFBA - Determine o valor de x que satisfaz equao log2 (x-3) + log2 (x-2) = 1. Resp: 4
6 - UFBA - Existe um nmero x diferente de 10, tal que o dobro do seu logaritmo decimal excede de duas unidades o
logaritmo decimal de x-9. Determine x.
Resp: 90
7 - PUC-SP - O logaritmo, em uma base x, do nmero y = 5 + x/2 2. Ento x igual a:
a) 3/2 b) 4/3 c) 2 d)5 *e) 5/2
2

8 - PUC-SP - Se x+y = 20 e x - y = 5 , ento log(x - y ) igual a:


a) 100 *b) 2 c) 25 d) 12,5 e) 1000
2
2
Sugesto: observe que x - y = (x - y) (x + y)
9 - Calcule o valor dos seguintes logaritmos:

a)

b)

c)

d)

e)

f)

10 - Calcule o valor da incgnita "N" em cada exerccio, aplicando a equivalncia fundamental:

a)

b)

c)

d)

11 - Calcule o valor da incgnita "a" em cada exerccio, aplicando a equivalncia fundamental:


a)

b)

c)

__________________________________________________________________________________________________________________________

ATUALIZADO AT MAIO/2012

www.CARREIRAPUBLICA.com.br

(48) 4141-3220

131

4141-3222

____________

_____________________________ MATEMTICA _________________________________ Prof. Roberto

12 - O nmero real x, tal que

(A)

(B)

(C)

(D)

(E)
13 - (PUCRS) Escrever

(A)
(B)
(C)
(D)

, equivale a escrever

(E)

14 - Se

, o valor de

(A) -2
(B) -1 X
(C) 0
(D) 1
(E) 2
15 - (PUCRS) A soluo real para a equao

, com a>0, a1 e b>0, dada por

(A)
(B)
(C)
(D)
(E)

16 - ( CESGRANRIO - RJ ) Se log ( 2x - 5 ) = 0 ento x vale:


a. 5

b. 4

c. 3

d. 7/3

e. 5/2

17 - ( FGV - SP ) A equao logx ( 2x +3 )= 2 apresenta o seguinte conjunto soluo:


a. { -1, 3 }

b. { -1 }

c. { 3 }

d. { 1, 3 }

e. nda

132__________________________________________________________________________________________________________________________
ATUALIZADO AT MAIO/2012

www.CARREIRAPUBLICA.com.br

(48) 4141-3220

4141-3222

____________

_____________________________ MATEMTICA _________________________________ Prof. Roberto


2

18 - (UEL-PR) correto afirmar que no universo IR o conjunto soluo da equao lo3 ( -x -10x ) = 2:
a.
b.
c.
d.
e.

unitrio
tem dois elementos irracionais
tem dois elementos inteiros
tem dois elementos racionais e no inteiros

19 - ( ESAL - MG ) O valor de x tal que log648 = x :


a. 2

b. 3

c. 2/3

d. 1/2

e.3/2
2

20 - ( PUC - SP ) Quanto a soluo da equao ( logx ) - 3. log x + 2 = 0 verdade que :


a.
b.
c.
d.
e.

s uma delas real


a maior delas 1000
a menor delas 100
a menor delas 10
a maior delas 1
2

21 - ( UEPG - PR ) Sendo ( log2x) - 3 log2x - 4 = 0 ento o produto entre as razes da equao vale:
a. -8

b. 16

c. -1/4

d. 4

e. 8

22 - ( CONSART - SP ) A soluo da equao log8x + log8 (3x-2) = 1 dada por:


a. -4/3

b. 1/2

c. -2

d. 2

e. nda

23 - ( PUC - SP ) O conjunto verdade da equao 2. log x = log 4 +log ( x + 3 ) :


a. { -2, 6 }

b. { -2 }

c. { 2, -6 }

d.

e. { 6 }
2

24 - ( CEFET - PR ) A soma das razes da equao log x - logx = 0 :


a. 1000

b. 1001

c. 101

d. 10001

e. 11

25 - ( UFSC ) Indica-se por log x o logaritmo decimal do nmero x. Se 4 + log x = 4. log 4, ento x igual a:
a. 16

b. 2,56

c. 0,4

d. 0,256

e. 0,0256
2

26 - ( UNIMEP - SP ) O logaritmo na base 2, do nmero x - x igual a 1. O valor de x que satisfaz a sentena :


a. 2 ou -1

b. -1 ou 0

c. 1

d. 0

e. 3

27 - ( PUC - SP ) Aumentando um nmero x de 16 unidades, seu logaritmo na base 3 aumenta de 2 unidades. Ento
x :
a. 2

b. 1

c. 3

d. 4

e. 5

28 - ( UEBA ) No universo IR a soluo da equao log2x + log2 ( x +1 )= 1 um nmero:


a. mpar

b. entre 0 e 1

c. maior que 3

d. mltiplo de 3

e. divisvel por 5

29 - ( UECE ) O conjunto soluo da equao log24x- log42 = 0 :


a. {

/4 }

b. {

/2}

c. {

d. {2

e. nda

__________________________________________________________________________________________________________________________

ATUALIZADO AT MAIO/2012

www.CARREIRAPUBLICA.com.br

(48) 4141-3220

133

4141-3222

____________

_____________________________ MATEMTICA _________________________________ Prof. Roberto

30 - ( CEFET - PR ) Se loga
a. 1

b. 4

c. 8

, ento b igual a :

d. 3

e. 9

31 - ( UEPG - PR ) Se log2x+log8x = 1 , ento x vale :


a.

b.

c. 2

d. 3

e. nda

32 - ( MACK - SP ) Se

a.
b.
c.
d.

, a > 0, a

1, ento o valor de x :

a
1/a
2
a
2
1/a

e.

33 - ( FGV - SP ) A soluo da equao


a. x= log2 ( 12/5 )

b. x = log2 ( 5/12 )

c. x = log5/122

d. x = log12/52

e. x = log125

34 - ( CEFETR - PR ) Se log2x - log4x = -1/2, ento x igual a:


a. 1/4

b. 4

c.

/2

d. 1/2

e.

35 - ( PUC - PR ) A diferena das solues da equao


a. 2

b. -2

c. 4

d. 0

, em modulo , :

e. 6
x

36 - (FUVEST-SP) O conjunto soluo da equao x . ( log53 + log521 ) + log5 ( 3/7) = 0 :


a.

17.

b. {0}

c. {1}

d. {0,2}

e. {0,-2}

EQUAES EXPONENCIAIS

17.1 Exerccios resolvidos

1)

2)

3)

4)

134__________________________________________________________________________________________________________________________
ATUALIZADO AT MAIO/2012

www.CARREIRAPUBLICA.com.br

(48) 4141-3220

4141-3222

____________

_____________________________ MATEMTICA _________________________________ Prof. Roberto

5)

6)

7)

1) Aplicando as propriedades de exponencial temos:


10x(x-1)=10-6
x(x-1)=-6
x2-x=-6
x2-x+6=0

Agora com as bases igualadas podemos cort-las.


Operando
Chegamos em uma equao do segundo grau, aplicando Bskara achamos os
resultados
Note que temos uma raiz quadrada de um nmero negativo! Isto no um nmero
do conjunto dos REAIS (R), portanto a resposta x R (x no pertence aos REAIS).

2) 4x2=256
4x2=44
x2=4

3) 2x2-7x+12=1
2x2-7x+12=20
x2-7x+12=0 (Bscara)
x=4 x=3
4)
Tirando MMC

5) 3x(x-4)=3-3
x(x-4)=-3
x2-4x=-3
x2-4x+3=0 (Bscara)
x'=3
x''=1

, ento "x" vale:

8) Se
(A) -1/6

6) 3x2-10x+7=3-2
x2-10x+7=-2
x2-10x+7+2=0
x2-10x+9=0 (Bscara)
x'=9
x''=1

82x+2x=18
92x=18
2x=2
x=1
7) 4-(x-1)=42(x+2)
-(x-1)=2(x+2)
-x+1=2x+4
-x-2x=4-1
-3x=3
x=-1

(B) -1/3

(C) -1/2

(D)

(E) 1/5

- Primeiro vamos transformar os decimais (nmeros com vrgula) em fraes:

- Veja que podemos simplificar a frao da esquerda e transformar em potncia o lado direito da igualdade:

- As bases esto quase igualadas, s que uma o inverso da outra. Vamos inverter uma delas e adicionar o
expoente "-1".
__________________________________________________________________________________________________________________________

ATUALIZADO AT MAIO/2012

www.CARREIRAPUBLICA.com.br

(48) 4141-3220

135

4141-3222

____________

_____________________________ MATEMTICA _________________________________ Prof. Roberto

- Agora sim, com as bases igualadas podemos cort-las:

Resposta certa letra "B".


9) (PUC-RS) A soma das raizes da equao 95x2-2x+1=5625 :
(A) -4

(B) -2

(C) -1

(D) 2

(E) 4

- Primeiro vamos "passar" o nove que est multiplicando o lado esquerdo para o lado direito dividindo:
x2-2x+1

=5625/9
=625

x2-2x+1

5
- Fatorando:

x2-2x+1

=5

- Cortando as bases:
x2-2x+1=4

x2-2x+1-4=0

x2-2x-3=0

- Sendo a frmula da soma das raizes S=-b/a, temos:


S=-(-2)/1
S=2 Resposta certa letra "D".
10) (UFRGS) Sabendo-se que 6x+2=72, tem-se que 6-x vale:
(A) -4

(B) -2

(C) 0

(D)

(E) 2
-x

- Para resolver este problema, no precisamos achar o valor de "x" . pedido quanto vale 6 , se ns calcularmos
quanto 6x podemos calcular o que pedido. Veja s:
x+2

=72

6 6 =72

6 36=72

6 =72/36

6 =2

- Agora podemos inverter ambos os lados que a igualdade continua verdadeira:

136__________________________________________________________________________________________________________________________
ATUALIZADO AT MAIO/2012

www.CARREIRAPUBLICA.com.br

(48) 4141-3220

4141-3222

____________

_____________________________ MATEMTICA _________________________________ Prof. Roberto

- Aplicando as propriedades de potenciao:


-x

6 =
Resposta certa letra "D"

17.2 EXERCCIOS PROPOSTOS

11) (UFRGS) O valor de x que verifica a equao


*(A) -1

(B) -1/2

(C) 0

(D)

(E) 1

12) (UFRGS) A soluo da equao


(A) -2

*(B)

(C) 14/15
x

(D) 15/14

(E) 2

x-1

13) (UFRGS) Sabendo que 4 -4 =24 ento x1/2 vale


(A)

/2

(B)

/2

(C)

(D)

/5

*(E)

14) (PUCRS) A soma das raizes da equao


*(A) -2

(B) -1

(C) 0

(D) 1

/2

(E) 2

15 - ( CESGRANRIO - RJ ) Se 8 = 32, ento x igual a:


a. 5/2 b. 5/3

c. 3/5

16 - ( UEPG - PR ) Se 8
a. 1

b. 2

c. 4

x-9

d. 2/5

e. 4

x/2

= 16 , ento

d. 5

igual a:

e. nda
3x-1

17 - ( PUC - SP ) O valor de x que satisfaz a equao 3


a. 1

b. 3

c. 5/2

d. 1/3

2 3

e.

3-x

= 27

e. 2/5

18 - ( FUVEST - SP ) Sendo x = (2 ) , y =
a.
b.
c.
d.

2x+3

.9

ez=

, calcule x . y . z :

21

2
10
2
23
2
4
2
20
2

19 - ( VUNESP - SP ) Se

, ento :

a. m = 0,1 b.
2
b. m = ( 0,1)
3
c. m = ( 0,1 )
__________________________________________________________________________________________________________________________

ATUALIZADO AT MAIO/2012

www.CARREIRAPUBLICA.com.br

(48) 4141-3220

137

4141-3222

____________

_____________________________ MATEMTICA _________________________________ Prof. Roberto


4

d. m = ( 0,1 )
e.
5
m = ( 0,1 )
x

20 - ( UFRN ) Se 2 = 2048, ento, x vale :


a. 7

b. 11

c. 13

d. 17

e. 19

, ento os valores de x so :

21 - ( PUC - SP ) Se
a. 1 e 3

b. 2 e 3

c. 1 e 2

d. 1 e 4

e. 2 e 4
2x

22 - ( FCC - BA ) A soluo da equao 0,5 = 0,25


a. 0 < x < 1

b. 1 < x < 2

c. 2 < x < 3

3 -x+2

23 - ( CEFET - PR ) Se ( 7 )
b. -9

a.

c. 49

,x

d.

1/2

1-x

um nmero x, tal que:

d. x > 3

e. x < 0

valer:

e. 1

-x

-x

24 - . ( UEL - PR ) Se 2 = u e 3 = t, o valor da expresso 12 + 18 :

a.
b.
c.
2
2
d. u + t
e.
3
3
u +t

25 - ( UFMG ) A soma das razes da equao


a. 0

b. -1

c. 1

d. 7

e. 8
x

26 - ( UFPA ) A raiz da equao ( 7 - 2


a. irracional negativo
d. inteiro negativo

).(7 +2

b. irracional positivo

) = 9 um nmero:

c. par

e. inteiro positivo

27 - ( PUC - RS ) Se 3 - 3
b. 16

, :

b. 15 c. 14

2-x

= 2 , ento 15 - x vale:

d. 11

e. 6
x

-x

-x

28 - ( UFBA ) O conjunto soluo da equao 2 - 2 = 5 ( 1 - 2 ) :


a. { 1; 4 }

b. {1 ; 2 }

c. { 0; 1 }

d. { 0; 2 }

e.

138__________________________________________________________________________________________________________________________
ATUALIZADO AT MAIO/2012

www.CARREIRAPUBLICA.com.br

(48) 4141-3220

4141-3222

____________

_____________________________ MATEMTICA _________________________________ Prof. Roberto


2x

29 - ( UEPG - PR ) A soma das razes da equao 3 - 12. 3 + 27 = 0 pertence ao intervalo:


a. [ 10, 12 ]

b. [ 0, 3 ]
x

c. [ 1, 2 ]

d. ( 10, 12 )

-x

e. ( 1, 3 )

-x

30 - ( UFPR ) Se 2 + 2 = 3, ento o valor de 8 + 8 :


a. 12

b. 18

c. 21

d. 24
16

31 - ( FUVEST - SP ) Se 4
a. 24

b. 25

c. 26

25

.5

e. 27
=

d. 27
x

. 10 , com 1

<10, ento n igual a:

e. 28
x

32 - ( FGV - SP ) A equao 4 + 6 = 2.9 tem como soluo o conjunto:


a. {1}

b. {2}

c. {3}
m

2n

33 - ( UECE ) Se 7 - 3

a. 16

b. 64

d. {0}

= 1672 e

c. 128

d. 256

34 - ( PUC - MG ) A expresso
a.
b.
c.
d.
e.

e. nda
n

- 3 = 22, ento m igual a:

e. nda

igual a:

2
-x
2
-3
2
7
8

35 - ( UFCE ) A soma das razes da equao x


a. 5 b. 6

c. 8

d. 9

f(x)

= 1, onde f(x) = x - 7x + 12, igual a :

e. 10
x+1

36 - ( CESGRANRIO - RJ ) Os nmeros inteiros x e y satisfazem 2


a. 0

b. 1

c. 2

d. 3

y+2

+2 =3

- 3 . Ento x :

e. 4

18 ANLISE COMBINATRIA

Foi a necessidade de calcular o nmero de possibilidades existentes nos chamados jogos de azar que levou ao
desenvolvimento da Anlise Combinatria, parte da Matemtica que estuda os mtodos de contagem. Esses estudos
foram iniciados j no sculo XVI, pelo matemtico italiano Niccollo Fontana (1500-1557), conhecido como Tartaglia.
Depois vieram os franceses Pierre de Fermat (1601-1665) e Blaise Pascal (1623-1662).
A Anlise Combinatria visa desenvolver mtodos que permitam contar - de uma forma indireta - o nmero de
elementos de um conjunto, estando esses elementos agrupados sob certas condies

18.1 Fatorial
Notao : ( ! )
Def . : Denomina-se fatorial de um nmero positivo inteiro n ao produto
n ( n - 1 ) ( n - 2 ) ( n - 3 ) ( n - 4 ) ..... e anotamos n !
__________________________________________________________________________________________________________________________

ATUALIZADO AT MAIO/2012

www.CARREIRAPUBLICA.com.br

(48) 4141-3220

139

4141-3222

____________

_____________________________ MATEMTICA _________________________________ Prof. Roberto

Ex . : a ) 7 ! = 7.6.5.4.3.2.1 = 5040

b) 4! = 4.3.2.1 = 24

Por Definio
1! = 1

0! = 1

Simplifique as expresses abaixo:


a)

10!
8!

b)

( n + 1)!
( n 1)!

e)

12 !
9 !.3!

f)

m!
(m 2)!

c)

( n + 1)!
n

d)

m!
( m 1)!

18.2 Princpio fundamental da contagem


O princpio fundamental da contagem nos diz que sempre devemos multiplicar os nmeros de opes entre as
escolhas que podemos fazer. Por exemplo, para montar um computador, temos 3 diferentes tipos de monitores, 4
tipos de teclados, 2 tipos de impressora e 3 tipos de "CPU". Para saber o numero de diferentes possibilidades de
computadores que podem ser montados com essas peas, somente multiplicamos as opes:
3 x 4 x 2 x 3 = 72
Ento, tm-se 72 possibilidades de configuraes diferentes.
Um problema que ocorre quando aparece a palavra "ou", como na questo:
Quantos pratos diferentes podem ser solicitados por um cliente de restaurante, tendo disponvel 3 tipos de arroz, 2 de
feijo, 3 de macarro, 2 tipos de cervejas e 3 tipos de refrigerante, sendo que o cliente no pode pedir cerveja e
refrigerante ao mesmo tempo, e que ele obrigatoriamente tenha de escolher uma opo de cada alimento?
A resoluo simples: 3 x 2 x 3 = 18 , somente pela comida. Como o cliente no pode pedir cerveja e refrigerantes
juntos, no podemos multiplicar as opes de refrigerante pelas opes de cerveja. O que devemos fazer aqui
apenas somar essas possibilidades:
(3 x 2 x 3) x (2 + 3) = 90
Resposta para o problema: existem 90 possibilidades de pratos que podem ser montados com as comidas e bebidas
disponveis.
Outro exemplo:
No sistema brasileiro de placas de carro, cada placa formada por trs letras e quatro algarismos. Quantas placas
onde o nmero formado pelos algarismos seja par, podem ser formadas?
Primeiro, temos de saber que existem 26 letras. Segundo, para que o numero formado seja par, teremos de limitar o
ultimo algarismo um numero par. Depois, basta multiplicar.
26 x 26 x 26 = 17.567 -> parte das letras
10 x 10 x 10 x 5 = 5.000 -> parte dos algarismos, note que na ltima casa temos apenas 5 possibilidades, pois
queremos um nmero par (0 , 2 , 4 , 6 , 8).
Agora s multiplicar as partes: 17.567 x 5.000 = 87.835.000
Resposta para a questo: existem 87.835.000 placas onde a parte dos algarismos formem um nmero par.
18.3 Arranjo simples
Na aplicao An , p calculamos quantos nmeros de 2 algarismos distintos podemos formar com 1,2,3,4
12,13,14,21,23,24,31,32,34,41,42,43

140__________________________________________________________________________________________________________________________
ATUALIZADO AT MAIO/2012

www.CARREIRAPUBLICA.com.br

(48) 4141-3220

4141-3222

____________

_____________________________ MATEMTICA _________________________________ Prof. Roberto

Observe que os nmeros em questo diferem ou pela ordem dentro do grupamento (12 21 ) ou pelos elementos
componentes (13 24 ) cada nmero se comporta como uma sequncia, isto :
( 1,2 2,1 ) a ordem diferente, mas os nmeros so os mesmos.
( 1,3 3,4 ) os nmeros so diferentes
A este tipo de grupamento chamamos de arranjo simples
Def . : Dado um conjunto com n elementos, chama-se a chama-se arranjo do p elementos distintos a todo grupo
formado por p dos n elementos, sendo um grupo distinto do outro quando em cada um dos dois houver, pelo menos :
Um dos p elementos distintos
Frmula :
p
A=
n!
n ( n-p )!

ou

dois dos p elementos em ordem diferente

{p,n}N

n = nmero total de elementos p = nmero de elementos de cada arranjo


Ex . : Seja o conjunto de trs elementos { a, b, c } e formemos arranjos de dois elementos
( A 2,3)
p
2
A n,p = n!
A=
3! . = 3. 2. 1 = 6
n
(n-p)!
3 (3-2)!
1

Arranjo simples: No ocorre a repetio de qualquer elemento em cada grupo de p elementos.


Frmula: As(n,p) = n!/(n-p)!
Clculo para o exemplo: As(4,2) = 4!/2!=24/2=12.
Exemplo: Seja Z={A,B,C,D}, n=4 e p=2. Os arranjos simples desses 4 elementos tomados 2 a 2 so 12 grupos que
no podem ter a repetio de qualquer elemento mas que podem aparecer na ordem trocada. Todos os
agrupamentos esto no conjunto:
As={AB,AC,AD,BA,BC,BD,CA,CB,CD,DA,DB,DC}
Arranjo com repetio: Todos os elementos podem aparecer repetidos em cada grupo de p elementos.
p

Frmula: Ar(n,p) = n .
2

Clculo para o exemplo: Ar(4,2) = 4 =16.


Exemplo: Seja C={A,B,C,D}, n=4 e p=2. Os arranjos com repetio desses 4 elementos tomados 2 a 2 so 16 grupos
que onde aparecem elementos repetidos em cada grupo. Todos os agrupamentos esto no conjunto:
Ar={AA,AB,AC,AD,BA,BB,BC,BD,CA,CB,CC,CD,DA,DB,DC,DD}
Arranjo condicional: Todos os elementos aparecem em cada grupo de p elementos, mas existe uma condio que
deve ser satisfeita acerca de alguns elementos.
Frmula: N=A(n1,p1).A(n-n1,p-p1)
Clculo para o exemplo: N=A(3,2).A(7-3,4-2)=A(3,2).A(4,2)=612=72.
Exemplo: Quantos arranjos com 4 elementos do conjunto {A,B,C,D,E,F,G}, comeam com duas letras escolhidas no
subconjunto {A,B,C}?
Aqui temos um total de n=7 letras, a taxa p=4, o subconjunto escolhido tem n1=3 elementos e a taxa que este
subconjunto ser formado p1=2. Com as letras A,B e C, tomadas 2 a 2, temos 6 grupos que esto no conjunto:
__________________________________________________________________________________________________________________________

ATUALIZADO AT MAIO/2012

www.CARREIRAPUBLICA.com.br

(48) 4141-3220

141

4141-3222

____________

_____________________________ MATEMTICA _________________________________ Prof. Roberto

PABC = {AB,BA,AC,CA,BC,CB}
Com as letras D,E,F e G tomadas 2 a 2, temos 12 grupos que esto no conjunto:
PDEFG = {DE,DF,DG,ED,EF,EG,FD,FE,FG,GD,GE,GF}
Usando a regra do produto, teremos 72 possibilidades obtidas pela juno de um elemento do conjunto PABC com um
elemento do conjunto PDEFG. Um tpico arranjo para esta situao CAFG.

18.3.1 EXERCCIOS
1. Dado o conjunto de 4 elementos { a, b, c, d}, formar arranjos com 3 elementos. R. 24
2. Dado o conjunto {1, 2, 3, 4, 5}, determinar o nmero de arranjo de 4 elementos.
R. 120
3. No Estado de Santa Catarina as placas de automveis possuem 3 letras seguidas de 4 algarismos. Determinar o
nmero de placas que comeam por ABC e no tem algarismos repetidos.
4. Doze atletas disputam uma prova. De quantas maneiras distintas podemos formar o grupo dos 3 primeiros
colocados (medalha de ouro, prata e bronze)? R. 1.320
5. Resolver os problemas:
a) Quantos nmeros de 5 algarismos distintos podem ser formados com os algarismos de 1 a 8. R. 6.720
b) Vinte tenistas disputam um torneio regional. De quantos modos diferentes podem ser formado o grupo dos 5
primeiros colocados, que iro disputar num torneio internacional? R. 1.860.480
c) Os 25 alunos de uma classe querem formar uma comisso de 3 alunos, sendo um presidente, um secretrio e um
tesoureiro. Quantas comisses diferentes podero formar? R. 13.800
6. Quantas palavras de 3 letras, sem repetio, podemos formar com as 9 primeiras letras do nosso alfabeto ?
7. Com os algarismos 1, 2, 3, 4, 5 e 6 so formados nmeros de 4 algarismos distintos. Dentre eles, quantos so
divisveis por 5 ?
8. Considere o conjunto A = { 2, 4, 5, 6 }
a) Calcule quantos algarismos diferentes podemos formar com os elementos de A
b) Dos nmeros obtidos no tem anterior, quantos so mltiplos de 5 ?
9. Numa sala de 20 alunos, deseja-se formar grupos de estudos de 3 elementos, que tenham projetos diferentes.
a) De quantos modos diferentes se pode escolher os alunos ? R. 6.840
b) De quantas maneiras se podem escolher os alunos, sabendo-se que 2 dos alunos no podem pertencer ao
mesmo grupo ? R. 6.732
10. Resolver a equao A n,6 + A n,5 . = 9
A n,4

R. 7

18.4 Permutao simples


Consideramos os nmeros de 3 algarismos distintos formados com os algarismos 1, 2 e 3 esses nmeros so:
123, 132, 213, 231, 321, 312
A quantidade desses nmeros dada por A3. 3 = 6
Esses nmeros se diferem entre s somente pela posio de seus elementos . Cada nmeros chamado de
permutao simples , obtida com algarismos 1, 2 e 3.

142__________________________________________________________________________________________________________________________
ATUALIZADO AT MAIO/2012

www.CARREIRAPUBLICA.com.br

(48) 4141-3220

4141-3222

____________

18.4.1

_____________________________ MATEMTICA _________________________________ Prof. Roberto

Definio

Dado um conjunto com n elementos, chama-se permutao simples de n elementos distintos a todo grupo formado
pelos n elementos, sendo um grupo distinto do outro quando em cada um dos dois houver pelo menos, dois dos n
elementos em ordem diferente
Pn = n!
Ex.: Considere a palavra CAF. Quantas permutaes podemos formar?
Pn = n!

P4 = 4!

P4 = 4.3.2.1 = 24

18.4.2 EXERCCIOS
1. Na palavra ATREVIDO.
a) Quantas permutaes podemos formar
b ) Quantos anagramas comeam com a letra A
c ) Quantos anagramas comeam com a slaba TRE
d ) Quantos anagramas possuem a slaba TRE

R. 40320
R. 5040
R.
120
R.
720

2 ) Quantos anagramas podemos formar com a palavra CAPTULO


3 ) Quantos nmeros de 4 algarismo distintos podemos formar com os dgitos 2, 4, 6, 8 ?
4 ) De quantos modos diferentes 6 pessoas podem sentar em 6 cadeiras alinhadas ?

R.

24

R. 720

5 ) Cinco candidatas participam de um concurso de beleza. Quantos so os possveis resultados ? R . 120


6 ) Possuo 5 livros de Matemtica, 4 de Estatstica e 3 de Raciocnio lgico. De quantas maneiras posso dispo-los em
uma prateleira, se desejo que os livros de uma mesma matria fiquem sempre juntos? R. 103.680
Pm + m.P(m-2) . = 3/8
P(m + 1)
8) Permutando os algarismos 2, 4, 6 e 8, formamos nmeros. Dispondo esses nmeros em ordem crescente, qual o
nmero que ocupa a vigsima Segunda posio ?

7 ) Calcule o valor de m que verifica a equao

9) Considere os nmeros obtidos do nmero 12.345, efetuando-se todas as permutaes de seus algarismos.
Colocando esses nmeros em ordem crescente, qual o lugar ocupado pelo nmero 43.521 ?
10) De quantos modos podemos ordenar 2 livros de Matemtica, 3 de Portugus e 4 de Fsica, de modo que os livros
de uma mesma matria fiquem sempre juntos e, alm disso, os de Fsica fiquem, entre si, sempre na mesma ordem.

18.4.3
, ,
Pn=

Permutao com elementos repetidos

n!
.
!
!!

Ex . : Qual o nmero de anagramas que podemos formar com a palavra ARARA.

3,2
=A=3
Pn =
n!
. P5 = 5! . P5 = 10
=R=2
! !
3! 2!
n=5
__________________________________________________________________________________________________________________________

ATUALIZADO AT MAIO/2012

www.CARREIRAPUBLICA.com.br

(48) 4141-3220

143

4141-3222

____________

_____________________________ MATEMTICA _________________________________ Prof. Roberto

18.4.4 Exerccios
1 ) Qual o nmero de permutaes que podemos formar com a palavra PARALELA ?
2 ) Quantos anagramas podemos formar com a palavra MATEMTICA , quantos anagramas comeam com a slaba
MA.
3 ) Quantos anagramas podemos formar com a palavra :
a ) Diferente
b ) Determinao
c ) Elemento
4 ) Determine a quantidade de nmeros distintos que podemos obter permutando-se os algarismos dos nmeros a)
73.431 b) 343.434
5 ) Quantos anagramas podem ser formados com as letras da palavra ARAPONGA, de modo que a letra P ocupe
sempre o ltimo lugar.
6 ) Usando uma vez a letra A, uma vez a letra B e ( n- 2 ) vezes a letra C, podemos formar 20 anagramas diferentes
com n letras em cada anagrama. Calcule n.

18.5 Combinao simples

13.5.1 Def . : Dado um conjunto de n elementos, chama-se de combinao simples de p elementos distintos a
todo subconjunto formado por p dos n elementos.
Um subconjunto distinto de outro, quando em cada um dos dois houver pelo menos um dos p elementos distinto.
Frmula
p
C =
n

n! .
p!(n-p)!

Exemplos
1 ) Quantos subconjuntos de 3 elementos tem um conjunto de 5 elementos
Cn,p =

n !
p!(n-p)!

C5,3 =

5!
.
3! ( 5 - 3 )!

C5,3 = 10

2 ) Dado um conjunto de 7 elementos calcule a quantidade de subconjuntos de 2 elementos.


C7,2 =

7 !
2!(7-2)!

C7,2 = 21

Combinao simples: No ocorre a repetio de qualquer elemento em cada grupo de p elementos.


Frmula: C(n,p) = n!/[(n-p)! p!]
Clculo para o exemplo: C(4,2)=4!/[2!2!]=24/4=6
Exemplo: Seja C={A,B,C,D}, n=4 e p=2. As combinaes simples desses 4 elementos tomados 2 a 2 so 6 grupos
que no podem ter a repetio de qualquer elemento nem podem aparecer na ordem trocada. Todos os
agrupamentos esto no conjunto:
Cs={AB,AC,AD,BC,BD,CD}
Combinao com repetio: Todos os elementos podem aparecer repetidos em cada grupo at p vezes.

144__________________________________________________________________________________________________________________________
ATUALIZADO AT MAIO/2012

www.CARREIRAPUBLICA.com.br

(48) 4141-3220

4141-3222

____________

_____________________________ MATEMTICA _________________________________ Prof. Roberto

Frmula: Cr(n,p)=C(n+p-1,p)
Clculo para o exemplo: Cr(4,2)=C(4+2-1,2)=C(5,2)=5!/[2!3!]=10
Exemplo: Seja C={A,B,C,D}, n=4 e p=2. As combinaes com repetio desses 4 elementos tomados 2 a 2 so 10
grupos que tm todas as repeties possveis de elementos em grupos de 2 elementos no podendo aparecer o
mesmo grupo com a ordem trocada. De um modo geral neste caso, todos os agrupamentos com 2 elementos formam
um conjunto com 16 elementos:
Cr={AA,AB,AC,AD,BA,BB,BC,BD,CA,CB,CC,CD,DA,DB,DC,DD}
mas para obter as combinaes com repetio, deveremos excluir deste conjunto os 6 grupos que j apareceram
antes, pois AB=BA, AC=CA, AD=DA, BC=CB, BD=DB e CD=DC, assim as combinaes com repetio dos
elementos de C tomados 2 a 2, so:
Cr={AA,AB,AC,AD,BB,BC,BD,CC,CD,DD}

18.5.1 Exerccios
1 ) Quantas comisses com 4 alunos podemos formar numa classe de 20 alunos ?

R. 4.845

2 ) Quantas comisses de 8 pessoas podemos formar com um grupo de 10 pessoas ?

R.

45

3 ) Com 7 professores quantas comisses de 3 professores podemos formar ?

R.

35

4 ) 12 pessoas participam de um jantar. No final eles se despedem com um aperto de mo. Quantos apertos de mo
sero dados no total ? R. 66
5 ) Com 4 homens e 6 mulheres, quantas comisses de 5 pessoas podemos formar se desejo que em cada
comisso esteja presente, pelo menos, um homem ?
6) Com 5 mulheres e 5 homens, quantas comisses de 4 pessoas podemos formar se desejo que cada comisso
tenha, no mximo, 2 mulheres ?
7 ) Calcule x na equao abaixo
2
4 C = 60
R. 6
x
8 ) Complete: x vale ......
4
3
C : C = 1
R. 7
x
x
9 ) Calcule o numero de diagonais de um hexgono regular. R. 9
10 ) Calcule o nmero de diagonais de um icosgono. R. 170
11) De quantas maneiras podemos escalar um time de futebol de salo dispondo de 8 jogadores ?
12) Sobre uma reta marcam-se 8 pontos e sobre uma outra reta, paralela primeira, marcam-se 5 pontos.Quantos
tringulos obteremos unindo 3 pontos quaisquer desses pontos ? R. 220
13) Uma empresa formada por 6 scios brasileiros e 4 japoneses. De quantos modos podemos formar uma diretoria
de 5 scios, sendo 3 brasileiros e 2 japoneses ?
14) Um examinador dispe de 6 questes de lgebra e 4 de Geometria para montar uma prova de 4 questes.
Quantas provas diferentes ele pode montar usando 2 questes de lgebra e 2 de Geometria ?

18.5.2 Exerccos resolvidos

__________________________________________________________________________________________________________________________

ATUALIZADO AT MAIO/2012

www.CARREIRAPUBLICA.com.br

(48) 4141-3220

145

4141-3222

____________

_____________________________ MATEMTICA _________________________________ Prof. Roberto

1 - Maria vai sair com suas amigas e, para escolher a roupa que usar, separou 2 saias e 4 blusas. De quantas
maneiras ela pode se arrumar?
Soluo: O chamado Princpio Fundamental da Contagem (PFC) diz, se alguma escolha pode ser feita de M
diferentes maneiras e alguma escolha subseqente pode ser feita de N diferentes maneiras, h MN diferentes
maneiras pelas quais essas escolhas podem ser feitas sucessivamente. Observe a tabela abaixo:
blusa 1

blusa 2

blusa 3

blusa 4

saia 1 saia 1 e blusa 1 saia 1 e blusa 2 saia 1 e blusa 3 saia 1 e blusa 4


saia 2 saia 2 e blusa 1 saia 2 e blusa 2 saia 2 e blusa 3 saia 2 e blusa 4
Contando as possibilidades, vemos que Maria pode se arrumar de 8 maneiras distintas. De fato, a ao constituda
de duas etapas sucessivas. A primeira (vestir a saia) pode ser realizada de 2 maneiras distintas. Para cada uma
dessas possibilidades, a segunda (vestir a blusa) pode ser realizada de 4 maneiras distintas. Assim, pelo princpio
fundamental da contagem (PFC), o nmero de efetuar a ao completa 2 4 = 8.

2 - H quatro estradas ligando as cidades A e B, e trs estradas ligando as cidades B e C. De quantas maneiras
distintas pode-se ir de A a C, passando por B?
Soluo: A ao constituda de duas etapas sucessivas. A primeira (ir de A at B) pode ser realizada de 3
maneiras. Para cada uma dessas possibilidades, a segunda (ir de B at C) pode ser realizada de 4 maneiras. Ento,
pelo PFC, o nmero de maneiras de ir de A at C 3 4 = 12.
3 - Uma prova de Matemtica consta de 10 questes do tipo V ou F. Se todas as questes forem respondidas ao
acaso, qual o nmero de maneiras de preencher a folha de resposta?
Soluo: O PFC tambm conhecido como PRINCPIO MULTIPLICATIVO e pode ser generalizado para aces
constitudas de mais de duas etapas sucessivas.
Resolver uma prova de 10 questes do tipo V ou F representa uma ao constituda de 10 etapas sucessivas, que
correspondem resoluo das 10 questes propostas. Para cada questo, h duas possibilidades de escolha de
10
resposta: V ou F. Logo, pelo PFC, o resultado procurado : 2 2 2 2 2 2 2 2 2 2 = 2 = 1024.
4 - De quantas maneiras podemos arrumar 5 pessoas em fila indiana?
Soluo: Ao escolher uma pessoa para ocupar a primeira posio na fila temos 5 possibilidades; para o segundo
lugar, como uma pessoa j foi escolhida, temos 4 possibilidades; para o terceiro lugar sobram 3 pessoas a serem
escolhidas; para o quarto lugar 2 pessoas, e para o ltimo lugar na fila sobra apenas a pessoa ainda no escolhida.
Ento, pelo PFC temos: 5 4 3 2 1 = 120. Assim, calculamos o nmero de modos de ordenar ("embaralhar") 5
elementos distintos. Em outras palavras , calculamos o nmero de permutaes simples de 5 elementos, ou seja, P5
= 120.
5 - Uma multiplicao do tipo N (N - 1) (N - 2) ... 1 chamada Fatorial do nmero N (N natural e N > 1) e
representada por N! ( lemos N fatorial ). Definimos ainda 1! = 1 e 0! = 1. O nmero de permutaes simples de N
elementos N! (por exemplo, o nmero de permutaes de 5 elementos 5! = 120). Calcule 10! / (6!4!).
Soluo:

6 - Para a eleio do corpo dirigente de uma empresa candidatam-se oito pessoas. De quantas maneiras podero ser
escolhidos presidente e vice-presidente?
Soluo: Para escolher o presidente temos 8 possibilidades; para escolher o vice-presidente, como uma pessoa j foi
escolhida, temos 7 possibilidades. Assim pelo PFC temos: 8 7 = 56 maneiras.
Por outro lado, poderamos usar a frmula:

146__________________________________________________________________________________________________________________________
ATUALIZADO AT MAIO/2012

www.CARREIRAPUBLICA.com.br

(48) 4141-3220

4141-3222

____________

_____________________________ MATEMTICA _________________________________ Prof. Roberto

Este procedimento chamado de clculo do nmero de arranjos simples de 2 elementos escolhidos entre 8
elementos, ou seja A8,2 = 56
7 - Em uma obra havia trs vagas para pedreiro. Cinco candidatos se apresentaram para preencher as vagas. De
quantas formas o encarregado da obra pode escolher os trs de que precisa?
Soluo: Note que ele no vai usar todos os candidatos, de 5 escolher apenas 3. Alm disso, a ordem em que ele
vai escolh-los no faz diferena (se escolher primeiro Mrio, depois Jos e por ltimo Pedro, ou primeiro Jos,
depois Pedro e por ltimo Mrio, o grupo escolhido o mesmo). Se a ordem de escolha dos candidatos importasse,
poderamos usar o PFC. Nesse caso, teramos 5 candidatos para a primeira vaga, 4 candidatos para a segunda e 3
candidatos para a ltima. A soluo seria 5 4 3 = 60. No entanto, usando o PFC, contamos vrias vezes o mesmo
grupo de trs candidatos. Para "tirar" as repeties, vamos ter que dividir o resultado pelo nmero de vezes que eles
se repetem na contagem. Os grupos repetidos so as formas de "embaralhar" trs candidatos escolhidos. Sabemos
que "embaralhar" trs objetos o mesmo que fazer permutaes de trs objetos.
Logo, basta dividir 60 por 3!, ou seja, dividir 60 por 6 para no contarmos as repeties dentro de cada grupo
formado. Isso significa que h 10 maneiras de escolher os trs novos pedreiros, entre os 5 candidatos.
De outra maneira, podemos usar a frmula do nmero binomial:

Assim, calculamos o nmero de combinaes simples de 5 objetos (os 5 candidatos) tomados 3 a 3 (apenas 3 sero
escolhidos), isto , C5,3 = 10

8 - Quantos veculos podem ser emplacados num sistema em que cada placa formada por 2 letras (de um total de
26) e 4 algarismos (de 0 a 9)?
Soluo: Pelo PFC temos: 26 26 10 10 10 10 = 6760000 veculos.
9 - Quantos veculos podem ser emplacados num sistema em que cada placa formada por 3 letras (de um total de
26) e 4 algarismos (de 0 a 9)?
Soluo: Pelo PFC temos: 26 26 26 10 10 10 10 = 175760000 veculos
10 - ANAGRAMA uma palavra formada pela transposio (troca, ou permutao, ou "embaralhamento") de letras
de outra palavra. Por exemplo: ROMA, MARO, OMAR, RAMO, MORA, RAOM, so alguns dos 24 anagramas da
palavra AMOR . Quantos so os anagramas da palavra MARTELO?
Soluo: Pelo PFC temos: 7654321 = 7! = 5040 anagramas.
11 - A senha de um carto eletrnico formada por duas letras distintas seguidas por uma sequncia de trs
algarismos distintos. Quantas senhas poderiam ser "confeccionadas"?
__________________________________________________________________________________________________________________________

ATUALIZADO AT MAIO/2012

www.CARREIRAPUBLICA.com.br

(48) 4141-3220

147

4141-3222

____________

_____________________________ MATEMTICA _________________________________ Prof. Roberto

Soluo: Pelo PFC temos: 26251098 = 468000 senhas.


12 - O quadrangular final de um torneio mundial de basquete disputado por quatro selees: Brasil, Cuba, Rssia e
EUA. De quantas maneiras distintas podemos ter os trs primeiros colocados?
Soluo: Pelo PFC temos: 432 = 24 maneiras distintas (nmero de arranjos de trs selees escolhidos entre 4
selees).
13 - Um colgio Estadual da cidade do Rio de Janeiro quer organizar um torneio de futebol com 8 equipes, da forma
que cada equipe jogue exatamente uma vez com cada uma das outras (octogonal). Quantos jogos ter o torneio?
Soluo: Dado o conjunto de 8 equipes, cada jogo um subconjunto de 2 equipes (a ordem no tem importncia).
Assim, o nmero de jogos o nmero de combinaes de 2 equipes escolhidos entre 8 equipes, ou seja, C8,2 = (87)
/ 2! = 56/2 = 28 jogos.
14 - Considere a palavra ARARA. Se todas as 5 letras (elementos) fossem distintas, teramos 5! = 120 anagramas
(permutaes). Entretanto, devemos dividir esse nmero por 3! (que o nmero de permutaes das letras A, A e A,
porque elas no so distintas) e por 2! (nmero de permutaes das letras R e R, porque elas no so distintas).
Assim. a palavra ARARA tem 10 anagramas. Quantos anagramas podemos formar com a palavra CARRETA?
Soluo: O nmero de anagramas : 7!/(2!2!) = 7!/4 = 5040/4 =1260 anagramas.
15 - De quantas formas 12 pessoas podem sentar-se ao redor de uma mesa circular?
Soluo: O nmeros de maneiras de colocar 12 pessoas em uma fila o nmero de permutaes simples de 12
elementos, isto , 12!. Ao colocarmos estas 12 pessoas sentadas ao redor de uma mesa circular teremos 12
permutaes correspondendo a uma nica permutao, pois, agora o primeiro da "fila" vizinho do ltimo. Na
verdade no existe mais o primeiro e nem o ltimo. Agora temos uma "fila" sem incio e sem fim. Ento, no nmero de
permutaes de 12 elementos, existem 12 permutaes repetidas.
Logo, temos que "tirar" estas 12 permutaes do clculo dividindo 12! por 12. Assim, o nmero de permutaes
pedido 12! / 12 = 11! = 39916800.
Este procedimento freqentemente chamado de clculo do nmero de permutaes circulares ou cclicas. De um
modo geral, o nmero de permutaes circulares de n elementos (n natural e n > 0) n! / n .
16 - Um edifcio tem 16 portas. De quantas formas uma pessoa poder entrar no edifcio e sair por uma porta
diferente de que usou para entrar?
Soluo: Para entrar existem 16 possibilidades, em seguida, para sair existem 15 possibilidades. Ento pelo PFC,
existem 1615 = 240 possibilidades.
17 - Ao final de uma reunio com 16 pessoas, cada um dos presentes cumprimentou os demais com um aperto de
mo uma nica vez. Quantos apertos de mo foram trocados?
Soluo: Temos um grupo de 16 pessoas. Uma pessoa qualquer desse grupo deve ter apertado a mo de 16-1 = 15
pessoas, e isso verdade para cada uma das 16 pessoas presentes. Mas para no contarmos duas vezes (2!) o
aperto de mo dado por duas pessoas quaisquer, temos que contar o nmero total de apertos de mo como o
nmero de combinaes de 2 pessoas escolhidas entre 16 pessoas, ou seja, C16,2 = (1615) / 2! = 120 apertos de
mo.
Quantos divisores positivos tem o nmero 3888?
4

Soluo: Decompondo em fatores primos, vem que: 3888 = 2 3 . Ento, cada divisor de 3888 da forma 2 3
onde a pode ser 0, 1, 2, 3, 4, e b pode ser 0, 1, 2, 3, 4, 5. Portanto, existem 5 possibilidades para a e 6 possibilidades
para b. Logo, pelo PFC, o nmero de divisores 56 = 30 divisores.
18 - Em um baralho de 52 cartas, trs so escolhidas sucessivamente. Quantas so as seqncias de resultados
possveis se a escolha for feita com reposio?
Soluo: Pelo PFC temos: 525252 = 140608 seqncias.
19 - Em um baralho de 52 cartas, trs so escolhidas sucessivamente. Quantas so as seqncias de resultados
possveis se a escolha for feita sem reposio?
Soluo: Pelo PFC temos: 525150 = 132600 seqncias.

148__________________________________________________________________________________________________________________________
ATUALIZADO AT MAIO/2012

www.CARREIRAPUBLICA.com.br

(48) 4141-3220

4141-3222

____________

_____________________________ MATEMTICA _________________________________ Prof. Roberto

20 - So sorteados na mega-sena seis nmeros escolhidos entre os nmeros de 1 a 60. Quantos so os resultados
possveis para o sorteio da mega-sena?
Soluo: No jogo da mega-sena, a ordem com que os seis nmeros so escolhidos no tem importncia, portanto, o
resultado deste jogo no uma seqncia.
Como o resultado um subconjunto, ento o nmero de resultados possveis neste tipo de loteria o nmero de
combinaes de 6 nmeros escolhidos entre 60 nmeros, ou seja,
C60,6 = (60 59 58 57 56 55) / 6! = 10 59 29 19 14 11 = 50063860 resultados possveis.
Nota: Observe que a probabilidade (nmero de chances) que uma pessoa tem de acertar nesta loteria fazendo uma
nica aposta de seis nmeros de 1 em 50063860.
21 - Uma prova de Matemtica contm dez questes do tipo mltipla escolha, tendo cada questo cinco alternativas.
Se todas as questes forem respondidas ao acaso, qual o nmero de maneiras de preencher a folha de resposta?
Soluo: Resolver a prova representa uma ao constituda de 10 etapas sucessivas, que correspondem resoluo
das 10 questes propostas. Para cada questo, h 5 possibilidades de escolha de resposta.
10
Ento, pelo PFC temos: 5 5 5 5 5 5 5 5 5 5 = 5 = 9765625 maneiras.
22 - Sobre uma circunferncia, tomam-se 10 pontos. Quantos tringulos podem ser construdos com vrtices nesses
pontos?
Soluo: Para construir um tringulo precisamos escolher 3 pontos (vrtices) dentre os 10 pontos disponveis, e
mais, a ordem com que esta escolha feita no tem importncia. Logo, o nmero de tringulos o nmero de
combinaes de 3 vrtices escolhidos entre 10 pontos, ou seja, C10,3 = (10 9 8) / 3! = 720 / 6 = 120 tringulos.
18.5.3 Exerccios
1 ) Determine o nmero de placas de carro que podem ser formadas contendo 2 letras distintas, seguidas por 3
algarismos, com o primeiro diferente de zero. R. 585.500
2 ) Uma agncia de propaganda deve criar o nome de um produto novo a partir de 4 slabas significativas, j
definidas. Qualquer uma dessas 4 slabas, sozinhas ou combinadas com uma ou mais das outras trs, poder formar
um nome atraente. Calcule o nmero de nomes diferentes possveis de serem montados, sem repetio de slabas.
R. 64
3 ) Uma empresa distribui a cada candidato a emprego um questionrio com trs perguntas. Na primeira, o candidato
deve declarar a sua escolaridade, escolhendo uma das cinco alternativas. Na Segunda, deve escolher em ordem de
preferncia, trs dos seis locais onde gostaria de trabalhar. Na ltima, deve escolher dois dias da semana em que
quer folgar. Quantos questionrios com conjuntos diferentes de respostas pode o examinador encontrar? R. 12.600
4 ) Uma empresa tem 5 diretores e 10 gerentes. Quantas comisses constitudas de 1 diretor e 4 gerentes podem
ser formadas ? R. 1.050
5 ) Calcule o nmero de diagonais do dodecgono. R . 54
6 ) So dados 12 pontos em um plano, dos quais 5 e somente 5 esto alinhados. Quantos tringulos podem ser
formados com vrtices em 3 dos 12 pontos. R. 210
7 ) Num acidente automobilstico, depois de ouvidas vrias testemunhas, conclui-se que o motorista culpado pelo
acidente dirigia um veculo cuja placa era constituda de duas vogais distintas e quatro algarismos diferentes, e o
algarismo das unidades era o dgito 2. Calcule o nmero de carros suspeitos do acidente. R 10.080
8 ) Do cardpio de uma festa constavam dez diferentes tipos de salgadinhos., dos quais s quatro seriam servidos
quentes. O garom encarregado de arrumar a travessa e servi-la foi instrudo para que a mesma contivesse s 2
diferentes tipos de salgadinhos frios e s 2 diferentes tipos quentes. De quantos modos diferentes o garom teve
liberdade de selecionar os salgadinhos para compor a travessa, respeitando as instrues ? R. 90
9 ) Resolva a equao 2.A n, 2 + 50 = A 2n, 2

R. 5

10) De quantos modos podemos guardar 12 bolas distintas em 4 caixas, se a primeira caixa deve conter 3 bolas, a
segunda caixa, 5 bolas, a terceira caixa, 3 bolas e a Quarta caixa, 1 bola ? R. 110.880

__________________________________________________________________________________________________________________________

ATUALIZADO AT MAIO/2012

www.CARREIRAPUBLICA.com.br

(48) 4141-3220

149

4141-3222

____________

_____________________________ MATEMTICA _________________________________ Prof. Roberto

11) Numa Kombi viajam 9 pessoas, das quais 4 podem dirigir. De quantas maneiras diferentes possvel acomodlas ( 3 no banco da frente, 3 no banco do meio e 3 no banco de trs ) de forma que uma das 4 pessoas que dirigem
ocupe o lugar na direo?R. 161.280
12) Em um campeonato de dois turnos em que devem jogar 12 equipes de futebol, qual o nmero total de jogos a
serem realizados? R. 132
19 Probabilidades
Experimento aleatrio: So experimentos que, mesmo repetidos vrias vezes sob condies semelhantes,
apresentam resultados imprevisveis.
Espao amostral ou conjunto universo: o conjunto de todos os resultados possveis de um fenmeno ou
experimento aleatrio.
Ponto amostral: qualquer elemento do espao amostral.
Evento certo: o prprio conjunto universo. Intuitivamente, o fato que ocorre sempre, com certeza.
Evento impossvel: o subconjunto vazio do conjunto universo.
Evento soma: a reunio de dois eventos.
Evento produto: a interseco de dois eventos.
Eventos mutuamente exclusivos: So dois eventos que nunca ocorrem simultaneamente.
Eventos complementares ou contrrios: So dois eventos cuja soma o prprio espao amostral.
1

- Completar, dando os espaos amostrais das seguintes experincias:


a) Distribuio dos 3 filhos de uma famlia, quanto ao sexo.
U = { .................................................................................. )
b) Lanamento de um dado e uma moeda.
U = {.................................................................................... )
c) Retirada sucessiva de duas bolas de uma urna, que contem trs bolas numeradas de 1 a 3.
U = { .................................................................................. )

19.1 Probabilidade
Se, num fenmeno aleatrio, o nmero de elementos do conjunto universo n(U) e o nmero de elementos do
evento A n(A), ento, a probabilidade de ocorrer o evento A o nmero P(A), tal que:
P(A) =

n(A) .
n(U)

=== Casos favorveis


====> Casos possveis

- No lanamento de um dado, determinar a probabilidade de se obter:


a) O nmero 5
b) Um nmero mpar
c) Um nmero menor ou igual a 5
d) Um nmero maior ou igual a 5

- No lanamento de dois dados, determinar a probabilidade de se obter:


a) Soma dos pontos igual a 8
b) Pares de pontos iguais
c) Soma de pontos igual a 4

- No lanamento simultneo de duas moedas, calcule a probabilidade de se obter:

150__________________________________________________________________________________________________________________________
ATUALIZADO AT MAIO/2012

www.CARREIRAPUBLICA.com.br

(48) 4141-3220

4141-3222

____________

_____________________________ MATEMTICA _________________________________ Prof. Roberto

a) Duas caras
b) Duas coroas
c) Uma cara e uma coroa
5

- No lanamento de um dado, calcular a probabilidade de ocorrer valor menor ou igual a 2 ou maior ou igual a 5.

- No lanamento de trs moedas, calcular a probabilidade de ocorrer:


a) Trs caras
b) Pelo menos, uma cara
c) No mximo, uma coroa
d) Coroa na primeira moeda.

- No lanamento de dois dados, calcular a probabilidade da:


a) A soma dos pontos ser mpar
b) A soma dos pontos ser maior do que 5
c) A soma dos pontos ser menor que 9
d) Ocorrncia de resultados iguais nos dois dados

8 - Qual a probabilidade de sair o s de ouros quando retiramos uma carta de um baralho de 52 cartas?
9 - Qual a probabilidade de sair um rei quando retiramos uma carta de um baralho de 52 cartas?

p = 1/52

p = 1/13

10 - Em um lote de 12 peas, 4 so defeituosas. Sendo retirada uma pea, calcule:


a. a probabilidade de essa pea ser defeituosa. p = 1/3
b. a probabilidade de essa pea no ser defeituosa. p = 2/3
11 - De dois baralhos de 52 cartas retiram-se, simultaneamente, uma carta do primeiro baralho e uma carta do
segundo. Qual a probabilidade de a carta do primeiro baralho ser um rei e a do segundo ser o 5 de paus? p =
1/676
12 - Uma urna A contm: 3 bolas brancas, 4 pretas, 2 verdes; uma urna B contm: 5 bolas brancas, 2 pretas, 1 verde;
uma urna C contm: 2 bolas brancas, 3 pretas, 4 verdes. Uma bola retirada de cada urna. Qual a
probabilidade de as trs bolas retiradas da primeira, segunda e terceira urnas serem, respectivamente, branca,
preta e verde? p = 1/27
13 - De um baralho de 52 cartas retiram-se, ao acaso, duas cartas sem reposio. Qual a probabilidade de a primeira
carta ser o s de paus e a segunda ser o rei de paus? p = 1/2652
14 - Qual a probabilidade de sair uma figura quando retiramos uma carta de um baralho de 52 cartas? p = 3/13
15 - Qual a probabilidade de sair uma carta de copas ou de ouros quando retiramos uma carta de um baralho de 52
cartas? p = 1/2
16 - No lanamento de um dado, qual a probabilidade de se obter um nmero no-inferior a 5?

p = 1/3

17 - So dados dois baralhos de 52 cartas. Tiramos, ao mesmo tempo, uma carta do primeiro baralho e uma carta do
segundo. Qual a probabilidade de tirarmos uma dama e um rei, no necessariamente nessa ordem? p = 2/169
18 - Num clube desportivo 30 meninos praticam futebol. Doze treinam para o ataque, quinze para a defesa e cinco
para guarda-redes. Qual a probabilidade de escolhendo um desportista ao acaso ele treinar para a defesa e o
ataque? p = 1/15

19.2 Problemas resolvidos


1 Uma moeda viciada, de forma que as caras so trs vezes mais provveis de aparecer do que as coroas.
Determine a probabilidade de num lanamento sair coroa.
__________________________________________________________________________________________________________________________

ATUALIZADO AT MAIO/2012

www.CARREIRAPUBLICA.com.br

(48) 4141-3220

151

4141-3222

____________

_____________________________ MATEMTICA _________________________________ Prof. Roberto

Soluo:
Seja k a probabilidade de sair coroa. Pelo enunciado, a probabilidade de sair cara igual a 3k.
A soma destas probabilidades tem de ser igual a 1.
Logo, k + 3k = 1 \ k = 1/4.
Portanto, a resposta 1/4 = 0,25 = 25%.
2 Uma moeda viciada, de forma que as coroas so cinco vezes mais provveis de aparecer do que as caras.
Determine a probabilidade de num lanamento sair coroa.
Resposta: 5/6 = 83,33%
3 Trs estudantes A, B e C esto em uma competio de natao. A e B tm as mesmas chances de vencer e,
cada um, tem duas vezes mais chances de vencer do que C.
Pede-se calcular a probabilidades de A ou C vencer.
Soluo:
Sejam p(A), p(B) e p(C), as probabilidades individuais de A, B, C, vencerem. Pelos dados do enunciado, temos:
p(A) = p(B) = 2.p(C).
Seja p(A) = k. Ento, p(B) = k e p(C) = k/2.
Temos: p(A) + p(B) + p(C) = 1.
Isto explicado pelo fato de que a probabilidade de A vencer ou B vencer ou C vencer igual a 1. (evento certo).
Assim, substituindo, vem:
k + k + k/2 = 1 \ k = 2/5.
Portanto, p(A) = k = 2/5, p(B) = 2/5 e p(C) = 2/10 = 1/5.
A probabilidade de A ou C vencer ser a soma dessas probabilidades, ou seja 2/5 + 1/5 = 3/5.
4 Uma moeda viciada, de maneira que as CARAS so trs vezes mais provveis de aparecer do que as
COROAS. Calcule as probabilidades de num lanamento sair COROA.
Resp: 1/4.
5 Um dado viciado, de modo que cada nmero par tem duas vezes mais chances de aparecer num lanamento,
que qualquer nmero mpar. Determine a probabilidade de num lanamento aparecer um nmero primo.
Soluo:
Pelo enunciado, podemos escrever:
p(2) = p(4) = p(6) = 2.p(1) = 2.p(3) = 2.p(5).
Seja p(2) = k. Poderemos escrever:
p(2) + p(4) + p(6) + p(1) + p(3) + p(5) = 1, ou seja: a soma das probabilidades dos eventos elementares igual a 1.
Ento, substituindo, vem:
k + k + k + k/2 + k/2 + k/2 = 1 \ k = 2/9.
Assim, temos:
p(2) = p(4) = p(6) = 2/9
p(1) = p(3) = p(5) = 2/18 = 1/9.
O evento sair nmero primo corresponde a sair o 2, ou o 3 ou o 5. Logo,
p(2) + p(3) + p(5) = 2/9 + 1/9 + 1/9 = 4/9.
6 Use o mesmo enunciado anterior e determine a probabilidade de num nico lanamento sair um nmero mpar.

152__________________________________________________________________________________________________________________________
ATUALIZADO AT MAIO/2012

www.CARREIRAPUBLICA.com.br

(48) 4141-3220

4141-3222

____________

_____________________________ MATEMTICA _________________________________ Prof. Roberto

Resp: 1/3
7 Um carto retirado aleatoriamente de um conjunto de 50 cartes numerados de 1 a 50. Determine a
probabilidade do carto retirado ser de um nmero primo.
Soluo:
Os nmeros primos de 1 a 50 so: 2, 3, 5, 7, 11, 13, 17, 19, 23, 29, 31, 37, 41, 43 e 47, portanto, 15 nmeros primos.
Temos, portanto, 15 chances de escolher um nmero primo num total de 50 possibilidades. Portanto, a probabilidade
pedida ser igual a
p = 15/50 = 3/10.
8 - Use o mesmo enunciado anterior e determine a probabilidade de numa nica retirada, sair um carto com um
nmero divisvel por 5.
Resp: 1/5.
9 Das 10 alunas de uma classe, 3 tem olhos azuis. Se duas delas so escolhidas ao acaso, qual a probabilidade
de ambas terem os olhos azuis?
Soluo:
Existem C10,2 possibilidades de se escolher duas pessoas entre 10 e, existem C3,2 possibilidades de escolher duas
alunas de olhos azuis entre as trs. Logo, a probabilidade procurada ser igual a:
P = C3,2 / C10,2 = (3.2/2.1)/(10.9/2.1) = 6/90 = 3/45 = 1/15.
Comentrios sobre o clculo de Cn,p.
Como j sabemos da Anlise Combinatria ,

Esta a forma tradicional de se calcular Cn,p.


Na prtica, entretanto, podemos recorrer ao seguinte expediente: Cn,p possui sempre p fatores no numerador a partir
de n, decrescendo uma unidade a cada fator e p fatores no denominador a partir de p, decrescendo uma unidade a
cada fator.
Exemplos:
C10,4 = (10.9.8.7)/(4.3.2.1) = 210.
C8,3 = (8.7.6)/(3.2.1) = 56.
C16,3 = (16.15.14)/(3.2.1) = 560.
C12,4 = (12.11.10.9)/(4.3.2.1) = 495.
C10,5 = (10.9.8.7.6)/(5.4.3.2.1) = 252.
10 Considere o mesmo enunciado da questo anterior e calcule a probabilidade de na escolha de duas alunas,
nenhuma ter olhos azuis.
Resp: 7/15.
__________________________________________________________________________________________________________________________

ATUALIZADO AT MAIO/2012

www.CARREIRAPUBLICA.com.br

(48) 4141-3220

153

4141-3222

____________

_____________________________ MATEMTICA _________________________________ Prof. Roberto

Dica: como nenhuma das alunas deve ter olhos azuis, restam
10 3 = 7 alunas. Portanto, ...
11 - ESAF) Num sorteio concorreram 50 bilhetes com nmeros de 1 a 50. Sabe-se que o bilhete sorteado mltiplo
de 5. A probabilidade de o nmero sorteado ser 25 :
a) 15%

b) 5%

c) 10%

d) 30%

e) 20%

Soluo: Como os nmeros mltiplos de cinco (5, 10, 15, ..., 50) formam uma PA de razo r = 5, a1 = 5 e an = 50,
temos que: 50 = 5 + 5(n-1). Ento n = 45/5 + 1 = 9 + 1 = 10. Sendo 1 o nmero de casos favorveis e 10 o nmero de
casos possveis, segue que a probabilidade procurada 1/10 = 0,1 = 10% (resp c).
12 - Um casal decidiu que vai ter 5 filhos. Qual seria a probabilidade de que tivesse pelo menos 2 meninos?
Soluo: A ao constiuda de 5 etapas. Para cada etapa existem 2 possibilidades (menino ou menina). Pelo PFC,
o nmero de casos possveis 22222 = 32.
O nmero de casos favorveis o nmero de maneiras de ter pelo menos 2 meninos (dois meninos ou mais), ou
seja, de ter 2 meninos e 3 meninas, ou, 3 meninos e 2 meninas, ou, 4 meninos e 1 menina, ou, 5 meninos e 0
meninas. Observe que estamos contando permutaes com repeties, ento o nmero de casos favorveis :
5!/(2!3!) + 5!/(3!2!) + 5!/(4!1!) + 5!/(5!0!) = 10 + 10 + 5 + 1 = 26
Logo, a probabilidade de que tivesse pelo menos 2 meninos P = 26/32 = 13/16 = 81,25%

19.3 Propriedades
P( A - B ) = P( A ) P( A B )

A
8

- Fez-se uma pesquisa entre um grupo de pessoas e chegou-se concluso de que o conjunto A das pessoas
que falam francs tem 40 elementos, o conjunto B das pessoas que falam ingls tem 70 elementos e o conjunto
A B tem 100 elementos. Qual a probabilidade de que, escolhida ao acaso uma dessas 100 pessoas, ela fale
francs mas no fale ingls.
P(AB)=P(A)+P(B)P(AB)
A

- Considere o lanamento de um dado. Calcular a probabilidade de que o resultado seja par ou mltiplo de 3.

10 - Uma moeda ser lanada duas vezes. Calcular a probabilidade de que:


a) No ocorra cara nenhuma vez.
b) Obtenha-se cara na primeira ou na segunda jogada.

154__________________________________________________________________________________________________________________________
ATUALIZADO AT MAIO/2012

www.CARREIRAPUBLICA.com.br

(48) 4141-3220

4141-3222

____________

_____________________________ MATEMTICA _________________________________ Prof. Roberto

19 .4 PROBABILIDADE CONDICIONAL
Se A e B so eventos de um espao amostral U, com P (B ) 0, ento a probabilidade condicional do evento A, tendo
ocorrido o evento B, indicada por P ( A?B ) e definida pela razo
P( A/B ) = P ( A B ) .
P(B)
11 - Numa escola com 100 alunos, 40 estudam s Biologia, 30 estudam s Alemo e 20 estudam Biologia e Alemo.
Qual a probabilidade de um aluno que j estuda Biologia, estudar tambm Alemo ?
12 - Sorteando ao acaso um nmero do conjunto U = { 1, 2, 3, 4, 5, 6, 7, 8, 9, 10 }, qual a probabilidade dele ser um
mltiplo de 3, com a condio de ser mpar ?
19.4.1 Propriedades
A probabilidade do evento-produto igual ao produto de P ( A ) por P ( B/A ) a probabilidade condicional do evento
B, tendo ocorrido o evento
P ( A B ) = P ( A ) . P ( B/A )
Se A e BV forem eventos independentes, temos P ( B /A ) = P ( B ) e, portanto
P(AB) =P(A).P(B)
13 - Considere uma famlia com 3 filhos e os seguintes eventos
A: 2 so meninos.
B: O mais velho menino
a) Qual a probabilidade de A, visto que ocorreu B?
b) A e B so eventos independentes?
14 - Uma urna contm 10 bolas brancas, 15 pretas e 5 vermelhas. Duas bolas so retiradas sucessivamente. Qual a
probabilidade de que a primeira bola seja branca e a segunda, preta? ( Considerar o problema com ou sem
reposio da primeira bola, antes da retirada da segunda).

20 Geometria
20.1 Teorema de Pitgoras e relaes mtricas no tringulo retngulo
Em todo tringulo retngulo, o quadrado da medida da hipotenusa igual soma do quadrado das medidas
dos catetos.
a = Hipotenusa

b = cateto

c = cateto

m = projeo do cateto b

n = projeo do cateto c

h = altura

c
h

__________________________________________________________________________________________________________________________

ATUALIZADO AT MAIO/2012

www.CARREIRAPUBLICA.com.br

(48) 4141-3220

155

4141-3222

____________

_____________________________ MATEMTICA _________________________________ Prof. Roberto

a = b + c
Relaes importantes no tringulo retngulo
h = m . n

b.c=a.h

b = a . m

c = a . n

a=m+n

20.1.1 Exerccios
1- Calcule a altura de um tringulo eqiltero cujo lado mede 8 cm. R: 43 cm
2- Quanto mede a diagonal de um quadrado cujo lado mede 52 cm? R: 10 cm
3- Qual a rea de um tringulo eqiltero cujo lado mede 3 cm ? R: 93 cm
4
4- A diagonal de um quadrado mede 15 cm. Qual a sua rea ? R: 112,5 cm
5- O permetro de um quadrado mede 28 cm. Quanto mede sua diagonal ? R: 72 cm
6- Em um tringulo eqiltero, a altura mede 23 cm. Qual o permetro? R: 12 cm
7- Num tringulo retngulo, a hipotenusa mede 26 cm e a diferena entre os catetos 14 cm. Determine a medida
dos catetos.
8- Num tringulo retngulo, a hipotenusa excede de 3 unidades um dos catetos. Sabendo que a razo dos catetos
5 : 12, determine seus lados.
9- Num tringulo retngulo, sabe-se que a soma dos catetos vale 7k e a rea, 6k + k 2.
a) Determine a hipotenusa em funo de k.
b) Quanto vale a hipotenusa quando k = 2 ?
10- Num tringulo retngulo, a hipotenusa a mede 25 cm e a altura relativa hipotenusa mede 12 cm. Calcule os
elementos b, c, m e n.
11- Determine os lados a, b e c de um tringulo retngulo ABC, sabendo que b + c = 7 dm e h + AD mede 2,4 dm.
12- Num tringulo retngulo ABC, o cateto menor b = 35 cm. A diferena entre as projees m e n dos catetos
sobre a hipotenusa 1. Calcule a medida da hipotenusa:
13- Duas torres, de 13 m e 37 metros de altura, distam 30 m uma da outra. Qual a distncia entre seus extremos?
(As torres se localizam num terreno plano). R: 641 m
14- Determine os catetos de um tringulo retngulo, sabendo que a razo de suas medidas 1 : 2 e que a
hipotenusa mede 85 cm. R: 8 cm e 16 cm
15- Num tringulo retngulo ABC, a diferena entre os catetos 2 cm e o seu produto 48 cm. Determine:
a) a hipotenusa. R: a = 10 cm
b) A altura relativa hipotenusa. R: h = 4,8 cm
c) As projees dos catetos sobre a hipotenusa. R: m = 6,4 cm e n = 3,6 cm
16- Num tringulo retngulo, a altura relativa hipotenusa mede 12,5 cm e a divide em dois segmentos cuja
diferena 11,5 cm. Determine seus lados. R: 14,98 cm, 23,30 cm e 27,7 cm
17- Calcule a rea de um tringulo retngulo, sabendo que um de seus catetos mede o triplo do outro e seu
permetro vale 8 + 210 unidades. R: 6 unidades de rea
18- Num tringulo retngulo ABC, o cateto b mede 12 cm e a projeo do cateto c sobre a hipotenusa vale 25/ 13
cm. Calcule a, c, h e n. R: a = 13 cm, c = 5 cm, h = 60 cm e n = 144
13
13
19- Num tringulo retngulo de rea s, a diferena dos catetos d.
a) Calcule a hipotenusa em funes d e s. R: d + 4s
b) Quanto vale a hipotenusa quando d = 2 e s = 8 ? R: 6
20- Num tringulo retngulo ABC, sabe-se que a rea vale 2s e que a razo entre os catetos b/c = k. Calcule seus
lados. R: 2 s(k+1) ; 2ks ; 2s
k
k
21- Determine a hipotenusa de um tringulo retngulo, dadas a soma de seus catetos 5k + 5 e sua rea 3k + 8k 3.
R: 13k+18k+37
22- Calcule a rea de um tringulo retngulo, sabendo que um de seus catetos o triplo do outro e que seu
permetro p. R: p(13-410)
12

156__________________________________________________________________________________________________________________________
ATUALIZADO AT MAIO/2012

www.CARREIRAPUBLICA.com.br

(48) 4141-3220

4141-3222

____________

20.2

_____________________________ MATEMTICA _________________________________ Prof. Roberto

reas de figuras planas e polgonos inscritos numa circunferncia


Retngulo
Quadrado

Tringulo

Paralelogramo

Losango

Trapzio

Tringulo equiltero

Crculo

__________________________________________________________________________________________________________________________

ATUALIZADO AT MAIO/2012

www.CARREIRAPUBLICA.com.br

(48) 4141-3220

157

4141-3222

____________

_____________________________ MATEMTICA _________________________________ Prof. Roberto

Hexgono regular
R = raio
a = aptema
l = lado
l=R
a = R 3
2
Quadrado
l = R 2
a = R2/2

Tringulo

l = R 3

a = l/2

a = R/2

20.2.1 Exerccios
1 - O lado de um quadrado mede 10 cm. O crculo circunscrito ao quadrado est inscrito num tringulo equiltero.
Calcule o lado do tringulo. R. 10 \/ 6 cm

158__________________________________________________________________________________________________________________________
ATUALIZADO AT MAIO/2012

www.CARREIRAPUBLICA.com.br

(48) 4141-3220

4141-3222

____________

_____________________________ MATEMTICA _________________________________ Prof. Roberto

2 - A diferena entre as medidas do lado de um tringulo equiltero e de um hexgono regular inscritos no mesmo
crculo 0,4 cm. O raio do crculo mede
a) 10 cm b) 0,546 cm c) 54 cm d) N.R.A.
3

- O lado do hexgono regular circunscrito mede 2,5 cm. O aptema do quadrado inscrito no mesmo crculo mede
a) 2,5 cm b) 25 cm c) 1,53 cm d) N.R.A.

- A soma das medidas dos aptemas de um tringulo equiltero e de um quadrado inscritos no mesmo crculo
5 cm. O raio do crculo mede
a) 4,142 cm b) 4 cm c) 4,3 cm d) N.R.A.

- Escolha o equacionamento adequado para a resoluo do problema abaixo:


Quais as dimenses de um retngulo que tem 28,4 m de permetro e 49,6 m de rea. (B.B.)
a) x + y = 14,2 b)x + y = 28,4 c)x + y = 14,2 d)2x + 2y = 28,4 e) x + y = 14,2
2xy = 49,6
xy = 49,6
x + y = 49,6 xy = 49,6
xy = 49,6

- Os lados de um tringulo equiltero tem medida l . Determine a altura em funo do lado.

- Calcule a hipotenusa de um tringulo retngulo issceles, cujo permetro 4 m .

R.4(2 -1)m

8 - Nos dois terrenos vizinhos da figura, AB = 30 m, BC = 40 m e AD = 20 m. O terreno ABC k vezes maior que o
ACD. O valor de k : (Fiscal do ICM RS)
a) 1,2
b) 1,5 c) 1,8 d) 2 e) 3

C
D

9 - A base de um tringulo mede 1 palito mais 3 cm e a sua altura 1 palito menos 2 cm. Sabendo-se que a sua rea
de 12 cm, quantos centmetros mede a sua base ? ( B.B.)
a) 6 b) 8 c) 10 d) 12 e) 14
10 - O permetro de um losango mede 52 cm e uma de suas diagonais mede 24 cm. A rea desse losango, em
centmetros quadrados, mede:
a) 120 b) 96 c) 108 d) 90 e) 100 ( DASP Ag. Adm.)
11 - As diagonais de um losango medem 16 m e 12 m. Ento, a distncia entre dois lados paralelos, medido em
centmetros :
a) 1000 b) 1920 c) 960 d) 1200 (Fiscal de Mercadorias em Trnsito SC)
12 - Num losango de 28 cm de lado, a distncia entre dois lados paralelos 19 cm. A rea desse losango, em
centmetros quadrados :
a) 266 b) 532 c) 846 d) 984 ( DASP AG. Adm.)
13. Calcular a rea de um losango, sabendo que sua diagonal maior mede 5 cm e a diagonal menor mede 2,4 cm.
2
R: 6 cm
14.Sabendo que a base maior de um trapzio mede 12 cm, base menor mede 3,4 cm e sua altura mede 5 cm.
Calcule a rea deste trapzio.
2
R: 38,5 cm
15.PRF-1998) Para ladrilhar o piso retangular de um salo de 6,30m por 8,10m, uma pessoa comprou ladrilhos
quadrados, de 30cm de lado. Calcule o nmero necessrio (aproximado) de material, considerando que
aconselhvel 20% a mais de peas de ladrilhos, por causa das eventuais quebras (estragos).
A alternativa correta :
a) 600 b) 6800 c) 680 X d) 6000

__________________________________________________________________________________________________________________________

ATUALIZADO AT MAIO/2012

www.CARREIRAPUBLICA.com.br

(48) 4141-3220

159

4141-3222

____________

_____________________________ MATEMTICA _________________________________ Prof. Roberto

16.(SOLDADO-PM) Uma pessoa deseja cercar um terreno que tem a forma de um tringulo retngulo, utilizando 8
fios em torno dele. Sabendo-se que seus lados perpendiculares medem 15m e 20m, a quantidade de fio, em metros,
que ser utilizado, :
a) 480 b) 400 c) 440 d) 560 X
17.( CEFET-SC ) As cidades de Quito e Cingapura se encontram prximo da linha do Equador e em pontos
diametralmente opostos no globo terrestre. Considerando o raio terrestre igual a 6370 Km, pode-se afirmar que um
avio saindo de Quito, voando em mdia com velocidade de 870 Km/h, leva para chegar em Cingapura,
aproximadamente: ( no considerando as escalas )
a) 20 horas b) 16 horas c) 23 horas X d) 32 horas
18.( TCNICO-INSS)
x+3
x+1

A rea da regio mostrada acima de 15 cm Considerando que as medidas indicadas na figura esto em cm, podese afirmar que o permetro do retngulo, em centmetros, de:
a) 16 X b) 14 c) 10 d) 12 e) 8
19.(FEPESE -2006) Na figura, a regio sombreada formada por diversos tringulos e por um quadrado. As medidas
2
esto indicadas em centmetros. Nessas condies, a rea da regio no sombreada, em cm , igual a:

a) 20 X b) 16 c) 12 d) 8
20.(FEPESE -2006) Um indivduo pretende colocar uma cerca em torno de seu terreno, que possui rea total de 364
2
m . O terreno tem o formato de um retngulo, sendo que um dos lados mede 13 metros. Assinale a alternativa que
indique o comprimento total da cerca que ele dever comprar para cercar o terreno.
a) 52 metros. b) 54 metros. c) 26 metros. d) 69 metros. e) 82 metros.
Resposta: e
2

21.(PRF-1998) Um tringulo tem 0,675 m de rea e sua altura corresponde a 3/5 da base. A altura do tringulo, em
decmetros, igual a:
a) 0,9 b) 1,5 c) 9,0 X d) 15,0 e) 24,0
22.(ACAFE) Com uma corda de 23 metros contorna-se um canteiro em forma de trapzio issceles, cuja base maior
3 unidades maior que a menor e os lados oblquos tm medidas iguais base menor. Se com essa mesma corda
se contorna um retngulo com uma das dimenses igual da base menor do trapzio, pode-se afirmar que a razo
entre a rea do retngulo e a rea do trapzio :
R: 12,5
23.(ACAFE) A base de um tringulo mede 72cm e sua altura, em cm, h. Se a base for aumentada em 48cm e a
altura em 32cm, obtm-se um novo tringulo, cuja rea o triplo da rea do primeiro. O valor da altura h, em cm, :
a)12. b)64. c)80. d)20. e)40. X

160__________________________________________________________________________________________________________________________
ATUALIZADO AT MAIO/2012

www.CARREIRAPUBLICA.com.br

(48) 4141-3220

4141-3222

____________

_____________________________ MATEMTICA _________________________________ Prof. Roberto

24.(FEPESE -2006) Em um terreno retangular cujos lados medem 12 metros e 30 metros, foi construda uma casa
que ocupa 88 metros quadrados do terreno. O proprietrio est projetando a construo de uma piscina com a forma
de um trapzio, cujas medidas esto na figura a seguir:

rea do Trapzio =

(Base maior + Base menor ) x h

2
Entretanto, antes de construir a piscina, o proprietrio ficou preocupado com a rea livre que
ainda teria no terreno. Se considerarmos que a rea livre ser dada pela rea total do terreno,
excluindo-se a rea que a casa e a piscina ocupam, assinale a alternativa que representa a
medida da rea livre do terreno.
a) 251 m b) 257 m X c) 260 m d) 263 m

e) 272 m

25.(FEPESE-2006) Assinale a afirmativa correta relacionada com as figuras geomtricas que seguem:

a) A soma dos permetros igual a 24 centmetros.


b) A rea do quadrado igual rea do retngulo.
c) O permetro do quadrado a metade do permetro do retngulo.
d) A soma das reas igual a 12 centmetros quadrados. X
e) A rea do retngulo igual a: 2 cm + 2 cm + 4 cm + 4 cm = 12 cm.
26.(ACAFE) Numa madeireira esto empilhadas 75 tbuas, umas de 2cm de espessura e outras de 3cm. A pilha tem
1,80m de altura. Com base nessas informaes, correto afirmar que:
a) O nmero de tbuas de 2cm o dobro do nmero de tbuas de 3cm.
b) A altura da pilha que se pode obter somente com tbuas de 2cm 90cm. X
c) A diferena entre o nmero de tbuas de cada espessura 30.
d) A altura da pilha obtida somente com tbuas de 3cm 60cm.
e) Os nmeros que representam as quantidades das tbuas de cada espessura so mltiplos de 10.

21 Relaes trigonomtricas no tringulo retngulo


21.1 Razes trigonomtricas
Catetos e Hipotenusa

Em um tringulo chamamos o lado oposto ao ngulo reto de hipotenusa e os lados


adjacentes de catetos.
Observe a figura:
__________________________________________________________________________________________________________________________

ATUALIZADO AT MAIO/2012

www.CARREIRAPUBLICA.com.br

(48) 4141-3220

161

4141-3222

____________

_____________________________ MATEMTICA _________________________________ Prof. Roberto

Hipotenusa:

Catetos:

Seno, Cosseno e Tangente


Considere um tringulo retngulo BAC:

Hipotenusa:

, m(

) = a.

Catetos:

ngulos:

, m(

) = b.

, m(

) = c.

Tomando por base os elementos desse tringulo, podemos definir as seguintes razes
trigonomtricas:
Seno de um ngulo agudo a razo entre a medida do cateto oposto a esse ngulo e a medida da
hipotenusa.

Assim:

162__________________________________________________________________________________________________________________________
ATUALIZADO AT MAIO/2012

www.CARREIRAPUBLICA.com.br

(48) 4141-3220

4141-3222

____________

_____________________________ MATEMTICA _________________________________ Prof. Roberto

Cosseno de um ngulo agudo a razo entre a medida do cateto adjacente a esse ngulo e a medida da
hipotenusa.

Assim:

Tangente
Tangente de um ngulo agudo a razo entre a medida do cateto oposto e a medida do
cateto adjacente a esse ngulo.

Assim:

__________________________________________________________________________________________________________________________

ATUALIZADO AT MAIO/2012

www.CARREIRAPUBLICA.com.br

(48) 4141-3220

163

4141-3222

____________

_____________________________ MATEMTICA _________________________________ Prof. Roberto

Exemplo:

Observaes:
1. A tangente de um ngulo agudo pode ser definida como a razo entre seno deste
ngulo e o seu cosseno.
Assim:

2. A tangente de um ngulo agudo um nmero real positivo.


164__________________________________________________________________________________________________________________________
ATUALIZADO AT MAIO/2012

www.CARREIRAPUBLICA.com.br

(48) 4141-3220

4141-3222

____________

_____________________________ MATEMTICA _________________________________ Prof. Roberto

3. O seno e o cosseno de um ngulo agudo so sempre nmeros reais positivos


menores que 1, pois qualquer cateto sempre menor que a hipotenusa.

As razes trigonomtricas de 30, 45 e 60


Considere as figuras:

quadrado de lado l e diagonal

Tringulo eqiltero
de lado I e altura

Seno, cosseno e tangente de 30


Aplicando as definies de seno, cosseno e tangente para os ngulos de 30, temos:

Seno, cosseno e tangente de 45


Aplicando as definies de seno, cosseno e tangente para um ngulo de 45, temos:

__________________________________________________________________________________________________________________________

ATUALIZADO AT MAIO/2012

www.CARREIRAPUBLICA.com.br

(48) 4141-3220

165

4141-3222

____________

_____________________________ MATEMTICA _________________________________ Prof. Roberto

Seno, cosseno e tangente de 60


Aplicando as definies
de seno, cosseno e tangente para
um ngulo de 60,

temos:
Resumindo
x

sen x

cos x

tg x

30
45
60

EXEMPLOS
1-) Vamos calcular o sen, o cos e a tg dos dois ngulos agudos do tringulo abaixo:

Resoluo:
sen = 3/5 ; sen = 4/5
cos = 4/5 ; cos = 3/5
tg = 3/4 ; tg = 4/3

2-) Com o auxlio da tabela trigonomtrica, vamos calcular o valor do lado X no


tringulo retngulo dado:

166__________________________________________________________________________________________________________________________
ATUALIZADO AT MAIO/2012

www.CARREIRAPUBLICA.com.br

(48) 4141-3220

4141-3222

____________

_____________________________ MATEMTICA _________________________________ Prof. Roberto

Resoluo:
cos 40o = X/10
X = 10 . cos 40o
X = 10 . 0,766
X = 7,66
21.2 Exerccios
1

- um caminho sobe uma rampa que forma um ngulo de 30 com a horizontal. Aps ter andado 5 km, qual a
altura em relao a horizontal ? R. 2,5 km

- Calcule a altura de um tringulo equiltero de lado igual a 7 m, sabendo que sen 60 = 3/2 .

3 - O comprimento de uma escada de bombeiros 20 m. A base da escada est a 2 m do cho, sobre um caminho.
A que altura do cho est o topo da escada quando ela forma um ngulo de 70 com a horizontal? sen 70 = 0,940
R. 20,8 m
4- Para determinar a distncia entre os pontos B e C de um lago, um agrimensor determinou as medidas AC = 400 m
e = 55. Determine BC na figura. Sen 55 = 0,819 R. 327,6 m
C

55
A

5- Um avio est a 7.000 m de altura em relao pista de um aeroporto. O piloto tem ordens de aterrisar sob
um ngulo constante de 17 em relao horizontal. A que distncia da pista o piloto deve iniciar a sua descida ?
sen 17 = 0,306
6- No tringulo retngulo issceles, a tangente de um dos ngulos agudos :
a) 1/2
b) 2
c) 1
d) 1/2
e) 3/2
7- Duas pessoas observam um objeto sob ngulos de 30 e 60 respectivamente. Se a distncia entre os
observadores de 100 m, calcular as distncias entre os observadores e o objeto e a altura do objeto em relao a
horizontal.
Exerccios complementares
1) Uma pessoa est distante 80m da base de um prdio e v um ponto mais alto do prdio sob um ngulo de 16 em
relao horizontal. Qual a altura do prdio?
2) Um avio levanta vo em B, e sobe fazendo um ngulo constante de 15 com a horizontal. A que altura estar e
qual a distncia percorrida quando passar pela vertical que passa por uma igreja situada a 2km do ponto de partida?
3) Uma torre vertical de altura 12m vista sob um ngulo de 30 por uma pessoa que se
encontra a uma distncia x da sua base e cujos olhos esto no mesmo plano horizontal dessa base. Determina a
distncia x.
4) Dois observadores A e B vem um balo, respectivamente, sob ngulos visuais de 20 e 40. Sabendo que a
distncia entre A e B de 200m, calcula a altura do balo. Obs.: os observadores encontram-se do mesmo lado em
relao ao balo.
5) Num exerccio de tiro, o alvo se encontra numa parede cuja base est situada a 82m do atirador. Sabendo que o
atirador v o alvo sob um ngulo de 12 em relao horizontal, calcula a que distncia do cho est o alvo.
__________________________________________________________________________________________________________________________

ATUALIZADO AT MAIO/2012

www.CARREIRAPUBLICA.com.br

(48) 4141-3220

167

4141-3222

____________

_____________________________ MATEMTICA _________________________________ Prof. Roberto

6) A partir de um ponto, observa-se o topo de um prdio sob um ngulo de 30. Caminhando 23m em direo ao
prdio, atingimos outro ponto, onde se v o topo do prdio segundo um ngulo de 60. Desprezando a altura do
observador, calcula, em metros, a altura do prdio.
7) Queremos encostar uma escada de 8m de comprimento numa parede, de modo que forme um ngulo de 60 com
o solo. A que distncia da parede devemos apoiar a escada no solo?
8) Um avio levanta vo sob um ngulo de 30. Quando tiver percorrido meio quilmetro, a que altura estar do solo?
9) Um observador em A v uma torre vertical CD sob um ngulo de 30 e caminhados 40m em direo a torre passa
a v-la sob 40. Sabendo que a altura do observador 1,70m, calcula a altura da torre e a que distncia ela se
encontra do observador.
10) Um mergulhador percorreu uma distncia de 40m, entre a superfcie e o fundo do mar, segundo uma trajetria
retilnea que forma um ngulo de 50 com a superfcie.
a) Qual , aproximadamente, a profundidade do local alcanado pelo mergulhador?
b) Subindo verticalmente para a superfcie, a que distncia do ponto em que mergulhou ele sair aproximadamente?
Respostas
1) h = 22,93 m (sem levar em conta a altura da pessoa).
2) h = 0,53589 km = 535,89 m d = 2,07055 km = 2070,55 m
3) x = 20,78 m
4) h = 128,56 m
5) d = 17,43 m
6) h = 19,92 m
7) d = 4 m
8) h = 0,25 km = 250 m
9) h = 75,73 m d = 128,23 m
10) a) h = 30,64 m b) x = 25,71 m

TABELA TRIGONOMTRICA
Podemos tabular os valores trigonomtricos dos ngulos agudos, isto , ngulos entre 1o e 89o.
Abaixo temos a tabela:
ngulo

sen

cos

tg

0,017452

0,999848

0,017455

0,034899

0,999391

0,034921

0,052336

0,99863

0,052408

0,069756

0,997564

0,069927

0,087156

0,996195

0,087489

0,104528

0,994522

0,105104

0,121869

0,992546

0,122785

0,139173

0,990268

0,140541

0,156434

0,987688

0,158384

10

0,173648

0,984808

0,176327

11

0,190809

0,981627

0,19438

12

0,207912

0,978148

0,212557

13

0,224951

0,97437

0,230868

14

0,241922

0,970296

0,249328

15

0,258819

0,965926

0,267949

16

0,275637

0,961262

0,286745

168__________________________________________________________________________________________________________________________
ATUALIZADO AT MAIO/2012

www.CARREIRAPUBLICA.com.br

(48) 4141-3220

4141-3222

____________

_____________________________ MATEMTICA _________________________________ Prof. Roberto

17

0,292372

0,956305

0,305731

18

0,309017

0,951057

0,32492

19

0,325568

0,945519

0,344328

20

0,34202

0,939693

0,36397

21

0,358368

0,93358

0,383864

22

0,374607

0,927184

0,404026

23

0,390731

0,920505

0,424475

24

0,406737

0,913545

0,445229

25

0,422618

0,906308

0,466308

26

0,438371

0,898794

0,487733

27

0,45399

0,891007

0,509525

28

0,469472

0,882948

0,531709

29

0,48481

0,87462

0,554309

30

0,5

0,866025

0,57735

31

0,515038

0,857167

0,600861

32

0,529919

0,848048

0,624869

33

0,544639

0,838671

0,649408

34

0,559193

0,829038

0,674509

35

0,573576

0,819152

0,700208

36

0,587785

0,809017

0,726543

37

0,601815

0,798636

0,753554

38

0,615661

0,788011

0,781286

39

0,62932

0,777146

0,809784

40

0,642788

0,766044

0,8391

41

0,656059

0,75471

0,869287

42

0,669131

0,743145

0,900404

43

0,681998

0,731354

0,932515

44

0,694658

0,71934

0,965689

45

0,707107

0,707107

46

0,71934

0,694658

1,03553

47

0,731354

0,681998

1,072369

48

0,743145

0,669131

1,110613

49

0,75471

0,656059

1,150368

50

0,766044

0,642788

1,191754

51

0,777146

0,62932

1,234897

52

0,788011

0,615661

1,279942

53

0,798636

0,601815

1,327045

54

0,809017

0,587785

1,376382

55

0,819152

0,573576

1,428148

56

0,829038

0,559193

1,482561

57

0,838671

0,544639

1,539865

58

0,848048

0,529919

1,600335

__________________________________________________________________________________________________________________________

ATUALIZADO AT MAIO/2012

www.CARREIRAPUBLICA.com.br

(48) 4141-3220

169

4141-3222

____________

_____________________________ MATEMTICA _________________________________ Prof. Roberto

59

0,857167

0,515038

1,664279

60

0,866025

0,5

1,732051

61

0,87462

0,48481

1,804048

62

0,882948

0,469472

1,880726

63

0,891007

0,45399

1,962611

64

0,898794

0,438371

2,050304

65

0,906308

0,422618

2,144507

66

0,913545

0,406737

2,246037

67

0,920505

0,390731

2,355852

68

0,927184

0,374607

2,475087

69

0,93358

0,358368

2,605089

70

0,939693

0,34202

2,747477

71

0,945519

0,325568

2,904211

72

0,951057

0,309017

3,077684

73

0,956305

0,292372

3,270853

74

0,961262

0,275637

3,487414

75

0,965926

0,258819

3,732051

76

0,970296

0,241922

4,010781

77

0,97437

0,224951

4,331476

78

0,978148

0,207912

4,70463

79

0,981627

0,190809

5,144554

80

0,984808

0,173648

5,671282

81

0,987688

0,156434

6,313752

82

0,990268

0,139173

7,11537

83

0,992546

0,121869

8,144346

84

0,994522

0,104528

9,514364

85

0,996195

0,087156

11,43005

86

0,997564

0,069756

14,30067

87

0,99863

0,052336

19,08114

88

0,999391

0,034899

28,63625

89

0,999848

0,017452

57,28996

22 Juros simples
22.1 Conceitos bsicos
A Matemtica Financeira uma ferramenta til na anlise de algumas alternativas de investimentos ou
financiamentos de bens de consumo. Consiste em empregar procedimentos matemticos para simplificar a operao
financeira a um Fluxo de Caixa.
Capital
O Capital o valor aplicado atravs de alguma operao financeira. Tambm conhecido como: Principal,
Valor Atual, Valor Presente ou Valor Aplicado. Em ingls usa-se Present Value (indicado pela tecla PV nas
calculadoras financeiras).
Juros
Juros representam a remunerao do Capital empregado em alguma atividade produtiva. Os juros podem ser
capitalizados segundo dois regimes: simples ou compostos.

170__________________________________________________________________________________________________________________________
ATUALIZADO AT MAIO/2012

www.CARREIRAPUBLICA.com.br

(48) 4141-3220

4141-3222

____________

_____________________________ MATEMTICA _________________________________ Prof. Roberto

JUROS SIMPLES: o juro de cada intervalo de tempo sempre calculado sobre o capital inicial
emprestado ou aplicado.
JUROS COMPOSTOS: o juro de cada intervalo de tempo calculado a partir do saldo no incio de
correspondente intervalo. Ou seja: o juro de cada intervalo de tempo incorporado ao capital inicial
e passa a render juros tambm.

O juro a remunerao pelo emprstimo do dinheiro. Ele existe porque a maioria das pessoas prefere o consumo
imediato, e est disposta a pagar um preo por isto. Por outro lado, quem for capaz de esperar at possuir a quantia
suficiente para adquirir seu desejo, e neste nterim estiver disposta a emprestar esta quantia a algum, menos
paciente, deve ser recompensado por esta abstinncia na proporo do tempo e risco, que a operao envolver. O
tempo, o risco e a quantidade de dinheiro disponvel no mercado para emprstimos definem qual dever ser a
remunerao, mais conhecida como taxa de juros.
Quando usamos juros simples e juros compostos?
A maioria das operaes envolvendo dinheiro utiliza juros compostos. Esto includas: compras a mdio e longo
prazo, compras com carto de crdito, emprstimos bancrios, as aplicaes financeiras usuais como Caderneta de
Poupana e aplicaes em fundos de renda fixa, etc. Raramente encontramos uso para o regime de juros simples:
o caso das operaes de curtssimo prazo, e do processo de desconto simples de duplicatas.

22.2

Taxa de juros

A taxa de juros indica qual remunerao ser paga ao dinheiro emprestado, para um determinado perodo. Ela vem
normalmente expressa da forma percentual, em seguida da especificao do perodo de tempo a que se refere:
8 % a.a. - (a.a. significa ao ano).
10 % a.t. - (a.t. significa ao trimestre).
Outra forma de apresentao da taxa de juros a unitria, que igual a taxa percentual dividida por 100, sem o
smbolo %:
0,15 a.m. - (a.m. significa ao ms).
0,10 a.q. - (a.q. significa ao quadrimestre)

22.3 Clculo dos juros simples e do montante


Juros a remunerao paga pela aplicao de um capital.

Montante , por definio, a soma do capital com os juros.

Chamamos de regime de juros simples aquele onde se admite que os


juros so diretamente proporcionais ao tempo e a taxa da operao indicada. Neste
regime, s o capital rende juros.
J= C . i . t
100

M=C+J

Pra que possamos usar as frmulas acima, devemos nos lembrar que a taxa e o
tempo sempre devem ser expressos na mesma unidade de tempo.
__________________________________________________________________________________________________________________________

ATUALIZADO AT MAIO/2012

www.CARREIRAPUBLICA.com.br

(48) 4141-3220

171

4141-3222

____________

_____________________________ MATEMTICA _________________________________ Prof. Roberto

Assim, se temos a taxa expressa ao ms, por exemplo, devemos exprimir o tempo em meses, tambm, e viceversa.

22.4 Taxas proporcionais


Dizemos que duas taxas so proporcionais quando seus valores formam uma proporo direta com os respectivos
tempos, considerados numa mesma unidade. Desta forma, 60% a. e 5 a.m. so taxas proporcionais. Pois
60 % = 5 % aplicando o princpio fundamental das propores e considerando
12
o ano = 12 meses, teremos uma identidade
22.5 Taxas equivalentes
Dizemos que duas taxas so equivalentes quando produzem os mesmos juros, desde que sejam aplicadas ao
mesmo capital e ao mesmo perodo de tempo.
Convm lembrar que no regime de juros simples as taxas equivalentes sempre sero proporcionais.

22.6

Prazo mdio

Dado um conjunto com duas ou mais aplicaes a juros simples, cada qual com seus prprios valores de capital,
taxa e tempo, dizemos que prazo mdio um prazo nico tal que, substituindo os prazos de cada uma das
aplicaes dadas, produzir o mesmo total de juros das aplicaes originais.
Ex. Trs capitais de R$ 1.000,00, R$ 2.000,00 e R$ 3.000,00 foram aplicados s taxas de juros simples de 2%, 3% e
4% ao ms durante 3 meses, 2 meses e 1 ms, respectivamente. Qual o prazo mdio para estas trs aplicaes ?
PRAZOS
(meses)
3
2
1

CAPITAIS (R$)

TAXAS (%)

1000
2000
3000

2
3
4

PRODUTOS
3 . 1000 . 2 = 6000
2 . 2000 . 3 = 12000
1 . 3000 . 4 = 12000

PESOS
1000 . 2 = 2000
2000 . 3 = 6000
3000 . 4 = 12000

Prazo mdio = 6000 + 12000 + 12000 = 1,5 m


2000 + 6000 + 12000
Isso implica que, se trocssemos todos os prazos pelo prazo mdio, o juro total obtido pelas trs aplicaes
continuaria o mesmo.

22.7

Taxa mdia

Taxa mdia uma nica taxa tal que, substituindo as taxas de cada uma das aplicaes dadas, produzir o mesmo
juro total das aplicaes originais.
Utilizando os dados do exemplo anterior, temos:

TAXAS (%)

CAPITAIS (R$)

3
2
1

1000
2000
3000

PRAZOS
(MESES)
2
3
4

PRODUTOS
3 . 1000 . 2 = 6000
2 . 2000 . 3 = 12000
1 . 3000 . 4 = 12000

PESOS
1000 . 2 = 2000
2000 . 3 = 6000
3000 . 4 = 12000

Prazo mdio = 6000 + 12000 + 12000 = 1,5% a.m.


2000 + 6000 + 12000
22.8

Exerccios

172__________________________________________________________________________________________________________________________
ATUALIZADO AT MAIO/2012

www.CARREIRAPUBLICA.com.br

(48) 4141-3220

4141-3222

____________

_____________________________ MATEMTICA _________________________________ Prof. Roberto

- Calcule os juros anuais de R$ 6.000 a 6 % a. a.

R. R$ 360,00

- Calcule o juro mensal de R$ 40.000,00 taxa de 24 % a. a. R. R$ 800,00

- Qual a taxa anual que um capital de R$ 1.440,00 rende R$ 33,00 de juros em 2 meses e 15 dias ? R. 11 % a. a.

- Para que taxa um capital produz 1/5 do seu valor em 2 anos ? R. 10% a. a.

- Calcule o tempo para que uma quantia depositada a 12% a.a. triplique. R. 16 a e 8 m

Dois capitais so aplicados a 4% a. a. durante 8 meses e 3% a. a. durante 9 meses, respectivamente,


rendendo juros iguais. Calcule esses capitais sabendo que a diferena entre eles de R$ 125,00 R. R$ 675,00
e R$ 800,00

- Depositei certa importncia a 5% a. a. . No fim do primeiro ano, somei os juros ao capital e depositei a soma a
6% a. a., recebendo no fim do ano o juro de R$ 1.260,00. Que quantia foi inicialmente depositada ? R. R$
20.000,00

- Um capital de R$ 100,00 rendeu juros de R$ 10,80 em 90 dias. Quanto renderia em 12 meses, a uma taxa
mensal 0,1% maior do que a primeira ? (Banco do Brasil)
a) R$ 26,40 b) R$ 42,00 *c) R$ 44,40 d) R$ 55,20 e) R$ 79,20

- Se aplicarmos determinada quantia durante 8 meses, seu montante ser de R$ 63.000,00. Caso a aplicao
durasse 13 meses, seu montante seria de R$ 74.250,00. Qual a taxa mensal empregada ? (Banco do Brasil)
a) 4% *b) 5% c) 6% d) 7% e) 8%

10 - Um capital foi aplicado durante 5 meses taxa de 8% a. m. . Seu montante foi retirado e aplicado a 10% a. m.
durante 3 meses. Qual a taxa nica, na forma unitria, que poderia ser aplicado esse capital, durante o prazo
total para que no final desse prazo o montante fosse o mesmo?
a) 0,105 *b) 0,1025 c) 0,10 d) 0,0092 e) 0,087
11 - Uma pessoa coloca 2/3 dos seus haveres a 6% a. a. e o resto a 5% a. a., recebendo um juro anual de R$
340,00. Calcule o capital aplicado. R. R$ 6.000,00
12 - Um investidor aplicou 1/4 do seu capital a 36% a. a.; 2/3 do mesmo a 48% a. a. e o restante a 60% a.a. Aps 1
ano e 8 meses recebeu R$ 618.930,00 de juros. O capital inicial era: ( Fiscal de Tributos SC )
a) R$ 897.000,00 b) R$ 742.716,00 c) 742.776,50 *d) 807.300,00 e) 807.300,50
13 - Eu tinha certo capital para investir. Apliquei 2/3 dele a 9% a. m. durante 1 trimestre; 1/5 dele a 8% a.m. durante
dois bimestres e o restante 10% a. m. durante um certo prazo. Que prazo foi esse se o juro total obtido foi igual a
30% do capital que eu tinha para investir ? ( C.E.F. )
a) 6 m b) 5 m 3 d
c) 4,5 m * d) 4 m 6 d e) 4 m2 d
14 - Arno, Carlos, Dcio e Evaldo investiram um total de R$ 23.540,00 durante 1 ano, 1 ms e 10 dias, taxa de
54% a. a. . Ao receberem os juros, Carlos recebeu R$ 1.240,00 a mais que Arno; Dcio recebeu R$ 620,00 a
menos que Carlos e Evaldo, R$ 440,0 a menos que Dcio. Assinale a alternativa correta: ( Fiscal de Tributos
SC )
a) Arno investiu R$ 7.101,00 b) Arno investiu R$ 5.035,00
*c) Arno investiu R$ 5.335,00 d) Arno investiu R$ 3.021,00
e) Arno investiu R$ 7.335,00
15 - Um capital colocada, parte a 4% a. a. e parte a 5,5% a. a., d um juro anual de R$ 2.475,00. Se a parte
colocada a 4% a. a. fosse colocada a 5,5% a. a. e vice-versa, o juro seria de R$ 2.370,00. Calcule o capital total
investido. R. R$ 51.000,00
16 - Um capital aplicado a juros simples no dia 10 de fevereiro ao dia 24 de abril, do corrente ano, a uma taxa de
24 % ao ano. Nessas condies calcule o juro simples exato ao fim do perodo, como percentagem do capital
inicial, desprezando as casas decimais superiores segunda. (Auditor de Tributos Municipais Fortaleza -CE)
a) 4,70 % b) 4,75 % *c) 4,80 % d) 4,88 % e) 4,93 %
17 - Se no houvesse inflao e se a capitalizao dos rendimentos da caderneta de poupana fosse simples, a taxa
de juros seria ento de 5 % ao ms. Admitindo isso, o tempo t, em anos, necessrio para que um depsito em
__________________________________________________________________________________________________________________________

ATUALIZADO AT MAIO/2012

www.CARREIRAPUBLICA.com.br

(48) 4141-3220

173

4141-3222

____________

_____________________________ MATEMTICA _________________________________ Prof. Roberto

caderneta de poupana, aplicado taxa mensal de 0,5 %, produza juros simples iguais a 150 % de seu valor,
satisfaz condio (Ministrio Publico da Unio )
a) t < 10 b) 10 = t < 20 c) t = 20 *d) 20 < t < 30
e) t = 30
18 - Uma pessoa possui um financiamento (taxa de juros simples de 10% a.m.). O valor total dos pagamentos a
serem efetuados, juros mais principal, $ 1.400,00. As condies contratuais prevem que o pagamento deste
financiamento ser efetuado em duas parcelas. A primeira parcela, no valor de setenta por cento do total dos
pagamentos, ser paga ao final do quarto ms, e a segunda parcela, no valor de trinta por cento do total dos
pagamentos, ser paga no final do dcimo primeiro ms. O valor que mais se aproxima do valor financiado :
a) $ 816,55 * b) $ 900,00 c) $ 945,00 d) $ 970,00 e) $ 995,00
19 - Uma firma deseja alterar as datas e valores de um financiamento contratado.
Este financiamento foi contratado h 30 dias, a uma taxa de juros simples de
2% ao ms. A instituio financeira no cobra custas nem taxas para fazer
estas alteraes. A taxa de juros no sofrer alteraes.Condies pactuadas
inicialmente: Pagamento de duas prestaes iguais e sucessivas de $
11.924,00, a serem pagas em 60 e 90 dias.
Condies desejadas: O pagamento em trs prestaes iguais - a primeira no
final do10 ms; a segunda ao final do 30 ms; a terceira ao final do 70 ms.
Caso sejam aprovadas as alteraes, o valor que mais se aproxima do valor
unitrio de cada uma das novas prestaes :
a) $ 8.200,00 b) $ 9.333,33 c) $ 10.752,31 * d) $ 11.200,00 e) $ 12.933,60
19 - Uma pessoa deposita uma determinada importncia numa instituio financeira. No final de trs meses, ao
encerrar sua conta, verificou que o valor inicialmente depositado, acrescido dos juros creditados, totalizava R$
11.500,00. Esse valor , ento, integralmente depositado, em outra instituio financeira por um prazo de cinco
meses. No final desse perodo o montante acumulado na segunda instituio financeira totalizava R$ 14.375,00.
Sabendo-se que ambas as instituies remuneram seus depsitos a juros simples e a uma mesma taxa,
determinar o valor do depsito inicial na primeira instituio e a taxa de juro das duas instituies.R.: 5% e 10.000
20 Um investidor aplicou 5/6 do seu capital a juros simples comerciais, de 36 % a. a. , durante 4 meses e o
restante do capital tambm aplicou a juros simples comerciais, taxa de 72 % a. a., durante 8 meses. Qual o
valor do capital inicial total se a soma dos montantes recebidos das duas aplicaes totalizou R$ 212.400,00 ? (
AFTN )
a) R$ 160.000,00 b) R$ 192.000,00 c) R$ 168.000,00
*d) R$ 180.000,00 e) R$ 200.000,00
21 - Se 6/8 de uma quantia produzem 3/8 desta mesma quantia de juro simples, em 4 anos, qual a taxa unitria
aplicada ?
a) 0,12
b) 0,15 * c) 0,125 d) 0,128 e)N.R.A.
22 - Trs capitais so colocados a juro simples, o primeiro a 25% aa, durante 4 anos; o segundo a 24% a.a, durante
42 meses e o terceiro a 20% aa durante 2 anos e 4 meses, perfazendo um rendimento total de R$ 27.591,80 .
Sabendo que o segundo capital o dobro do primeiro e que o terceiro o triplo do segundo, calcule o valor do
terceiro capital. R. R$ 30.210,00
23 - H cinco meses passados, um capital de R$ 1.500,00 foi aplicado taxa de 7% ao ms. Se aplicarmos hoje um
capital de R$ 1.800,00 taxa de 10% ao ms, daqui a quantos meses os dois capitais tero produzido juros
iguais ? R.: 7
24 - Um capital aplicado do dia 5 de maio ao dia 25 de novembro do mesmo ano, a uma taxa de juros simples
ordinrio de 36% ao ano, produzindo um montante de R$ 4.800,00. Nessas condies, calcule o capital aplicado,
desprezando os centavos. (AFTN 98)
a) R$ 4.067,00 b) R$ 4.000,00 c) R$ 3.996,00 *d)R$ 3.986,00 e) R$ 3.941,00
25 - A quantia de R$ 10.000,00 foi aplicada a juros simples exatos do dia 12 de abril ao dia 5 de setembro do
corrente ano. Calcule os juros obtidos, taxa de 18% ao ano, desprezando os centavos.(AFTN 98)
*a) R$ 720,00 b) R$ 725,00 c) R$ 705,00 d) R$ 715,00 e) R$ 735,00
27- Os capitais de R$3.000,00, R$5.000,00 e R$ 8.000,00 foram aplicados todos no mesmo prazo, a taxas de
juros simples de 6% ao ms, 4% ao ms e 3,25% ao ms, respectivamente. Calcule a taxa mdia de aplicao
desses capitais. (AFTN 2002)
a) 4,83% a.m. b) 3,206% a.m. c) 4,4167% a.m. * d) 4% a.m. e) 4,859% a.m.

174__________________________________________________________________________________________________________________________
ATUALIZADO AT MAIO/2012

www.CARREIRAPUBLICA.com.br

(48) 4141-3220

4141-3222

____________

_____________________________ MATEMTICA _________________________________ Prof. Roberto

28 - Durante o ms de abril, um capital de R$ 20.000,00 foi colocado no open market (sistema de juros simples) pelo
prazo de 24 dias, tendo produzido um montante de
R$ 24.800,00. A taxa anual de juros simples a que esse
capital esteve aplicado foi de: (TCI PI/2000)
a) 30% b) 80% c) 120% *d) 360% e) 720%
29 - Um aplicador investiu R$ 12.000,00 numa instituio financeira, no perodo de 6 meses, taxa de juros simples
de 24 % ao ano. O montante recebido foi de (Ass. Adm. BRDES 2001)
a) R$ 12.640,00 *b) R$ 13.440,00 c) R$ 16.800,00 d) R$ 25.440,00 e) R$ 29.280,00
30 - Uma pessoa aplicou o valor de R$ 3.000,00 no mercado financeiro e, aps 12 dias, recebeu juros de R$ 72,00. A
taxa de juros simples dessa aplicao foi de: (Ass. Adm BRDES 2001 )
a) 0,06% ao ms. b) 0,06% ao dia. c) 0,6% ao ms. d) 0,6% ao dia. *e) 6 % ao ms.
31 - Os capitais de R$ 20.000,00, R$ 30.000,00 e R$ 50.000,00 foram aplicados mesma taxa de juros simples
mensal durante 4, 3 e 2 meses respectivamente. Obtenha o prazo mdio de aplicao desses capitais. (AFTN 98)
a) Dois meses e meio b) Trs meses e dez dias * c) Dois meses e vinte e um dias
d) Trs meses e nove dias e) Trs meses
32- (AFPS-2002) Uma pessoa fsica recebeu um emprstimo de um banco comercial no valor de R$ 10.000,00 por
um prazo de trs meses para pagar de volta este valor acrescido de 15% de juros ao fim do prazo. Todavia, a pessoa
s pode usar em proveito prprio 75% do emprstimo, porque, por fora do contrato, usou o restante para fazer uma
aplicao no prprio banco que rendeu R$ 150,00 ao fim dos trs meses. Indique qual foi a taxa efetiva de juros paga
pela pessoa fsica sobre a parte do emprstimo que utilizou em proveito prprio.
a) 12% ao trimestre
b) 14% ao trimestre
c) 15% ao trimestre
d) 16% ao trimestre
e) 18% ao trimestre X

23 Desconto simples
Desconto o abatimento que uma dvida sofre quando ela paga antes do vencimento.
O documento que atesta uma dvida chamado genericamente de ttulo de crdito.
So exemplos de ttulos de crdito as notas promissrias, duplicatas e as letras de cmbio.
Valor nominal o valor do ttulo de crdito, ou seja, o seu valor de face.
Valor atual o valor pelo qual o ttulo acabou de ser negociado. De outra forma, o valor pago.
Quando tomamos como referncia o valor nominal, temos o desconto comercial ou por fora.
Caso a referncia seja o valor lquido, temos o desconto racional ou por dentro.
Quando no se menciona o tipo de desconto, adota-se o comercial.

Temos, ento
D=V.i.t
100

d= V. i . t
100 + i t

Onde D = desconto comercial d = desconto racional

i = taxa t = tempo

23.1 Exerccios
1

- A que taxa anual uma letra de R$ 250,00, paga 25 dias do vencimento, foi descontada se o desconto obtido foi
de R$ 5,00 ? R. 28,8 % a.a.
__________________________________________________________________________________________________________________________

ATUALIZADO AT MAIO/2012

www.CARREIRAPUBLICA.com.br

(48) 4141-3220

175

4141-3222

____________

_____________________________ MATEMTICA _________________________________ Prof. Roberto

- Calcule o valor atual de uma letra de R$ 17.400,00 descontada a 6 7/8 % a.a. , 1 ano 2 meses e 4 dias antes do
vencimento. R. R$ 15.991,09

- Calcule o desconto racional de uma letra de R$ 250.000, a 6% a. a., que paga a 7 de agosto, sendo a sua
data de vencimento no dia 18 de outubro mesmo ano.

- Um ttulo de R$ 182.700,00 descontado a 9% a. a., 60 dias antes do vencimento. Calcule a diferena entre os
descontos comercial e racional. R. R$ 40,50

- Calcule o tempo que foi antecipado o pagamento de um ttulo de R$ 25.600,00, a uma taxa de 6% a. a., sendo
que foi pago R$ 23.722,70. R. 1a 2m 20 d

- Uma duplicata de R$ 180.900,00, a 6% a. a., foi paga um ms antes do vencimento. Calcule o desconto
racional. R. R$ 900,00

- Calcule a diferena entre os descontos comercial e racional, a 8% a. a., sobre um ttulo de R$ 12.000,00 pago
50 dias antes do vencimento. R. R$ 1,46

- Uma letra sofre um desconto racional a 9% a. a. , de R$ 3.600,00, com vencimento para 60 dias. Calcule o
desconto comercial. R. R$ 3.654,00

- Uma letra de R$ 1.200,00, com vencimento para 15 de agosto, foi paga em 6 de julho, com desconto comercial
de 9 % a. a. Calcule o valor pago. R. R $ 1.187,33

10 - Voc desconta em um banco uma letra de R$ 90.000,00, 40 dias antes do vencimento, a 3 % a. a. e paga ao
banco 1 1/8% sobre o valor nominal de comisses. Qual o valor lquido produzido pelo ttulo ?
11 - Uma Nota Promissria no valor de R$ 5.300,00 foi comprada, numa financeira, por R$ 5.000,00. Se a taxa de
juros simples exigida pelo comprador foi de 18% ao ano, sob o critrio do desconto racional, ento o vencimento
dessa Nota Promissria era de (Ass. Adm BRDES 2001)
a) 2 meses. b) 2 anos. c) 3 meses. d) 3 anos. e) 4 meses.

12 -Uma empresa devedora, em um banco, de dois ttulos de crdito, um no valor de R$


1.000,00 vencvel em 2 meses e outro no valor de R$ 3.000,00 vencvel em 6 meses. O
banco, cuja taxa de juros de 12 % ao ano, aceita a liquidao da dvida em um pagamento
nico vencvel em 8 meses. Adotando o critrio do desconto comercial simples, o valor desse
pagamento (Ass. Adm BRDES 2001)
a) R$ 3.680,60 b) R$ 3.800,00 c) R$ 4.130,43 d) R$ 4.500,80
e) R$ 5.000,00
13 - a promissria de R$240.000,00 descontada em um banco 60 dias antes do vencimento pelo desconto
comercial simples, aplicando-se uma determinada taxa de desconto. Se a operao resulta em uma taxa linear
efetiva de desconto de 12,5% ao ms, a taxa mensal de desconto comercial simples praticada pelo banco de
(TCI RJ 1999)
a) 15,0 % b) 10,0 % c) 9,5 % d) 8,5 % e) 6,5 %
14 - O desconto comercial simples de um ttulo quatro meses antes do seu vencimento de R$ 600,00. Considerando uma taxa
de 5% ao ms, obtenha o valor correspondente no caso de um desconto racional simples. (AFTN 1998)

a) R$ 400,00

b) R$ 600,00

c) R$ 800,00

d) R$ 700,00

e) R$ 500,00

15 - O desconto racional simples de uma nota promissria, cinco meses antes do vencimento,
de R$800,00, a uma taxa de 4% ao ms. Calcule o desconto comercial simples
correspondente, isto , considerando o mesmo ttulo, a mesma taxa e o mesmo prazo.(AFTN
2002)
a) R$ 960,00 b) R$ 666,67 c) R$ 973,32 d) R$ 640,00 e) R$ 800,00

176__________________________________________________________________________________________________________________________
ATUALIZADO AT MAIO/2012

www.CARREIRAPUBLICA.com.br

(48) 4141-3220

4141-3222

____________

_____________________________ MATEMTICA _________________________________ Prof. Roberto

16 - Uma empresa descontou em um banco uma duplicata de R$2 000,00 dois


meses e meio antes do seu vencimento, a uma taxa de desconto comercial de
4%am. A taxa efetiva de juros da operao no perodo foi: (ICMS 2000)
a) 10% b) 10,44% c) 10,77% d) 11,11% e) N.R.A.
17- (AFPS-2002) Um ttulo no valor nominal de R$ 10.900,00 deve sofrer
um desconto comercial simples de R$ 981,00 trs meses antes do seu
vencimento. Todavia uma negociao levou a troca do desconto comercial por
um desconto racional simples.
Calcule o novo desconto, considerando a mesma taxa de desconto mensal.
a) R$ 890,00 b) R$ 900,00 c) R$ 924,96 d) R$ 981,00 e) R$ 1.090,00

24 JUROS COMPOSTOS
Chamamos de regime de juros compostos aquele onde os juros de cada perodo so calculados sobre
o montante do perodo anterior.
n
S=P(1+i)

S = Montante P = Principal (Capital)


i = Taxa (unitria) n = N de perodos

24.1 Taxa nominal e taxa efetiva


Podemos definir taxa de juros como sendo a relao entre os juros cobrveis ou pagveis no fim de
um perodo anual, mensal, etc.) e a soma monetria devida no incio do perodo.
Sendo i a taxa de juros por perodo de capitalizao e havendo n perodos de capitalizao por ano, a
taxa de juros dita nominal quando o perodo de capitalizao no coincide com a unidade de tempo expressa pela
taxa. Por exemplo, a taxa de 12 % a.a. com capitalizao mensal, uma taxa nominal. As taxas nominais so muito
usadas como meio de exprimirmos a taxa de juros de uma forma mais simples para o entendimento.
Se a taxa for de 1 % a.m. com capitalizao mensal, temos uma taxa efetiva. Temos, nesse caso, que o perodo de
capitalizao coincide com a unidade de tempo da taxa.
Exemplos de taxa nominal: 24 % a.a., com capitalizao trimestral.
30 % a.m., com capitalizao diria
Exemplos de taxa efetiva: 2 % a.m., com capitalizao mensal
12 % a.a., com capitalizao anual.

24.2 Taxas equivalentes


Sempre que utilizamos as frmulas, devemos utilizar taxas efetivas. Contudo, as taxas nem sempre se
apresentam de maneira que se possa aplicar diretamente nas frmulas para o clculo.
Assim sendo, temos que achar uma taxa equivalente.

A relao de equivalncia entre as taxas dada abaixo


2

12

360

( 1 + i a) = ( 1 + i s) = ( 1 + i q) = ( 1 + i t) = ( 1 + i b) = ( 1 + i m) = ( 1 + id)
ia = taxa anual is = taxa semestral iq = taxa quadrimestral it = taxa trimestral
ib = taxa bimestral im = taxa mensal id = taxa diria
__________________________________________________________________________________________________________________________

ATUALIZADO AT MAIO/2012

www.CARREIRAPUBLICA.com.br

(48) 4141-3220

177

4141-3222

____________

_____________________________ MATEMTICA _________________________________ Prof. Roberto

24.3 Exerccios
1 ) Calcule o montante produzido por um capital de UM$ 1.000,00 aplicado a 2 % a.m. capitalizado mensalmente,
durante 10 meses. R.: UM$ 1.218,99
2) Calcule o montante produzido por um capital de UM $ 5.000,00 aplicado a 36 % a.a., capitalizado mensalmente,
durante um semestre. R.: UM $ 5.970,26
3) No final de 2 anos, uma pessoa dever efetuar um pagamento de UM $ 200.000,00 referente ao valor de um
emprstimo contrado hoje, mais os juros devidos, correspondentes a uma taxa de 4% a.m., capitalizados
mensalmente. Calcule o valor do emprstimo. R.: UM $ 78.024,29
4) Um capital de UM $ 300.000,00 foi aplicado a 3 % a.m., capitalizado mensalmente, produzindo um montante de
UM $ 427.728,26. Qual o tempo em que esteve aplicado?
R.: 1 ano
5 ) Um capital de UM $ 100.000,00 foi aplicado a 5 % a.m., capitalizado mensalmente, produzindo um montante de
UM $ 322.509,44. Qual o tempo em que esteve aplicado? R.: 2 anos
6 ) O montante de UM $ 296.048,85 foi obtido atravs da aplicao de um capital de UM $ 200.000,00 durante 10
meses. Qual foi a taxa mensal de aplicao ?
R.: 4 % a.m.
7 ) Um capital de UM $ 100.000,00 aplicado durante 1 ano produziu um montante de UM $ 179.585,93. A que taxa
mensal esteve aplicado ? R.: 5 % a.m.
8) Calcule o montante obtido atravs da aplicao UM $ 4.000,00, a juros compostos de 6,3 % a. a., ao final de 4
anos.
9) Calcule o montante obtido, pela aplicao de UM $ 1.000.000,00, aplicado a 1,8 % a. m. , capitalizados
mensalmente, durante 2 anos.
10) Calcule o montante produzido pela aplicao de um capital de UM $ 5.000,00 durante 2 anos, a uma taxa de 2,5
% a.b., capitalizados bimestralmente.
11) Calcule o montante obtido atravs da aplicao de UM $ 2.000,00, a 5% a.a. , a juros compostos, durante 5 a 8
m.
12) Calcule o capital aplicado a 5 % a. a., a juros compostos, sendo que o montante obtido no fim de 4 a 2 m 20 d foi
de UM $ 4.000,00.
13) Calcule o montante produzido por um capital de R$ 3.000,00, a juros compostos semestralmente a 6% a.a.,
durante 12 anos. R. R$ 6.098,37
14) Qual a taxa anual equivalente a 2,956 % semestrais?
15) Calcule as taxas semestrais equivalente e proporcional a 6 % a. a.
16) Coloquei a metade de certo capital a 5 % a. a., capitalizados semestralmente, durante 4 anos.Emprestei a outra
metade a juros simples durante o mesmo perodo a 6 % a. a..
Podemos afirmar que:
a) O melhor negcio foi o emprstimo a juros compostos.
b) O emprstimo a juros simples rendeu 10 % a mais que o feito a juros compostos.
c) O melhor negcio foi no emprstimo a juros simples.*
d) O emprstimo a juros compostos gerou 50 % dos juros obtidos pela aplicao do total do capital.
17) No trmino de 9 anos, retirei um montante de R$ 270.000,00 resultante da aplicao de R$ 150.000,00 Calcule a
taxa.
18) Uma pessoa aplicou um capital de R$ 20.000,00 durante 4 anos taxa nominal de 14% ao ano
capitalizada
semestralmente. Ao trmino desse perodo, somente os juros ganhos, foram reaplicados por 15 meses taxa
nominal de 12% ao trimestre capitalizada mensalmente. Qual o rendimento dessa ltima aplicao? (TCI RJ 1999)
a) R$ 10.308,29 b) R$ 11.504,53 c) R$ 12.718,97

178__________________________________________________________________________________________________________________________
ATUALIZADO AT MAIO/2012

www.CARREIRAPUBLICA.com.br

(48) 4141-3220

4141-3222

____________

_____________________________ MATEMTICA _________________________________ Prof. Roberto

d ) R$ 12.856,78 e) R$ 13.082,56
19) Dois capitais foram aplicados pelo prazo fixo de 2 anos. O primeiro taxa nominal de 20% ao ano capitalizada
semestralmente e o segundo, a 16% ao semestre capitalizada trimestralmente. Sabendo-se que ao trmino do prazo
os juros ganhos pelos dois capitais totalizaram R$ 2.042,14, e que o primeiro capital R$ 1.000,00 maior do que o
segundo, o valor de cada capital ,respectivamente, (TCI RJ 1999)
a) R$ 2.000,00 e R$ 1.000,00 b) R$ 2.180,00 e R$ 1.180,00
c) R$ 2.200,00 e R$ 1.200,00 d) R$ 2.240,00 e R$ 1.240,00
e) R$ 2.280,00 e R$ 1.280,00
20) Um capital foi aplicado por dois anos a juros efetivos compostos de 2% ao ano. No trmino desse prazo, um tero
dos juros ganhos foram reaplicados taxa efetiva composta de 5% ao ano, obtendo-se uma remunerao de R$
6.368,25 ao fim de 3 anos. Qual o valor do capital inicialmente aplicado? (TCI- 1999)
a) R$ 2.000.000,00 b) R$ 2.900.000,00 c) R$ 3.000.000,00
d) R$ 3.100.000,00 e) R$ 3.120.000,00
21) Indique qual a taxa de juros anual equivalente taxa de juros nominal de 8% ao ano com capitalizao
semestral. (AFTN 1998)
a) 8,20% b) 8,16% c) 8,10% d) 8,05% e) 8,00%
22) O capital de R$ 1.000,00 aplicado do dia 10 de junho ao dia 25 do ms seguinte, a uma taxa de juros
compostos de 21% ao ms. Usando a conveno linear, calcule os juros obtidos, aproximando o resultado em real.
(AFTN 1998)
a) R$ 331,00 b) R$ 340,00 c) R$ 343,00 d) R$ 342,00 e) R$ 337,00
23) Indique a taxa de juros anual equivalente taxa de juros nominal de 12% ao ano com capitalizao mensal.
(AFTN 2002)
a) 12,3600% b) 12,6825% c) 12,4864% d) 12,662%
e) 12,5508%
24) Um capital aplicado a juros compostos durante seis meses e dez dias, a uma taxa de juros de 6% ao ms. Qual
o valor que mais se aproxima dos juros obtidos como porcentagem do capital inicial, usando a conveno linear?
(AFTN 2002)
a) 46,11% b) 48,00% c) 41,85% d) 44,69% e) 50,36%
25) Um investidor aplicou R$ 10.000,00 em uma instituio de crdito que paga 10 % ao ms, no regime de
capitalizao composta. Se o juro recebido foi de R$ 3.310,00, o perodo em que o capital esteve aplicado foi de
(Ass. Adm. BRDES 2001)
2 meses. b) 3 meses. c) 4 meses e) 5 meses. e) 6 meses.
26) Uma pessoa deseja comprar um imvel. Para isso ela deposita a quantia de R$ 16.850,00 numa aplicao
financeira, taxa de juros compostos de 20 % ao ano capitalizados semestralmente. Em 6 anos, essa pessoa ter o
montante, desconsiderando-se os centavos, de (Ass. Adm. BRDES 2001)
R$ 29.841,00 b) R$ 45.000,00 c) R$ 50.297,00 d) R$ 52.882,52 e) R$ 55.000,00
27) 02. Aps a data de seu vencimento, uma dvida submetida a juros compostos com taxa mensal de 8%, alm de
ser acrescida de uma multa contratual correspondente a 2% da dvida original. Sabendo-se que log102 = 0,30 e
log103 = 0,48 e utilizando-se para todo o perodo o sistema de capitalizao composta, determine o tempo mnimo
necessrio, em meses, para que o valor a ser quitado seja 190% maior do que a dvida original. (C.E.F.)
a) 24
b) 23,5
c) 13
d) 11,5 X
e) 10
28) A taxa efetiva anual de 50%, no sistema de juros compostos, equivale a uma taxa nominal de i % ao semestre,
capitalizada bimestralmente. O nmero de divisores inteiros positivos de i (C.E.F.)
a) 4 X
b) 5
c) 6
d) 7
e) 8

__________________________________________________________________________________________________________________________

ATUALIZADO AT MAIO/2012

www.CARREIRAPUBLICA.com.br

(48) 4141-3220

179

4141-3222

____________

_____________________________ MATEMTICA _________________________________ Prof. Roberto

29- (AFPS-2002) Obtenha os juros como porcentagem do capital aplicado taxa de juros compostos de 10% ao
semestre por um prazo de quinze meses, usando a conveno linear para clculo do montante.
a) 22,5%
b) 24%
c) 25%
d) 26,906%
e) 27,05% X
30 - (AFPS-2002) Calcule o montante obtido ao fim de dezoito meses por um capital unitrio aplicado a uma taxa de
juros nominal de 36% ao ano com capitalizao mensal.
a) 1,54
b) 1,7024 X
c) 2,7024
d) 54%
e) 70,24%
TABELA I

FATOR DE ACUMULAO DE CAPITAL

an = (1 + i)n

1
2
3
4
5

i
n 1%
1,010000
1,020100
1,030301
1,040604
1,051010

2%
1,020000
1,040400
1,061208
1,082432
1,104081

3%
1,030000
1,060900
1,092727
1,125508
1,159274

4%
1,040000
1,081600
1,124864
1,169858
1,216652

5%
1,050000
1,102500
1,157625
1,215506
1,276281

6%
1,060000
1,123600
1,191016
1,262476
1,338225

7%
1,070000
1,144900
1,225043
1,310796
1,402552

8%
1,080000
1,166400
1,259712
1,360488
1,469329

9%
1,090000
1,188100
1,295029
1,411581
1,538624

10%
1,100000
1,210000
1,331000
1,464100
1,610510

12%
1,120000
1,254400
1,404928
1,573519
1,762341

15%
1,150000
1,322500
1,520875
1,749006
2,011357

18%
1,180000
1,392400
1,643032
1,938777
2,287758

6
7
8
9
10

1,061520
1,072135
1,082856
1,093685
1,104622

1,126162
1,148685
1,171659
1,195092
1,218994

1,194052
1,229873
1,266770
1,304773
1,343916

1,265319
1,315931
1,368569
1,423311
1,480244

1,340095
1,407100
1,477455
1,551328
1,628894

1,418519
1,503630
1,593848
1,689478
1,790847

1,500730
1,605781
1,718186
1,838459
1,967151

1,586874
1,713824
1,850930
1,999004
2,158925

1,677100
1,828039
1,992562
2,171893
2,367363

1,771561
1,948717
2,143588
2,357947
2,593742

1,973822
2,210681
2,475963
2,773078
3,105848

2,313061
2,660020
3,059023
3,517876
4,045558

2,699554
3,185474
3,758859
4,435454
5,233835

11
12
13
14
15

1,115668
1,126825
1,138093
1,149474
1,160969

1,243374
1,268242
1,293606
1,319479
1,345868

1,384233
1,425760
1,468533
1,512589
1,557967

1,539454
1,601032
1,665073
1,731676
1,800943

1,710339
1,795856
1,885649
1,979931
2,078928

1,898298
2,012196
2,132928
2,260903
2,396558

2,104852
2,252191
2,409845
2,578534
2,759031

2,331639
2,518170
2,719623
2,937193
3,172169

2,580426
2,812665
3,065804
3,341727
3,642482

2,853116
3,138428
3,452271
3,797498
4,177248

3,478549
3,895975
4,363493
4,887112
5,473565

4,652391
5,350250
6,152787
7,075706
8,137061

6,175926
7,287592
8,599359
10,147244
11,973748

16 1,172578 1,372786 1,604706 1,872981 2,182874 2,540351 2,952164 3,425942 3,970306 4,594972 6,130393 9,357621 14,129022
17 1,184304 1,400241 1,652847 1,947900 2,292018 2,692772 3,158815 3,700018 4,327633 5,054470 6,866040 10,761264 16,672246
18 1,196147 1,428246 1,702433 2,025816 2,406619 2,854339 3,379932 3,996019 4,717120 5,559917 7,689966 12,375453 19,673251

180__________________________________________________________________________________________________________________________
ATUALIZADO AT MAIO/2012

www.CARREIRAPUBLICA.com.br

(48) 4141-3220

4141-3222

Você também pode gostar